Download A 27-year-old HIV-positive man comes to the clinic for a periodic

Document related concepts

Prenatal testing wikipedia , lookup

Dental emergency wikipedia , lookup

Patient safety wikipedia , lookup

Medical ethics wikipedia , lookup

Electronic prescribing wikipedia , lookup

Adherence (medicine) wikipedia , lookup

List of medical mnemonics wikipedia , lookup

Transcript
A 27-year-old HIV-positive man comes to the clinic for a periodic health maintenance
examination. He contracted the disease 5 years ago from a former partner. He has been
followed in the community health clinic since that time. He has no other medical history
and takes only diazepam orally for anxiety. His last visit was 11 months ago. His
temperature is 37.0 C (98.6 F), blood pressure is 140/85 mm Hg, pulse is 78/min, and
respirations are 12/min. He has clear lung fields bilaterally, his skin is free of rashes or
excoriations, and his abdomen is soft and nontender. Blood work drawn a few weeks
ago reveals a CD4 count of 98 cells/mm3 and a hematocrit of 34% with an MCV of 95 fl.
His last tuberculin skin test was 3 months ago and was read as 4mm and flat. In addition
to initiating vitamin B12 and folate therapy for his patient, the most appropriate
intervention at this time is
A. antibiotic prophylaxis for PCP pneumonia
B. antibiotic prophylaxis for tuberculosis
C. a skin test for tuberculosis
D. treatment for active tuberculosis infection
E. none is indicated based upon his CD4 count at this time
Explanation:
The correct answer is A. Opportunistic infections occur in people with HIV and define
many of the components of the clinical syndrome known as AIDS. They are caused by a
wide variety of pathogens and all have the common etiology that the host is susceptible
due to the immune destruction brought on by the HIV virus. Prognosis depends on the
type of infection and often, even with appropriate therapy, morbidity and mortality is high.
There are means to prevent or reduce the likelihood of developing these infections. For
this patient, his CD4 count of less than 200 cells/mm3 indicates that he should begin
antibiotic prophylaxis, usually with TMP/SMX, for PCP pneumonia.
Starting antibiotic prophylaxis for tuberculosis (choice B) is not routine practice except in
persons with a PPD-positive skin test, which this patient does not have.
Since the patient was tested for tuberculosis within the last year, and there is no evidence
that he is anergic, there is no indication to test him again at this time (choice C).
The patient does not have active tuberculosis infection (choice D). Even if his PPD test
were positive, active infection requires documentation of the organism in sputum by PCR
or acid-fast staining.
Any patient with a CD4 count of less than 200 cells/mm3 should be considered for
prophylaxis therapy for at least PCP pneumonia and toxoplasmosis (choice E).
A 33-year-old man comes to the office complaining of 3 months of severe
headaches preceded by sweating and palpitations. He denies any chest pain
or shortness of breath with these episodes. He has not taken any new
medications and denies excessive caffeine intake and illicit drug use. His
temperature is 37.0 C (98.6 F), blood pressure is 120/70 mm Hg, pulse is
78/min, and respirations are 14/min. Physical and neurological examinations
are unremarkable. Thyroid function tests, a complete blood count, and a 24hour urine cortisol are all within normal limits. Urine catecholamines are
elevated. You order a CT scan of the abdomen, which shows an adrenal
mass. The most appropriate next step is to
A. begin therapy with phenoxybenzamine
B. begin therapy with propanolol
C. consult a surgeon for emergent laparoscopic adrenalectomy
D. order serial CT scans to evaluate progression
E. perform a fine needle aspiration of the mass
Explanation:
The correct answer is A. This patient most likely has a pheochromocytoma.
The classic triad of headache, palpitations, and diaphoresis should suggest
this diagnosis. Patients are frequently hypertensive but sometimes the
hypertension is paroxysmal and a normal blood pressure is found on exam.
Prior to surgery this patient must be started on an alpha-blocker
(phenoxybenzamine) to avoid intraoperative hypertensive crisis. Laparoscopic
adrenalectomy is the treatment of choice for a pheochromocytoma.
Treatment with a beta blocker (choice B) is initiated only after a patient is
started on an alpha blocker. Unopposed alpha stimulation can worsen
hypertension in patients with a pheochromocytoma.
Laparoscopic adrenalectomy (choice C) is the treatment of choice for
pheochromocytoma but medical stabilization is essential prior to surgery.
Pheochromocytoma is not an indication for emergent surgery.
Serial CT scans (choice D) are important in the management of nonfunctioning adrenal masses (incidentaloma). When an adrenal mass is
discovered incidentally and it is < 6cm repeat imaging is indicated to ensure
that the mass is not quickly growing. A rapidly growing adrenal mass is
suggestive of malignancy.
Fine needle aspiration (choice E) is an important tool in the workup of an
adrenal mass. However, prior to aspirating any adrenal mass,
pheochromocytoma must be ruled out to avoid precipitation of hypertensive
crisis. Fine needle aspiration is not indicated in the management of
pheochromocytoma since it can exacerbate a hypertensive crisis.
A 25-year-old man is admitted to the hospital after sustaining head injuries in a motor
vehicle accident. On his 2nd day in the hospital, he shows you a sore on his penis that he
developed a few days ago. He proudly admits to numerous sexual encounters in the
past 5 years, and tells you that he has been tested for HIV every 6 months, and that the
last negative test only was about 3 months ago. He is otherwise generally healthy, and
does not take any medications on a regular basis. He denies any penile discharge in the
past or present, and no history of other sexually transmitted diseases. On physical
examination, there is painful lymphadenopathy of the left groin region. On the distal
penis, there are 2 tender, ragged ulcers that appear punched out with surrounding
hyperemia. The base of the ulcers are covered with a purulent, dirty exudate, which
bleeds easily during the examination. This patient most likely has
A. chancroid and should be treated with azithromycin 1g orally in a single dose,
while cultures and tests for other sexual transmitted diseases are performed
B. gonorrhea dermatitis and he should receive therapy for both gonorrhea and
chlamydia, while cultures and tests for other sexual transmitted diseases are
performed
C. granuloma inguinale and should be treated with trimethoprim-sulfamethoxazole,
while cultures and tests for other sexual transmitted diseases are performed
D. lymphogranuloma venereum and should be treated with doxycycline, while
cultures and tests for other sexual transmitted diseases are performed
E. a primary syphilitic chancre and serum RPR should be checked to confirm the
diagnosis, while cultures and tests for other sexual transmitted diseases are
performed
Explanation:
The correct answer is A. Chancroid is correct because it usually presents as an
inflammatory papule that ruptures early with the formation of a ragged ulcer that lacks the
induration of a chancre. The ulcers have undermined irregular edges surrounded by mild
hyperemia, and the base is usually covered with purulent, dirty exudate. This is an
infectious, contagious, ulcerative, sexually transmitted disease caused by the Gramnegative bacillus Haemophilus ducreyi. It is characterized by 1 or more deep or
superficial tender ulcers on the genitalia and painful unilateral inguinal adenitis. However,
the diagnosis of chancroid does not rule out syphilis and the subsequent development of
syphilis should be anticipated since the incubation time for a chancre is longer than that
of chancroid.
Gonococcal dermatitis (choice B) is incorrect because it is a rare infection that occurs
mostly as erosions on the median raphe without urethritis. Grouped pustules on an
erythematous base is the usual presentation.
Granuloma inguinale (choice C) is incorrect because it is a mildly contagious, chronic,
granulomatous, locally destructive disease characterized by progressive indolent,
serpiginous ulcerations of the groins, pubis, genitalia, and anus. The disease begins as
single or multiple subcutaneous nodules, which erode through the skin to produce clean,
sharply defined lesions, which are usually painless. The lesions typically demonstrate
hypertrophic, vegetative granulation tissue which is soft, has a beefy-red appearance,
and bleeds readily. The regional lymph nodes are usually not enlarged. This is caused by
Calymmatobacterium granulomatis.
Lymphogranuloma venereum (choice D) is incorrect, because it is a sexually transmitted
disease characterized by suppurative inguinal adenitis with matted lymph nodes, inguinal
bubo with secondary ulceration, and constitutional symptoms. The primary lesion consists
of herpetiform vesicle or erosion develops on the glans penis followed by bilateral
lymphadenopathy. It is caused by Chlamydia trachomatis, serotypes L1, L2 and L3.
Primary syphilitic chancre (choice E) is incorrect, because it typically presents as a
crusted superficial erosion that becomes a round or oval, indurated, slightly elevated
papule, with an eroded, but not ulcerated surface that exudes a serous fluid. The lesion is
usually painless. The regional lymph nodes on one or both sides are usually enlarged,
firm, nontender and do not suppurate. With this said, when a patient presents with a
penile ulcer, it is wise to obtain a serum RPR since patients can often times have more
than one sexually transmitted disease.
A 66-year-old man with type II diabetes mellitus and atrial fibrillation comes to
the emergency department with right body weakness and slurred speech that
he noticed upon awakening in the morning. There were no complaints of word
finding difficulties and no dysesthesia. He smokes a pack of cigarettes a day
and "rarely exercises." His wife hands you a prescription medicine bottle of
warfarin and tells you that he has been taking this "for some time now." His
blood pressure is 210/95 mm Hg and his pulse is irregularly irregular. He has
left-sided neglect with slurred speech and weakness of the right body; face
and upper extremity worse than lower extremity. Routine chemistries and cell
counts are normal. His INR is 1.7. A CT scan of the head shows a large leftsided subdural hematoma. The most appropriate next step is to
A. administer fresh frozen plasma and vitamin K
B. give her intravenous labetalol immediately
C. order a brain MRI
D. start him on heparin
E. tell the family that he will die and do nothing further
Explanation:
The correct answer is A. You should reverse the warfarin. The risk of him
having a stroke from atrial fibrillation is far outweighed by the immediate goal
of stopping the intracranial bleeding.
This patient requires a high blood pressure to maintain cerebral perfusion. If
you drop his blood pressure with labetalol (choice B) this will cause his brain
to lose oxygen. Also, subdural bleeds are venous, so the high arterial blood
pressure would not be expected to increase the bleeding.
A brain MRI (choice C) will not provide additional useful information at this
time.
Heparin (choice D) is contraindicated during an intracranial bleed. The goal is
to reverse anticoagulation.
It is too early to predict outcome. You should not tell the family that he will die
and do nothing further (choice E).
A 27-year-old Caucasian man comes to the office because of an 8-month history of
fulminant episodes of chest palpitations, nausea, shortness of breath, and
hyperventilation. Each episode lasts approximately 5 minutes. The patient states that he
fears that he will die during these episodes and has had 4 emergency department
assessments in the last month with no organic etiology found. The patient's physical
examination, thyroid function tests, complete blood count, and metabolic workup are
within normal limits. The patient refuses psychiatric referral, but states that he will take
any medication prescribed by you to "make me feel less out of control". An appropriate
pharmacologic regimen for this patient may include
A. carbamazepine
B. haloperidol
C. lithium
D. methylphenidate
E. sertraline
Explanation:
The correct answer is E. The patient meets the criteria for a diagnosis of panic disorder.
The first line in treatment of panic disorder is selective serotonin reuptake inhibitors
(SSRIs), a group of medications including sertraline, paroxetine, fluoxetine, and
citalopram. Benzodiazepines may also be used during the titration of SSRI maintenance,
especially if panic attacks are disabling. Maintenance benzodiazepine treatment for panic
disorder, however, is discouraged due to risks of abuse, dependence, and potentially
serious withdrawal complications of long-term benzodiazepine treatment.
Carbamazepine (choice A) is an anticonvulsant that is also used for mood stabilization in
the treatment of bipolar disorder. It has no indication in the treatment of panic disorder.
Haloperidol (choice B) is an antipsychotic medication that is also commonly used to treat
agitation or combativeness regardless of etiology. It has no indication in the treatment of
panic disorder.
Lithium (choice C) is used primarily for mood stabilization in bipolar disorder and can also
be used as an adjunctive treatment in unipolar depression. It has no indication in the
treatment of panic disorder.
Methylphenidate (choice D) is a psychostimulant that is effective in the treatment of
attention deficit hyperactivity disorder and can also be used in the treatment of cachectic
medically ill depressed patients (such as those with terminal cancers and HIV). It has no
use in panic disorder and its stimulating qualities may actually exacerbate many anxiety
disorders.
You are called to the cardiac intensive care unit to evaluate a 73-year-old patient who
underwent an emergent cardiac catheterization for an acute myocardial infarction
yesterday. You are told that the patient has had a urine output of only 60 cc of urine over
the last 12 hours. Prior to this, his urine output had been within normal limits. His
temperature is 37 C (98.6 F), blood pressure is 130/70 mm Hg, pulse is 60/min, and
respirations are 14/min. He has no complaints at this time and his physical examination
is unremarkable. Reviewing the medical chart, you learn that his medical history is
significant for benign prostatic hypertrophy and hypertension. You catheterize his
bladder and get 20 cc of dark urine, which you send for urine analysis and culture. You
deliver a 500 cc normal saline fluid bolus and start intravenous fluid at 150 cc/hour. After
4 hours his urine output does not improve. Laboratory studies show:
Sodium
144 mEq/dL
Potassium 5.3 mEq/dL
Chloride
98 mEq/dL
Bicarbonate 21 mEq/dL
BUN
28 mg/dL
Creatinine 2.2 mg/dL (*Admission Creatinine- 1.3 mg/dL)
Urinalysis
Color
Muddy brown
Specific gravity
1.020
Osmolality
55 mOsmol/kg
Leukocyte esterase Negative
Nitrite
Negative
Protein
Trace
Blood
Negative
Microscopic
Many muddy brown granular casts
Urine eosinophils None
The most likely cause of this patient's new condition is
A. acute interstitial nephritis
B. acute tubular necrosis
C. postrenal azotemia
D. prerenal azotemia
E. rapidly progressive glomerulonephritis
Explanation:
The correct answer is B. This patient has acute tubular necrosis (ATN) and acute renal
failure. Remember that when confronted with acute renal failure, the cause is divided into
prerenal or "dry", postrenal or "obstructed", and intrarenal. The history and the urinalysis
provide us with the correct diagnosis. Patients with ATN can have muddy, granular casts
in their urine. They will also have an elevated BUN and creatinine, but in a ratio of less
than 20:1 as is seen in patients with prerenal azotemia. The history of recent cardiac
catheterization should also provide a clue since the patient received a large ionic dye
load during the procedure to image his coronary vessels. Dye is a common precipitant of
ATN and needs to be considered in anyone who has an elevated creatinine after a
procedure that involves contrast dyes.
Acute interstitial nephritis (AIN) (choice A) is another cause of acute renal failure. It is
associated with a transient maculopapular rash, fevers, eosinophiluria, hematuria, and
white cell casts. Drugs account for most cases of AIN.
Postrenal azotemia (choice C) would also be reasonable in our patient since he has a
history for BPH (which is the most common cause of obstruction). If a Foley catheter is
placed and a large amount of urine is found in the bladder, obstruction is a likely cause of
poor urine output. Since this patient had a virtually empty bladder, it would be unlikely
that he has a postrenal cause of his renal failure.
Prerenal azotemia (choice D) should be your first thought when evaluating a patient with
dressed urine output because it is the most common cause of acute renal failure. This
patient was given a fluid challenge and still did not increase his urine output. Therefore,
prerenal azotemia should be lower on the differential. Other clues that this patient is not
prerenal is that his BUN to creatine ratio is less than 20:1 and his urinalysis revealed
granular casts.
Rapidly progressive glomerulonephritis (choice E) is a rare cause of acute renal failure. It
is associated with hypertension and edema. Urinalysis reveals dysmorphic red blood
cells, red cell casts, and mild proteinuria.
A 64-year-old woman is brought to the emergency department after her husband called
911. The couple was eating dinner together when the patient suddenly experienced left
facial droop, left arm and leg weakness, and slurred speech. This occurred at
approximately 6:30 p.m. The husband, who suspected that his wife had had a stroke,
gave her an adult sized aspirin (325mg) before calling 911. She has a past medical
history significant for diabetes mellitus and has never had surgery. She takes metformin
for her diabetes and a daily multivitamin, but takes no other medications. Her family
history is significant for diabetes: her mother and both of her sisters have it. She is a
schoolteacher approaching retirement. She denies any smoking history. She rarely
drinks alcohol and denies any drug use in the past. On arrival to the hospital, her
temperature is 37.0 C (98.6 F), blood pressure is 160/85mm Hg, and pulse is 82/min.
She has normal lung and heart sounds. She is awake, alert, and fully oriented. Her
language is natural and fluent. She has a severe left facial droop and severe dysarthria.
She has grade 0/5 strength in her left arm and in her left leg. She has intact sensation
for fine touch and pinprick. Her glucose level is 95mg/dL, a platelet count of
355,000/mm3, a partial thromboplastin time of 21.0 sec, and an INR of 0.95. A CT of the
head, done without contrast, is interpreted by a radiologist as being normal. You
diagnose the patient with ischemic stroke and note that it is now 8:30 p.m. Your
neurological consultant agrees that there are no contraindications for thrombolysis. The
most appropriate treatment at this time is to
A. administer heparin, intravenously after a weight-appropriate loading bolus
B. administer tissue plasminogen activator, intravenously
C. give her aspirin 325mg, in addition to the dose she had been given at 6:30 p.m.
by her husband
D. give her clopidogrel 75mg
E. give her no additional treatment as she has already received a dose of aspirin
Explanation:
The correct answer is B. The patient has had an acute ischemic stroke. The Federal Drug
Administration (FDA) has approved IV-tPA for acute ischemic stroke provided there are
no contraindications and that the drug is given within 3 hours of symptom onset. This
approval, in 1996, was based on the results of the National Institutes of Neurologic
Disorders and Stroke (NINDS) study which showed that patients who received IV-tPA
after fulfilling certain criteria have significantly better outcomes than those who did not.
Examples of contraindications include hemorrhage on CT scan; early ischemic changes
on CT scan; uncontrolled hypertension; minor or resolving presenting symptoms; recent
GI hemorrhage, and/or abnormal PT, PTT, platelet count, or glucose. In this case, the
patient has severe stroke symptoms, has been worked up within 2 hours of symptoms
onset, and has no contraindications for the use of IV-tPA.
Heparin (choice A) is incorrect. There is no known role for heparin in the treatment of
acute stroke. Its use remains controversial today, and is not approved by the FDA for
acute stroke.
Additional aspirin (choice C) is incorrect. There is no proven role for multiple doses of
aspirin in the setting of acute stroke. The use of aspirin is based on the results of the
International Stroke Trial of 1997, which showed that 300mg daily of this drug showed a
modest benefit in reducing new neurologic symptoms 6 months after the first stroke.
Clopidogrel (choice D) is incorrect. There is no known role for clopidogrel in the treatment
of acute stroke. Its use in prevention of stroke is based on the European Stroke
Prevention Study (ESPS-II) of 1996, which showed a synergistic effect of aspirin and
dipyridamole (in the same class as clopidogrel) when used together, in the prevention of
stroke, but was never investigated in the setting of acute stroke.
No treatment (choice E) is incorrect. As mentioned above, aspirin is moderately effective
in the prevention of new strokes, but it is not adequate in the setting of an acute stroke,
especially in the case of a patient who qualifies for IV-tPA.
An 8-year-old girl is brought to the office for a well-child visit. She has no complaints at
this time. The mother is very concerned about her daughter's health because she has
been reading about the threats of bioterrorism on the Internet. She is particularly worried
about smallpox and anthrax. At this time you should
A. administer a vaccine for anthrax
B. explain that while there is no routinely recommended prophylaxis for smallpox
or anthrax, there are treatment options available if she acquires either of these
diseases
C. prescribe ciprofloxacin as a prophylaxis for anthrax
D. reassure the mother that her daughter has received all of the recommended
childhood immunizations and should be immune to the smallpox virus
E. tell the mother that you understand her concerns but that there is no prophylaxis
for either smallpox or anthrax
Explanation:
The correct answer is B. Anthrax and smallpox are potential biological weapons that can
be used for bioterrorism. Smallpox has been eradicated with vaccines and so the vaccine
is no longer part of the routinely recommended childhood immunizations. Cidofovir is a
treatment option for an established infection. Anthrax is a potentially deadly bacteria that
can cause cutaneous, gastrointestinal, and respiratory diseases. A vaccine is available,
but it is only routinely given to persons at a significant continuing risk (military personnel).
Ciprofloxacin is used for prophylaxis and treatment for anthrax in adults and penicillin is
used for children.
It is inappropriate to administer a vaccine for anthrax (choice A) because it is only
routinely given to persons at a significant continuing risk. This patient does not seem to
be at a "significant continuing risk" at this time.
Since there is no reason to believe that this patient is going to be exposed to anthrax, you
should not prescribe ciprofloxacin as a prophylaxis for anthrax (choice C). Also,
ciprofloxacin is not approved by the FDA for use in persons under 18 years of age. It may
be indicated, however, for an established serious or life-threatening infection.
Since the smallpox vaccine is not routinely given as part of the recommended childhood
immunizations in the United States, it is incorrect to reassure the mother that her
daughter has received all of the recommended childhood immunizations and should be
immune to the smallpox virus (choice D).
It is incorrect to tell the mother that you understand her concerns but that there is no
prophylaxis for either smallpox or anthrax (choice E) because as stated above, there is
prophylaxis for anthrax and there is an effective vaccine for smallpox, but neither of these
are indicated for this patient. According to the Centers of Disease Control in March 2001,
pre-exposure vaccination against smallpox is not recommended for any group other than
laboratory or medical personnel working with non-highly attenuated orthopoxviruses.
You are notified that one of your patients, a 3 -year-old previously healthy girl,
is admitted to the hospital because of 6-day history of fever, irritability, and
erythema of the hands and feet. Her physical examination on admission
showed a temperature of 39.7 C (103.4 F), bilateral conjunctival injection, an
enlarged left-sided cervical lymph node (2.0 cm), fissured lips, a red tongue
with red papillae, pharyngeal hyperemia, erythematous and edematous palms
and soles, and a confluent, blanching erythematous rash on the trunk. The
mother told the emergency department physician that she had been giving her
daughter aspirin for the past week to reduce her fever. In the emergency
department, intravenous fluids were started, the aspirin therapy was
continued, and laboratory studies were ordered. These laboratory studies just
returned and show an erythrocyte sedimentation rate of 28mm/h and a platelet
count of 490,000/mm3. The patient is extremely uncomfortable and now shows
desquamation of the fingers and toes. The mother is very concerned about
her daughter's condition. At this time the most correct statement about her
condition is:
A. Corticosteroids are necessary to decrease the risk of aneurysms within
10 days of the onset of fever
B. Influenza vaccination is necessary if this patient requires long-term
salicylate therapy
C. MMR vaccination should be given within a month if this patient receives
intravenous gamma globulin
D. Nasopharyngeal cultures will help to establish the diagnosis
E. There is a 40% risk of death associated with her disease
Explanation:
The correct answer is B. This patient most likely has Kawasaki disease, which
is treated with aspirin and intravenous gamma globulin. The disease is
characterized by a high fever for longer than 5 days, bilateral conjunctival
injection, fissured lips, a "strawberry tongue", mucosal change in the oral
pharynx, erythematous and edematous palms and soles with desquamation, a
polymorphous rash, cervical lymphadenopathy, an elevated erythrocyte
sedimentation rate, and thrombocytosis. The most important complication is
coronary artery aneurysms, which may be prevented by early treatment with
aspirin and intravenous gamma globulin. An echocardiogram is necessary to
evaluate cardiac involvement. The influenza vaccine is necessary if this
patient requires long-term salicylate therapy because of the possible
increased risk of Reye syndrome. Reye syndrome may develop in patients
with influenza who receive salicylates.
Corticosteroids are necessary to decrease the risk of aneurysms within 10
days of the onset of fever (choice A) is incorrect. Initially, intravenous gamma
globulin and aspirin are routinely used to decrease the risk of aneurysms in
patients with Kawasaki disease. Some believe that steroids should be used
later if therapy with intravenous gamma globulin and aspirin is unsuccessful.
But steroids are not necessary to decrease the risk of aneurysms.
MMR vaccination should be given within a month if this patient receives
intravenous gamma globulin (choice C) is incorrect. MMR vaccination should
be given 11 months after treatment with intravenous gamma globulin. It should
not be given sooner because intravenous gamma globulin interferes with the
serologic response to the measles vaccine. If there is a high risk that this child
will be exposed to measles, which is unlikely in the United States,
immunization should be given, and if at 11 months serologic testing indicates
that the earlier immunization was unsuccessful, she should be re-immunized.
It is incorrect to say that nasopharyngeal cultures will help to establish the
diagnosis (choice D) because no single study can diagnose Kawasaki
disease. The exact cause of this disease is unknown.
There is a 40% risk of death associated with her disease (choice E) is
incorrect. In the United States, the mortality rate due to Kawasaki disease is
0.05%. Death is usually due to a myocardial infarction or ruptured coronary
aneurysm.
A 71-year-old man with osteoarthritis comes to the office complaining of a painful "bandlike" rash across his left chest. He denies ever having a similar rash before. He plays
golf 3 times per week and takes only nonsteroidal antiinflammatory agents for pain from
his arthritis. His temperature is 37.0 C (98.6 F). On his left chest, in the T5 dermatomal
distribution, is a macular-papular, erythematous rash that is painful to the touch. There is
mild weeping of some of the papules. The most appropriate therapy is at this time is
A. antibiotics
B. antifungal agents
C. corticosteroids
D. ganciclovir
E. gabapentin
Explanation:
The correct answer is D. The patient has herpes zoster, also known as shingles. The
disease is a result of reactivation of latent varicella zoster virus in the dorsal root ganglia.
The disease follows a dermatomal distribution and is very painful. The goals of therapy
are to hasten the resolution of the symptoms and to prevent the development of
postherpetic neuralgia, an often crippling neuropathic pain disorder resulting from the
shingles infection. The drugs best able to accomplish both of these goals are the oral
antiretroviral drugs, ganciclovir, acyclovir, and famciclovir.
Neither oral antibiotics (choice A) nor oral antifungal agents (choice B) have any role in
the treatment of this disease, since it is a reactivation of a viral infection. Unless there is
evidence of a superinfection with a bacterial or fungal etiology, these classes of drugs
should be avoided.
The use of corticosteroids (choice C) for this disease will exacerbate the symptoms.
Although dermatologists liberally utilize steroid therapy, its use in this case acts as an
immunosuppressive agent and will exacerbate the primary manifestations of rash and
pain.
Gabapentin (choice E) is used for the treatment of postherpetic neuralgia and other
neuropathic pain syndromes. Gabapentin is structurally related to the neurotransmitter
GABA (gamma-aminobutyric acid) but it does not interact with GABA receptors, as it is
not converted metabolically into GABA or a GABA agonist, and it is not an inhibitor of
GABA uptake or degradation. It has no utility in treatment of acute shingles.
You are seeing a 41-year-old man with alcoholic cirrhosis in your office for a follow-up
visit after a recent upper endoscopy showed significant lower esophageal varices. His
current medications include a multivitamin, folate, and thiamine. While he strongly
denies any continued alcohol use, you are suspicious that he is still drinking. His blood
pressure is 100/63 mmHg, pulse is 98/min, and respirations are 21/min. Physical
examination shows a slightly protuberant abdomen. Given his varices, you are
concerned about an upper gastrointestinal bleed, especially in the setting of continued
alcohol use. Given this concern, the most appropriate pharmacotherapy to add to his
treatment regimen is
A. aspirin
B. atorvastatin
C. isosorbide mononitrate
D. nadolol
E. warfarin
Explanation:
The correct answer is D. Non-selective beta antagonists such as nadolol have been
shown to decrease the risk of an initial variceal bleed (through a reduction in splanchnic
blood flow) in someone with esophageal varices (primary prevention).
Aspirin (choice A), an antiplatelet agent, has no role in the primary prevention of variceal
bleeding.
Atorvastatin (choice B), an HMG Co-A Reductase antagonist used in the management of
hyperlipidemia, has no role in the primary prevention of variceal bleeding.
Nitrates such as isosorbide mononitrate (choice C) should not be started as monotherapy
for the primary prevention of variceal bleeding in cirrhotics since it has been associated
with increased mortality when used alone. However, it can be used in combination with
beta antagonists.
Warfarin (choice E), an oral anticoagulant against vitamin K dependent clotting factors,
has no role in the primary prevention of variceal bleeding.
A 16-year-old boy comes to the emergency department complaining of severe
headache, dizziness, and difficulty concentrating. The patient has been working as a
painter for the summer. He had been working in a well-ventilated bedroom, but he
mistakenly spilled some paint remover on himself. About 2 hours later, he began
developing a severe headache. He brought the paint remover with him and you look on
the label and see that methylene chloride is listed as one of the solvents. His blood
pressure is 110/80 mm Hg, pulse is 65/min, and respirations are 14/min. A neurological
examination is normal. His lips are very red and there is evidence of cyanosis of his
fingers. Pulse oximetry, however, shows an oxygen saturation of 100% on room air. An
arterial blood gas shows a carboxyhemoglobin level of 42% by co-oximetry. The most
appropriate next step in management is to
A. administer methylene blue, intravenously
B. begin continuous positive airway pressure ventilation
C. give him hyperbaric oxygen therapy immediately
D. give him 100% supplemental oxygen by non-rebreather mask
E. intubate him and provide pressure support ventilation
Explanation:
The correct answer is C. This is a case of severe carbon monoxide poisoning from
methylene chloride exposure. Methylene chloride is a solvent found in many paint
removers and is readily absorbed through exposed areas of the skin. Once in the
bloodstream, methylene chloride is metabolized in the liver and one by-product of its
metabolism is carbon monoxide. Carbon monoxide poisoning typically presents with
vague neurological complaints and history of exposure. In severe cases, carbon
monoxide poisoning can lead to seizure, coma, or death. Cherry-red lips are a classical
physical finding associated with this condition. A routine pulse oximeter cannot
distinguish carboxyhemoglobin from normal hemoglobin. Carboxyhemoglobin levels must
be measured by co-oximetry, which can detect the spectral differences between
carboxyhemoglobin and hemoglobin. Treatment consists of administering 100%
supplemental oxygen. Patients with carboxyhemoglobin levels greater than 40% require
hyperbaric oxygen therapy to improve tissue oxygenation and reduce carboxyhemoglobin
levels.
Methylene blue (choice A) is the treatment of choice for severe methemoglobinemia. It
has no use in carbon monoxide poisoning.
Continuous positive airway pressure (choice B) can improve oxygenation by recruiting
alveoli and stent open airways. It is useful in pulmonary causes of hypoxia, but does not
treat the underlying carboxyhemoglobinemia.
For routine cases of carbon monoxide poisoning, 100% supplemental oxygen (choice D)
is the treatment of choice. However, this patient has very high carboxyhemoglobin levels
and requires hyperbaric oxygen treatment.
The patient is able to maintain his airway and breathe on his own. If there are more
severe complications of carbon monoxide poisoning like seizure or coma, intubation
(choice E) may be required.
A previously healthy 21-year-old woman comes to the local college clinic
because of a headache and low-grade fevers. She is sent home with
acetaminophen and advised to return if she does not improve. Approximately
three hours later her roommate calls 911 reporting that her friend is
unconscious and not arousable. On arrival the paramedics find a lethargic,
febrile female lying on the floor and unresponsive. The patient is stabilized
and she is rushed emergently to the local hospital where an abdominal CT
scan shows bilateral adrenal hemorrhages. The patient is transported to the
intensive care unit where a pulmonary artery catheter is inserted via a right
internal jugular vein. Her temperature is 39.3 C (102.8 F), blood pressure is
85/40 mm Hg, and pulse is 140/min. An electrocardiogram demonstrates
sinus tachycardia. Her extremities are warm to the touch and appear pink. A
cardiac output (CO), systemic vascular resistance (SVR), pulmonary capillary
wedge pressure (PCWP) and central venous pressure (CVP) are obtained.
The data acquired from her PA-line is most likely to be
A. CO 5.0 L/min, SVR 800 dynes-sec/cm5, PCWP 12 mm Hg, CVP 4 mm
Hg
B. CO 9.5 L/min, SVR 210 dynes-sec/cm5, PCWP 8 mm Hg, CVP 6 mm
Hg
C. CO 2.3 L/min, SVR 1500 dynes-sec/cm5, PCWP 30 mm Hg, CVP 10
mm Hg
D. CO 3.0 L/min, SVR 1500 dynes-sec/cm5, PCWP 8 mm Hg, CVP 1 mm
Hg
E. CO 9.5 L/min, SVR 1210 dynes-sec/cm5, PCWP 12 mm Hg, CVP 6 mm
Hg
Explanation:
The correct answer is B. The patient likely has septic shock that is
characterized by high cardiac output, low systemic resistance, and a relatively
normal contractile function (PCWP) and filling pressures (CVP).
CO 5.0 L/min, SVR 800 dynes-sec/cm5, PCWP 12 mm Hg, CVP 4 mm Hg
(choice A) is a profile of a normal, healthy person.
CO 2.3 L/min, SVR 1500 dynes-sec/cm5, PCWP 30 mm Hg, CVP 10 mm Hg
(choice C) is a profile of cardiogenic shock. This patient has a depressed
cardiac output, a markedly elevated PCWP reflecting failure, and an
appropriately high peripheral vasoconstrictive response to maintain blood
pressure.
CO 3.0 L/min, SVR 1500 dynes-sec/cm5, PCWP 8 mm Hg, CVP 1 mm Hg
(choice D) reflects hypovolemic shock with a low output, an appropriately high
peripheral resistance but very low filling pressures.
CO 9.5 L/min, SVR 1210 dynes-sec/cm5, PCWP 12 mm Hg, CVP 6 mm Hg
(choice E) is representative of a healthy person undergoing vigorous exercise,
with augmented cardiac output, normal systemic resistance and filling
pressures.
A 29-year-old man comes to the clinic because he and his wife have "not been able to have a
baby". The patient states that he has been happily married for 4 years and he and his wife have
been trying to have a child for the last 13 months. He has never fathered a child and his wife
has never been pregnant. His wife has been evaluated by her physician and no abnormalities
were identified. Your patient denies any history of cryptorchidism, sexually transmitted
diseases, urinary tract infections, genital trauma, or erectile dysfunction. He has not received
any chemotherapy nor does he have any known genetic disorders. Physical examination
reveals a circumcised phallus without meatal discharge. Testicles are descended bilaterally,
and are normal in size and contour. There is a grade 3 varicocele on the left side. No varicocele
is identified on the right. On rectal examination the prostate is normal to palpation. Serum
testosterone, LH, and FSH are normal. You send the patient for semen analysis. The results
are as follows
At this time you should
A. advise them that no intervention is indicated
B. obtain a transrectal ultrasound
C. perform a testicular biopsy
D. prescribe a testosterone patch
E. refer them for in vitro fertilization
F. refer him for a varicocelectomy (ligation of varicocele)
Explanation:
The correct answer is F. A varicocele is defined as a dilated vein or set of veins in the
pampiniform plexus in the spermatic cord, and is the most common identifiable cause of male
factor infertility. It is present in 15% of the total male population, but is found in approximately
40% of men with male factor infertility. A varicocelectomy (the ligation of varicoceles) improves
semen quality in approximately two-thirds of men and doubles the chance of conception.
Varicoceles form secondary to incompetent or absent valves in the spermatic veins. This
valvular deficiency, combined with the long vertical course of the internal spermatic vein on the
left side, leads to the formation of most varicoceles on the left side. A unilateral right sided
varicocele suggests venous thrombosis (from a tumor) in the inferior vena cava. The effect of a
varicocele on fertility has to do with the prevention of efficient blood flow out of the scrotum.
There is pooling of blood in the pampiniform plexus, leading to an increase in scrotal
temperature and an adverse effect on spermatogenesis. Varicoceles tend to cause a "stress
pattern" on semen analysis. This is characterized by a low sperm concentration, low sperm
motility, low sperm count, and low sperm morphology. The technique for varicocelectomy is
varied and may be performed via an inguinal, retroperitoneal, subinguinal, laparoscopic, or
embolization approach. The subinguinal approach with aid of a microscope, (microscopic
varicocelectomy), is the approach with the fewest complications.
No intervention (choice A) is incorrect. Any couple who has been unable to conceive for over 1
year warrant investigation. In general, 90% of normal couples conceive within 1 year of trying.
Some investigators advocate a simple, basic, cost-effective evaluation of both male and female
at the time of presentation, no matter how long they have been attempting to conceive.
A transrectal ultrasound (choice B) is a valuable diagnostic tool when there is an obstructive
process causing infertility. The majority of semen volume comes from the prostate, seminal
vesicles, and Cowper's gland. The majority of ejaculated sperm comes from the distal
epididymis. Therefore, the normal volume of ejaculate and the fact that the patient is not
azoospermic (no sperm present) makes obstruction of the vas, seminal vesicles, and
ejaculatory ducts, unlikely. The patient also has a normal semen pH which is caused by the
fructose within the semen. Fructose is secreted by the seminal vesicles, therefore this patient's
normal semen pH makes seminal vesicle obstruction unlikely. In this scenario, the yield of
transrectal ultrasound to look for seminal vesicles or ejaculatory duct dilatation, is low.
Testicular biopsy (choice C) is indicated in azoospermic patients with normal FSH levels or in
patients with abnormal hormone parameters. This patient has neither.
The patient has normal hormone parameters, and there is no evidence that increasing his
testosterone (choice D) will improve his semen parameters.
Referring this couple for in vitro fertilization (choice E) is not the correct management. The male
in the relationship has a potentially reversible cause of his infertility (varicocele) and having
them go through the emotional and financial hardship of assisted reproduction is not
recommended prior to attempting correction of other causes of infertility.
A 10- year-old girl is brought to the clinic because of a 2-day history of a sore
throat and fever. The mother reports that the fever has been as high as 39 C
(102.2 F) and that the child is complaining of pain on swallowing. She has had
no rhinorrhea, cough, or ear pain, and no one else is ill at home. Physical
examination reveals a well-appearing girl with an exudative pharyngitis and
multiple 1-2-cm tender submandibular lymph nodes. There is no other
adenopathy, rash, or hepatosplenomegaly present. A rapid antigen detection
test for group A streptococcus done on a swab of the posterior pharynx is
positive. The most appropriate action at this time is to
A. perform a complete blood count and monospot test
B. repeat the rapid antigen detection test
C. send a throat swab for culture
D. treat with oral amoxicillin-clavulanate
E. treat with oral penicillin
Explanation:
The correct answer is E. The positive rapid test for group A streptococcus (S.
pyogenes) confirms the diagnosis of streptococcal pharyngitis, and antibiotic
therapy is essential for the prevention of suppurative complications (e.g.,
adenitis, peritonsillar abscess) as well as rheumatic fever. Penicillin remains
the treatment of choice for streptococcal pharyngitis in non-allergic patients,
as treatment failures are extremely rare.
Infectious mononucleosis is another potential cause of fever, pharyngitis, and
rash in children. Associated findings would be generalized lymphadenopathy
and splenomegaly on physical examination. The lack of these makes the
diagnosis unlikely in this case. A complete blood count and monospot (choice
A) can help confirm this diagnosis when it is suspected.
The rapid antigen detection tests have excellent specificity (>95%) and thus a
positive test does not need to be confirmed by repetition (choice B).
Most practitioners choose to send a swab for bacterial culture (choice C) in
the setting of a negative rapid test in order to increase sensitivity because of
the potentially devastating sequelae of a missed case of streptococcal
pharyngitis, but this is unnecessary with a positive test.
Amoxicillin-clavulanate (choice D) is a combination of a beta-lactam antibiotic
and a beta-lactamase inhibitor. It is useful in the treatment of infections due to
organisms that have developed resistance via the production of a betalactamase enzyme. Group A streptococcus does not produce a betalactamase. Thus, this combination is unnecessary in this case.
A 53-year-old man comes to the office because of a 2-day history of knee
pain. He says that he was relaxing with his family in the living room, and when
he got up to get a beer, he realized that he was unable to bear weight on his
left knee. When he pulled up his left sweatpant leg, he saw that his knee was
very swollen. He is normally very active and until yesterday, had participated
in a 3-times a week racquetball game with his business partner and a daily
basketball game with his sons on the court in their backyard. He states that he
recently returned from a 2-week business trip to Southeast Asia. He tells you
that his wife went to a health spa in California while he was travelling. He
drinks "a couple of beers" a few times a week, smokes about a pack of
cigarettes per day, eats a "fairly healthy diet" of low-fat, high-fiber foods, and
does not take any medications. He cannot recall a history of trauma, does not
have any other symptoms, and has never had any similar episodes of joint
pain in the past. Physical examination shows a tender, erythematous, and
swollen left knee with pain on flexion, and a limited range of motion. The most
appropriate question that will help to establish a diagnosis is:
A. "Did you have unprotected sexual intercourse on your trip to Southeast
Asia?"
B. "Did you receive the Hepatitis B series of vaccinations before your trip to
Southeast Asia?"
C. "Do you ever wake up in the morning with severe stiffness in your hands
that lasts for several hours?"
D. "Have you experienced headaches, scalp tenderness, jaw pain, or
visual changes?"
E. "Have you had any crampy abdominal pain and explosive, watery
diarrhea while on your trip or since you returned?"
Explanation:
The correct answer is A. This patient most likely has acute septic arthritis, due
to a gonococcal infection, most likely acquired during unprotected sexual
intercourse on his business trip. The clues in the history include the onset of
symptoms after he returned from his trip, and the lack of any other symptoms
such as fatigue, malaise, and anorexia. He has never had any similar
symptoms, and he does not have a history of trauma. He had led an active,
healthy life until this trip, and so it seems that it may be related to an
experience that occurred in Asia. An arthrocentesis with joint fluid analysis will
most likely show Gram-negative diplococci. In addition, cultures should be
taken from all potential sites such as the urethra, anus, and mouth and should
be performed on a Thayer-Martin medium. The treatment is cephalosporins.
While Hepatitis B (choice B) can lead to immune complex related arthritis,
without fatigue, malaise, anorexia, and a rash, this diagnosis is unlikely. Also
the incubation period for Hepatitis B is usually between 1-6 months.
Morning stiffness in the hands (choice C) is associated with rheumatoid
arthritis, which is systemic disease that usually does not present with acute
monoarticular arthritis. It is characterized by symmetric inflammation, bony
erosions, and joint deformities as a consequence of synovial inflammation.
Laboratory studies often show rheumatoid factor, an elevated erythrocyte
sedimentation rate, and anemia of chronic disease.
Headaches, scalp tenderness, jaw pain, and visual changes (choice D) are
the symptoms of temporal arteritis, which are often associated with
polymyalgia rheumatica. It typically occurs in elderly patients and is
characterized by proximal muscle pain, morning stiffness, fatigue, weight loss,
and a low-grade fever. These patients have an elevated erythrocyte
sedimentation rate and normal muscle enzymes. A temporal artery biopsy is
necessary if there are complaints of visual changes and headaches. The
treatment is prednisone. It is unlikely that the patient in this case has
polymyalgia rheumatica.
Crampy abdominal pain and explosive, watery diarrhea (choice E) are the
symptoms of traveler's diarrhea caused by E. coli. Acute monoarticular
arthritis is not commonly associated with traveler's diarrhea.
A 64-year-old woman is brought to the emergency department after her husband called
911. The couple was eating dinner together when the patient suddenly experienced left
facial droop, left arm and leg weakness, and slurred speech. This occurred at
approximately 6:30 p.m. The husband, who suspected that his wife had had a stroke,
gave her an adult sized aspirin (325mg) before calling 911. She has a past medical
history significant for diabetes mellitus and has never had surgery. She takes metformin
for her diabetes and a daily multivitamin, but takes no other medications. Her family
history is significant for diabetes: her mother and both of her sisters have it. She is a
schoolteacher approaching retirement. She denies any smoking history. She rarely
drinks alcohol and denies any drug use in the past. On arrival to the hospital, her
temperature is 37.0 C (98.6 F), blood pressure is 160/85mm Hg, and pulse is 82/min.
She has normal lung and heart sounds. She is awake, alert, and fully oriented. Her
language is natural and fluent. She has a severe left facial droop and severe dysarthria.
She has grade 0/5 strength in her left arm and in her left leg. She has intact sensation
for fine touch and pinprick. Her glucose level is 95mg/dL, a platelet count of
355,000/mm3, a partial thromboplastin time of 21.0 sec, and an INR of 0.95. A CT of the
head, done without contrast, is interpreted by a radiologist as being normal. You
diagnose the patient with ischemic stroke and note that it is now 8:30 p.m. Your
neurological consultant agrees that there are no contraindications for thrombolysis. The
most appropriate treatment at this time is to
A. administer heparin, intravenously after a weight-appropriate loading bolus
B. administer tissue plasminogen activator, intravenously
C. give her aspirin 325mg, in addition to the dose she had been given at 6:30 p.m.
by her husband
D. give her clopidogrel 75mg
E. give her no additional treatment as she has already received a dose of aspirin
Explanation:
The correct answer is B. The patient has had an acute ischemic stroke. The Federal Drug
Administration (FDA) has approved IV-tPA for acute ischemic stroke provided there are
no contraindications and that the drug is given within 3 hours of symptom onset. This
approval, in 1996, was based on the results of the National Institutes of Neurologic
Disorders and Stroke (NINDS) study which showed that patients who received IV-tPA
after fulfilling certain criteria have significantly better outcomes than those who did not.
Examples of contraindications include hemorrhage on CT scan; early ischemic changes
on CT scan; uncontrolled hypertension; minor or resolving presenting symptoms; recent
GI hemorrhage, and/or abnormal PT, PTT, platelet count, or glucose. In this case, the
patient has severe stroke symptoms, has been worked up within 2 hours of symptoms
onset, and has no contraindications for the use of IV-tPA.
Heparin (choice A) is incorrect. There is no known role for heparin in the treatment of
acute stroke. Its use remains controversial today, and is not approved by the FDA for
acute stroke.
Additional aspirin (choice C) is incorrect. There is no proven role for multiple doses of
aspirin in the setting of acute stroke. The use of aspirin is based on the results of the
International Stroke Trial of 1997, which showed that 300mg daily of this drug showed a
modest benefit in reducing new neurologic symptoms 6 months after the first stroke.
Clopidogrel (choice D) is incorrect. There is no known role for clopidogrel in the treatment
of acute stroke. Its use in prevention of stroke is based on the European Stroke
Prevention Study (ESPS-II) of 1996, which showed a synergistic effect of aspirin and
dipyridamole (in the same class as clopidogrel) when used together, in the prevention of
stroke, but was never investigated in the setting of acute stroke.
No treatment (choice E) is incorrect. As mentioned above, aspirin is moderately effective
in the prevention of new strokes, but it is not adequate in the setting of an acute stroke,
especially in the case of a patient who qualifies for IV-tPA.
An 8-year-old girl is brought to the office for a well-child visit. She has no complaints at
this time. The mother is very concerned about her daughter's health because she has
been reading about the threats of bioterrorism on the Internet. She is particularly worried
about smallpox and anthrax. At this time you should
A. administer a vaccine for anthrax
B. explain that while there is no routinely recommended prophylaxis for smallpox
or anthrax, there are treatment options available if she acquires either of these
diseases
C. prescribe ciprofloxacin as a prophylaxis for anthrax
D. reassure the mother that her daughter has received all of the recommended
childhood immunizations and should be immune to the smallpox virus
E. tell the mother that you understand her concerns but that there is no prophylaxis
for either smallpox or anthrax
Explanation:
The correct answer is B. Anthrax and smallpox are potential biological weapons that can
be used for bioterrorism. Smallpox has been eradicated with vaccines and so the vaccine
is no longer part of the routinely recommended childhood immunizations. Cidofovir is a
treatment option for an established infection. Anthrax is a potentially deadly bacteria that
can cause cutaneous, gastrointestinal, and respiratory diseases. A vaccine is available,
but it is only routinely given to persons at a significant continuing risk (military personnel).
Ciprofloxacin is used for prophylaxis and treatment for anthrax in adults and penicillin is
used for children.
It is inappropriate to administer a vaccine for anthrax (choice A) because it is only
routinely given to persons at a significant continuing risk. This patient does not seem to
be at a "significant continuing risk" at this time.
Since there is no reason to believe that this patient is going to be exposed to anthrax, you
should not prescribe ciprofloxacin as a prophylaxis for anthrax (choice C). Also,
ciprofloxacin is not approved by the FDA for use in persons under 18 years of age. It may
be indicated, however, for an established serious or life-threatening infection.
Since the smallpox vaccine is not routinely given as part of the recommended childhood
immunizations in the United States, it is incorrect to reassure the mother that her
daughter has received all of the recommended childhood immunizations and should be
immune to the smallpox virus (choice D).
It is incorrect to tell the mother that you understand her concerns but that there is no
prophylaxis for either smallpox or anthrax (choice E) because as stated above, there is
prophylaxis for anthrax and there is an effective vaccine for smallpox, but neither of these
are indicated for this patient. According to the Centers of Disease Control in March 2001,
pre-exposure vaccination against smallpox is not recommended for any group other than
laboratory or medical personnel working with non-highly attenuated orthopoxviruses.
You are notified that one of your patients, a 3 -year-old previously healthy girl, is
admitted to the hospital because of 6-day history of fever, irritability, and erythema of the
hands and feet. Her physical examination on admission showed a temperature of 39.7 C
(103.4 F), bilateral conjunctival injection, an enlarged left-sided cervical lymph node (2.0
cm), fissured lips, a red tongue with red papillae, pharyngeal hyperemia, erythematous
and edematous palms and soles, and a confluent, blanching erythematous rash on the
trunk. The mother told the emergency department physician that she had been giving
her daughter aspirin for the past week to reduce her fever. In the emergency
department, intravenous fluids were started, the aspirin therapy was continued, and
laboratory studies were ordered. These laboratory studies just returned and show an
erythrocyte sedimentation rate of 28mm/h and a platelet count of 490,000/mm3. The
patient is extremely uncomfortable and now shows desquamation of the fingers and
toes. The mother is very concerned about her daughter's condition. At this time the most
correct statement about her condition is:
A. Corticosteroids are necessary to decrease the risk of aneurysms within 10 days
of the onset of fever
B. Influenza vaccination is necessary if this patient requires long-term salicylate
therapy
C. MMR vaccination should be given within a month if this patient receives
intravenous gamma globulin
D. Nasopharyngeal cultures will help to establish the diagnosis
E. There is a 40% risk of death associated with her disease
Explanation:
The correct answer is B. This patient most likely has Kawasaki disease, which is treated
with aspirin and intravenous gamma globulin. The disease is characterized by a high
fever for longer than 5 days, bilateral conjunctival injection, fissured lips, a "strawberry
tongue", mucosal change in the oral pharynx, erythematous and edematous palms and
soles with desquamation, a polymorphous rash, cervical lymphadenopathy, an elevated
erythrocyte sedimentation rate, and thrombocytosis. The most important complication is
coronary artery aneurysms, which may be prevented by early treatment with aspirin and
intravenous gamma globulin. An echocardiogram is necessary to evaluate cardiac
involvement. The influenza vaccine is necessary if this patient requires long-term
salicylate therapy because of the possible increased risk of Reye syndrome. Reye
syndrome may develop in patients with influenza who receive salicylates.
Corticosteroids are necessary to decrease the risk of aneurysms within 10 days of the
onset of fever (choice A) is incorrect. Initially, intravenous gamma globulin and aspirin are
routinely used to decrease the risk of aneurysms in patients with Kawasaki disease.
Some believe that steroids should be used later if therapy with intravenous gamma
globulin and aspirin is unsuccessful. But steroids are not necessary to decrease the risk
of aneurysms.
MMR vaccination should be given within a month if this patient receives intravenous
gamma globulin (choice C) is incorrect. MMR vaccination should be given 11 months
after treatment with intravenous gamma globulin. It should not be given sooner because
intravenous gamma globulin interferes with the serologic response to the measles
vaccine. If there is a high risk that this child will be exposed to measles, which is unlikely
in the United States, immunization should be given, and if at 11 months serologic testing
indicates that the earlier immunization was unsuccessful, she should be re-immunized.
It is incorrect to say that nasopharyngeal cultures will help to establish the diagnosis
(choice D) because no single study can diagnose Kawasaki disease. The exact cause of
this disease is unknown.
There is a 40% risk of death associated with her disease (choice E) is incorrect. In the
United States, the mortality rate due to Kawasaki disease is 0.05%. Death is usually due
to a myocardial infarction or ruptured coronary aneurysm.
A 53-year-old man is admitted to the hospital because of rapid onset of
shortness of breath. He reports that a little less than 2 weeks ago he noticed
that he was short of breath and since that time it has progressed to the point
where at rest, he is barely able to breath, and he is unable to walk without
"nearly passing out." He denies chest pain, pressure, any altered mental
status, cough, or fever. His past medical history is remarkable only for
hypertension treated with atenolol. The patient denies any recent travel,
occupational exposures, or sick contacts. On arrival to the emergency
department, the patient is mildly cyanotic and breathing at 24-28/min. He is
conversant and appropriate, but visibly short of breath. There are no obvious
signs of accessory muscle engagement. His room air oxygen saturation is
82%. The most appropriate management of this patient at this time is to
A. administer heliox
B. administer high flow oxygen via non-rebreathing mask
C. administer 3 liters/min oxygen via nasal prongs
D. administer 3 liters/min oxygen via simple face mask
E. perform endotracheal intubation
Explanation:
The correct answer is B. This patient has severe hypoxia of unknown etiology.
The nature of his illness and rapid course suggests a disease such as
interstitial pulmonary fibrosis. Most rapid cases such as this are idiopathic
(Hammond-Rich syndrome). Regardless of the cause, immediate
management is the same, provide adequate oxygen to determine if the
hypoxia can be corrected. High flow oxygen delivered via NRB mask offers
about 82-86% inspired oxygen concentration. If the shunt fraction is less than
50%, inspired oxygen of this amount will be able to correct the hypoxia. The
patient can then continue on oxygen until he can no longer protect his airway,
his work of breathing becomes too great, or he begins to desaturate.
Heliox (choice A) is a mixture of helium and oxygen that is used in patients
with severe bronchoconstriction. The combination gas is more laminar with its'
flow and allows better delivery of oxygen to the distal airways. It has no role in
the correction of hypoxia since it is a low oxygen concentration mixture.
Oxygen via nasal prongs (choice C) is inadequate for this patient. 3 L/min
offered in this manner is essentially 26-28% inspired oxygen concentration.
With this marginal escalation over ambient tensions, the patient will improve
minimally, or more likely, not at all.
In order to use a simple face mask for oxygen delivery (choice D) the flows
need to be greater than 6 L/min in order to effectively evacuate the expired
carbon dioxide from the mask and prevent rebreathing.
There is no indication to place an endotracheal tube at this time (choice E).
Although the patient is exerting tremendous effort to breathe, he is not in
distress, has no accessory muscle use, and is not discoordinate. The first
attempt at management should be to determine if oxygen, delivered via
external devices, can augment his oxygenation. If this is successful, his
respiratory rate will decline and his work of breathing will decrease
substantially.
An 18-year-old man comes to the clinic complaining of heaviness in his left
testicle. He noticed this for the first time 3 weeks ago after "pulling his groin" in
a high school football game. The groin pull has improved but the discomfort in
the testicle has not. He also states that he has noticed the left testicle is larger
than the right testicle. His pain is non-radiating, dull in character, and not
associated with any dysuria or discharge. He admits to an episode of
unprotected intercourse with a new partner approximately 1 month ago. There
is no weight loss, fever, cough, or headaches. Physical examination is
significant for a left testicle that is non-tender, hard, increased in size as
compared to the right, irregular in contour, and without transillumination. There
is no inguinal adenopathy. The right testicle is normal in size and shape. No
discharge is expressed per urethra. Urinalysis and urine culture are negative.
Beta-human chorionic gonadotropin (bHCG) level and alpha-fetoprotein (AFP)
levels are normal. The next most appropriate management for this condition is
A. to administer chemotherapy
B. external beam radiation therapy to scrotum
C. external beam radiation therapy to retroperitoneum
D. left radical orchiectomy via a scrotal incision
E. left radical orchiectomy via an inguinal incision
F. open testicular biopsy
G. prescribe antibiotics
H. recommend scrotal support, antiinflammatory drugs, and reevaluation in
2 weeks
I. schedule incision and drainage of scrotal abscess
Explanation:
The correct answer is E. This patient has testicular cancer until proven
otherwise. The most common symptom of testicular cancer is painless
enlargement of the testis. Patients frequently complain of a sensation of
testicular heaviness. 10% of patients will present with acute testicular pain as
a result of intratesticular hemorrhage or infarction and 10% of patients will
present with symptoms related to metastatic disease (back pain from
retroperitoneal metastases involving nerve roots, cough or dyspnea from
pulmonary metastases, etc). And 10% of patients are asymptomatic at
presentation and the mass may be picked up incidentally following trauma or
by the patient's sexual partner. An incorrect diagnosis is made at the initial
examination in up to 25% of patients with a testicular tumor and may result in
delay in treatment or surgery via a scrotal approach. It is important not to
violate the scrotal skin in the presence of testicular cancer. The lymphatic
drainage of the testis is the retroperitoneum, while the scrotal skin drains via
the inguinal nodes. Violating the scrotal skin has the potential to extend the
spread of metastases outside the normal anatomic path. This patient's
symptoms and physical findings are classic for testicular carcinoma. While
bHCG and AFP may be elevated in a variety of testicular cancers, they do not
have to be. The normal levels in this patient should not discourage one from
making the proper diagnosis. Epididymitis or epididymoorchitis are the most
common misdiagnoses in patients with testis cancer. These patients typically
have an enlarged tender epididymis that is separable from testicle. In
advanced stages the inflammation may spread to the testis and enlarge the
testicle as well. There is usually an acute onset of symptoms associated with
fever, urethral discharge, and irritative voiding symptoms. Hydrocele is also a
common misdiagnosis. Transillumination of the scrotum may reveal a
translucent, fluid-filled hydrocele versus a solid tumor. Since approximately 510% of testicular tumors may be associated with hydroceles, if there is any
question, then a scrotal ultrasound is mandatory.
Further therapy of testicular cancer depends on the type of tumor and clinical
stage. Chemotherapy (choice A) and retroperitoneal external beam radiation
therapy (choice C) both play a role in the adjuvant treatment of testicular
cancer. However, neither is appropriate management prior to radical
orchiectomy.
Radiation therapy to the scrotum plays no role in the treatment of testicular
cancer (choice B).
Initial treatment for all testicular cancer or presumed cancer is inguinal
exploration and high ligation of the spermatic cord with removal of the testicle
and spermatic cord (radical orchiectomy). As previously stated, scrotal
approaches (choice D) and open testicular biopsy (choice F) should be
avoided so as not to disrupt the lymphatic drainage system.
Antibiotics (choice G), scrotal support, and antiinflammatories (choice H) are
all appropriate therapy for patients with epididymitis or epididymoorchitis. As
previously explained, this patient does not have these diagnoses and
providing this form of treatment will only delay appropriate care.
Drainage of an abscess (choice I) is appropriate for any patient who has this
diagnosis. Patients with scrotal abscess have swelling, tenderness, erythema,
fluctuance, and possibly fever. This patient does not have any of these
symptoms, making this diagnosis extremely unlikely.
You are seeing a 23-year-old woman in your office for a follow up visit. She
had presented for an initial visit a month ago complaining of swelling in her
fingers. Today, she continues to describe edema and erythema of her
metacarpophalangeal joints. She has also had some edema of her left elbow.
She explains that her morning stiffness is lasting more than 1 hour. She is
currently on no medications and has no allergies to medications. Her mother
and maternal aunt have a history of severe rheumatoid arthritis. Her
temperature is 37.2 C (99.0 F), blood pressure is 123/65 mm Hg, pulse is
76/min, and respirations are 18/min. She has edema and erythema of the
metacarpophalangeal joints of both hands. The remainder of her joint
examination is unremarkable. Her breath sounds are clear, and her cardiac
rhythm is regular. The results of routine laboratory studies sent during her
previous visit are consistent with a diagnosis of rheumatoid arthritis. The most
appropriate pharmacologic intervention at this time to alter the course of her
disease is
A. aspirin
B. 5-fluorouracil
C. methotrexate
D. penicillamine
E. sertraline
Explanation:
The correct answer is C. It is reasonable to start the patient on a trial regimen
of methotrexate since studies have shown that disease modifying
antirheumatic drugs (DMARD) such as methotrexate have improved outcomes
with rheumatoid arthritis, and may alter the course of her disease. An NSAID
or COX-2 specific inhibitor should also be used.
Aspirin (choice A) is a first-time therapy that can help with the signs and
symptoms of her disease but will have minimal effect on the progression of
disease.
5-fluorouracil (choice B), an antineoplastic agent, has no proven benefit with
rheumatoid arthritis.
Penicillamine (choice D), while being a disease modifying antirheumatic drug
(DMARD), is associated with too many toxic side effects to be chosen as a
first-line DMARD.
Sertraline (choice E), a selective seratonin reuptake inhibitor (SSRI) used for
the treatment of depression, has no role in the management of rheumatoid
arthritis.
You are notified that one of your patients, a 76-year-old woman with hypertension,
osteoporosis, and hypothyroidism, is being discharged from the hospital where she has
been treated for an acute subdural hematoma sustained after a fall 2 weeks ago. A quick
review of her hospital chart reveals that she had an uneventful stay with her blood
pressures ranging from 125/90 mm Hg to 140/90 mm Hg, and pulse from 60/min to
70/min. You review her current medications and note that she is taking
hydrochlorothiazide, alendronate sodium tablets, and thyroxine. You schedule a visit with
the patient and her son to establish a discharge plan. She tells you that she lives alone
and is planning on going back to her own apartment, not to "nursing home hell." Her son
says that he will make sure that she has some type of part-time home care. The most
appropriate intervention to prevent another fall is to
A. discontinue her medications
B. keep phones at floor level
C. recommend a daytime home health care worker
D. schedule a home safety evaluation
E. tell her to walk around in stockings or socks at home
Explanation:
The correct answer is D. Falls are one of the most important problems in the elderly
population. Approximately one third of individuals over the age of 65 fall annually. Falls
are the most important cause of injury in this age group. There are many factors that
usually lead to a fall, both intrinsic and extrinsic, and include medications, a medical
illness, foot problems, and a "hazardous" home environment. After an acute medical
illness is ruled out or treated, targeted interventions must occur. Targeted interventions
include a full medication review with the elimination of unnecessary drugs, physical
therapy, and a home safety evaluation. Warped floorboards, inadequate lighting, and
throw rugs may all contribute to an unsafe home environment.
Discontinuing her medications (choice A) is inappropriate as she has a specific need for
each of these agents. It is possible to switch some of her medications, like possibly try a
different antihypertensive agent. However, the thyroxine for hypothyroidism and the
alendronate for osteoporosis, should not be discontinued.
Keeping phones at floor level (choice B) is recommended so that the patient can have
easy access if a fall occurs. This way they can get help as soon as possible. However,
this does not prevent a fall.
A daytime home health care worker (choice C) may be useful when they are there, but
she will be at greatest risk for a fall at night. Also, the worker cannot keep an eye on her
every second. Therefore, a home safety assessment is a better answer.
Telling her to always walk around in stockings or socks at home (choice E) is
inappropriate because they are slippery and can increase the risk of a fall. Your goal is to
prevent another fall, not to cause one.
A 56-year-old man comes to the emergency department with severe right
flank pain for the past 5 days. He says that the pain started after he returned
from a long hiking trip in the Grand Canyon, and despite taking some
ibuprofen, it has not improved. Infact, he came to the hospital today because
the pain has increased and he now has new onset of nausea and chills. He
tells you that he had similar pain several years ago that was diagnosed as uric
acid stones. He was treated conservatively and eventually passed all the
stones spontaneously. His temperature is 38.1 C (100.6 F), blood pressure is
130/80 mm Hg, and pulse is 115/min. On examination, the patient is unable to
lie still because of the pain and has significant right costovertebral tenderness
radiating to his right testicle. Leukocyte count is 16,000/mm3 and his creatinine
is 2.1 mg/dL. The most appropriate study after starting the patient on
intravenous hydration and antibiotics is
A. a CT scan of the abdomen and pelvis
B. a DMSA renal scan
C. an intravenous pyelogram
D. a radiograph of the kidneys, ureters, and bladder
E. a renal ultrasound
Explanation:
The correct answer is A. The patient's symptoms are consistent with an
infected and obstructing ureteral stone, especially in light of his past stone
history and probable dehydration from his hiking trip. Colicky flank pain
radiating to the groin typically marks the presence of a midureteral obstruction.
An obstruction in the presence of an infection is a medical emergency
because of the very high possibility of septic shock, therefore a quick
diagnosis is needed. The best imaging study for nephrolithiasis is a noncontrast CT scan which can visualize all types of stones and then accurately
locate them. It can also tell us whether the patient has hydronephrosis, hence,
disclosing whether the patient has an obstruction. Once the diagnosis is
made, a urologist will most likely need to put in a ureteral stent to correct the
obstruction.
A DMSA renal scan (choice B) is a radionucleotide study that evaluates renal
function and does not give much information about a renal obstruction. It also
takes 4-24 hours to obtain optimal images.
An IVP (choice C) is incorrect because of the patient's renal insufficiency at
the time of presentation. The elevated creatinine is a contraindication to IV
contrast. In any other cases of noninfected nephrolithiasis with a normal
creatinine, an IVP would be appropriate.
A KUB (choice D) would not be able to demonstrate the presence of a renal
obstruction. It can be helpful in verifying the presence of kidney stones.
However, uric acid stones are radiolucent and would not be seen on a plain
film.
A renal ultrasound (choice E) is incorrect because an ultrasound cannot
visualize the presence of ureteral stones. It can tell us whether the patient has
hydronephrosis, but nothing more.
A 71-year-old retired police officer comes to the clinic for his annual physical
examination. He has no complaints, but you notice that he is not his usual self.
He appears distraught and distant, speaks in a low voice, and avoids eye
contact. He seems to have lost some weight, and admits to not caring about
his meals. The physical examination and routine laboratory tests are
unremarkable. On further questioning the patient shares that his wife had died
several months ago. He lives alone, has no children, and misses her a lot.
You ask him about his daily activities. It seems that he stays to himself and
rarely sees his fellow officers or friends any more. The most important next
step is evaluation for
A. Alzheimer's dementia
B. feelings of guilt
C. religious preference
D. suicidal ideation
E. weight loss
Explanation:
The correct answer is D. It is important to ask this patient openly about
suicidal ideation, plans, or an existing wish not to live anymore. Apart from
anhedonia and other signs of depression or complicated grief, he seems to
have all the risk factors for suicide present. These risk factors include being
male, older, retired, widowed, having potential access to weapons since he
was a police officer, and having no close social support.
Alzheimer's dementia (choice A) in an elderly patient who seems to have
some cognitive difficulties, should be considered. However, in this case,
pseudodementia secondary to depression may be present. The imminent risk
of a possible suicide attempt is, in this case, more important to assess. If that
is ruled out, other tests and steps can be taken to rule out other medical
conditions.
Guilt feelings (choice B) may be a part of the depressive syndrome or grief, if
a patient feels that he should have been the one that deserved to die. Guilt
feelings can sometimes also lead to suicidal ideation. However, it is important
to ask about suicidal ideation independently before continuing further
assessment.
Religious preference (choice C) can be explored in light of the possible
suicidal ideation. If a patient is practicing his religion, even though suicidal
ideation may be present, the fact that most religious beliefs stand against selfharm, may be helpful in preventing suicide. However, exploring this comes
only after the presence of suicidal ideation is established.
Weight loss (choice E) in light of other normal findings is not the most urgent
concern. It should be considered as a part of the depressive syndrome and
monitored.
A 34-year-old woman comes to the office for a follow-up examination after
passing a kidney stone in the hospital last week. You were away on vacation
and so your partner was involved in her in-patient treatment. The patient tells
you that your partner did not tell her anything about her condition and always
seemed "as if he was late for his golf tee-off time." Passing the stone was
"more painful than the vaginal delivery of all 3 children combined" and so she
wants to make sure that she never has one again. She has no chronic medical
conditions, never had surgery, and takes no medications. Her father and
brother both suffer from nephrolithiasis. You go over to the computer to check
if the laboratory report on the composition of her kidney stone is complete.
You see that the stone was composed of calcium and that she had a 24-hour
urine collection done in the hospital that showed 295 mg of calcium and 15 mg
of oxalate. Her serum calcium level is 8.5 mg/dL. The most appropriate course
of action is to
A. prescribe allopurinol, orally
B. prescribe cholestyramine, orally, and a low-fat diet
C. prescribe hydrochlorothiazide, orally
D. recommend a high fluid intake, but no medications or supplements
E. recommend megadoses of vitamin C
Explanation:
The correct answer is C. This patient most likely has idiopathic hypercalcuria,
which is characterized by calcium kidney stones, normocalcemia, and
unexplained hypercalcuria. It is believed to be hereditary and is more common
in men. Hydrochlorothiazide has been shown to decrease urinary calcium
levels and therefore decrease the rate of calcium stone formation.
Allopurinol (choice A) is a xanthine oxidase inhibitor that is used in patients
with kidney stones composed of uric acid. It decreases serum and urinary uric
acid levels.
Cholestyramine (choice B), an oxalate binding resin, combined with a low-fat
diet, is used in patients with intestinal hyperoxaluria for the prevention of
calcium stones. These patients have a 24-hour urinary oxalate level greater
than 50 mg. It occurs in patients with prior bowel surgery and chronic diseases
of the gastrointestinal tract that lead to malabsorption and bacterial
overgrowth.
It is inappropriate to recommend a high fluid intake, with no medications or
supplements (choice D) to a patient with idiopathic hypercalcuria and kidney
stones. Thiazides have been shown to reduce the rate of kidney stone
formation in a patient with her condition.
Megadoses of vitamin C (choice E) are controversial and have been
associated with the formation of kidney stones. Megadoses of vitamin C are
not generally recommended for the prevention of calcium stones in patients
with idiopathic hypercalcuria.
A 29-year-old man comes to the emergency department because of a severe
"excruciating headache" for the past 12 hours. He is an associate professor at
a local college and recently had a 3-day "end of semester party" for some of
his favorite students at his "weekend house." He vaguely recalls that a couple
of these students complained of a headache on the last day of their stay, but
the previous night was "quite a party." He is generally healthy, takes no
medications, and "drinks a few beers" each night. His temperature is 37.8 C
(100.0 F), blood pressure is 150/90 mm Hg, pulse is 70/min, and respirations
are 16/min. He has mild nuchal rigidity and flank tenderness. A careful cardiac
examination reveals a mid-systolic click. Ophthalmologic examination shows a
flat optic disc. During the examination, he turns to the side and vomits on your
shoes. You decide to perform a lumbar puncture and place him in the lateral
decubitus position with his thighs flexed. You introduce the spinal needle and
note that the opening pressure is 230 mm Hg. As you collect the four tubes of
spinal fluid you realize that the fluid is red. You run the tubes to the laboratory
and put them in the centrifuge and one hour later see that the supernatant of
the centrifuged cerebrospinal fluid is a yellowish color. The most likely
explanation is
A. acute bacterial meningitis
B. acute viral meningitis
C. a brain tumor
D. a subarachnoid hemorrhage
E. a traumatic lumbar puncture
Explanation:
The correct answer is D. This patient most likely has a subarachnoid
hemorrhage, presenting with a severe headache, and therefore has gross
blood in the cerebrospinal fluid and xanthochromia (the yellow color) of the
supernatant of the centrifuged fluid. The xanthochromia is the result of lysis of
red cells and release of their intracellular contents into the cerebrospinal fluid,
which usually occurs about 2-4 hours after a bleed. The elevated opening
pressure can be associated with meningitis, a brain tumor, or a subarachnoid
hemorrhage. It is possible that this patient has polycystic kidney disease
because he has hypertension, mitral valve prolapse, a subarachnoid
hemorrhage, and flank pain. An ultrasound or a CT scan is indicated, and may
reveal renal cysts. A CT scan of the head will most likely show a subarachnoid
hemorrhage.
Acute bacterial meningitis (choice A) and acute viral meningitis (choice B) are
not usually associated with blood in the cerebrospinal fluid. It is possible to
have xanthochromatic CSF when the CSF protein is elevated to 150 to 200
mg/dL, however, this patient has many other signs that indicate that he may
have polycystic kidney disease and a subarachnoid hemorrhage. The
headaches that the students complained of were most likely due to heavy
alcohol consumption the previous night, not meningitis.
It is unlikely that this patient has a brain tumor (choice C). While a brain tumor
may present with a headache and it is possible to have xanthochromatic CSF,
the acute onset of the symptoms is more consistent with a subarachnoid
hemorrhage. Also, the hypertension, flank pain, and mid-systolic click are
suggestive of polycystic kidney disease, which is associated with berry
aneurysms and subarachnoid hemorrhages.
A traumatic lumbar puncture (choice E) is when a meningeal vessel is
penetrated during a procedure, and usually shows pinkish-red CSF that clears
progressively from tubes one to four. A clear, not yellow, supernatant is
generally found after the CSF is centrifuged. This is most likely because the
red blood cells have not had a chance to lyse and release their intracellular
contents into the cerebrospinal fluid
A 42-year-old man comes to the emergency department because of chest
pain that is very focal, just adjacent to the sternum at approximately the
junction of the left 4th rib. It does not hurt to take a deep breath, but the pain is
exacerbated with certain twisting movements. He denies fever, cough, sputum
production, nausea, vomiting, diaphoresis, or shortness of breath. He tells you
that he thinks that "it is most likely nothing," but he is very concerned since his
father died of a myocardial infarction at age 45 and he has not seen a
physician since he was 14 years old. He has no significant past medical
history and is on no medications. He is afebrile with normal vital signs.
Oxygen saturation is 100% on room air. He has point tenderness to light
palpation over the left 4th costochondral junction. Heart is regular with no
murmurs, rubs, or gallops. Lungs are clear and his abdomen is benign.
Extremities are normal. Laboratory studies show a leukocyte count of 8,100
mm3, hematocrit of 34%, creatinine of 0.7 mg/dL, blood urea nitrogen of 18
mg/dL, and cardiac enzymes and troponin are normal. Electrocardiogram is
normal. Chest x-ray is normal. You explain that he most likely has
costochondritis and “prescribe as needed” nonsteroidal antiinflammatory
medications. He tells you that he would like to follow up with you in the clinic
because he is concerned about his cardiovascular risk. You should tell him
that when he comes to the clinic you plan to order
A. no laboratory tests since his pain will most likely have resolved
B. a periodic fasting cholesterol profile and yearly blood pressure checks
C. a periodic fasting cholesterol profile, yearly blood pressure checks, and
an annual chest x-ray
D. a periodic fasting cholesterol profile, yearly blood pressure checks, and
a coronary angiogram
E. a periodic fasting cholesterol profile, yearly blood pressure checks, and
sigmoidoscopy
Explanation:
The correct answer is B. Despite the fact that his initial clinical presentation
was chest pain, he clearly had costochondritis that has now resolved.
Although he may have significant coronary artery disease, (just as anyone
could), there is no clinical evidence of that at this time. He does however have
some cardiac risk factors, namely his sex and family history. It is the
physician's responsibility to try to modify the risk factors that can be treated
such as hypercholesterolemia and high blood pressure. It is thus important to
perform periodic fasting cholesterol and blood pressure checks. Infact, even if
he had no family history or was a woman, it is still part of good health
maintenance to monitor these. For these reasons, no laboratory tests are
needed on follow up visits, since his pain will most likely have resolved (choice
A) is incorrect.
Annual screening chest x-rays (choice C) for lung cancer screening or for any
other reason in asymptomatic patients is not currently recommended. Early
detection with low dose CT scanning in high-risk patients is currently under
heavy investigation and shows much promise.
Although as mentioned earlier, anyone could have coronary artery disease,
but nothing is indicating this in him right now. Thus a coronary arteriogram
(choice D) would not be indicated at this time.
Sigmoidoscopy (choice E) is also used in screening and health maintenance,
but not recommended until the age of 50.
A 42-year-old woman comes into the clinic complaining of intermittent easy
bruising around her eyes and chest, especially after surfing and boogie
boarding with her son. She recently suffered a syncopal episode and a work
up, which included a stress echocardiogram, revealed a hypertrophic heart
with a speckled pattern. When she was discharged from the hospital, she was
given a diagnosis of congestive heart failure with a restrictive pattern. An
endomyocardial biopsy is scheduled for next week. She denies any significant
family history and has been healthy with the exception of easy bruising,
occasionally with vomiting. Routine urinalysis from her recent admission
revealed proteinuria. Upon further questioning, she has suffered carpal tunnel
syndrome bilaterally and occasional numbness and tingling of her toes. The
most useful study to diagnose this patient's condition is
A. complete blood count
B. liver function test
C. morning cortisol level
D. protein immunoelectrophoresis
E. thyroid function tests
Explanation:
The correct answer is D. Protein immunoelectrophoresis is the correct answer,
because this patient has primary systemic amyloidosis which is an uncommon
disease characterized by the extracellular deposition of amyloid fibrils in
multiple organs. Cutaneous manifestations of purpura and ecchymoses in the
periorbital and facial areas are characteristic features in 16% of AL
amyloidosus patients. Involvement of the heart, kidney, liver, thyroid, adrenal,
or bone marrow can result in organ dysfunction and early death. Part of the
disease process is clonal proliferation of plasma cells that produce
homogeneous (monoclonal) immunoglobulin proteins (M protein). Diagnosis of
M protein is accomplished with several methods including serum protein
electrophoresis, immunoelectrophoresis, immunofixation, and capillary
electrophoresis.
Although complete blood count (choice A), liver function tests (choice B),
morning cortisol levels (choice C), and thyroid function tests (choice E) may all
useful in the evaluation of a patient with systemic amyloidosis. The most
useful test at this time is immunoelectrophoresis to detect the M protein.
A 1-year-old boy and 2-year old girl are brought to the office by their new adoptive
parents for a routine physical examination. They were adopted just 4 days earlier and
the only information that the new parents can provide is that the boy is HIV positive. The
children have not received any previous medical care but both seem happy and appear
well. Complete physical examinations are unremarkable. Laboratory studies show that
the boy is not severely immunocompromised. The most appropriate next step in the
management of these children is to administer
A. a bacille Calmette-Guerin vaccine to the boy only
B. a hepatitis B vaccine to the girl only
C. a measles mumps rubella vaccine to both children
D. an oral polio vaccine to both children
E. no vaccines at this time
Explanation:
The correct answer is C. Both of these children should receive a measles mumps rubella
vaccine (MMR) at this time. The MMR is recommended for all children with HIV at 12
months and again as soon as 4 weeks after the first dose, except when they are severely
immunocompromised. All other children in the household should also be given the MMR
vaccine.
A bacille Calmette-Guerin (BCG) vaccine to the boy only (choice A) is incorrect because
this vaccine, which is used to prevent tuberculosis, is not recommended in the United
States.
A hepatitis B vaccine to the girl only (choice B) is incorrect because both children should
be given this vaccine. It is recommended that children with HIV should be given the
hepatitis B vaccine as soon as possible to prevent infection.
An oral polio vaccine to both children (choice D) is incorrect because neither of these
children should be given the live virus vaccine. The inactivated polio virus vaccine (IPV)
should be given to both of these children.
No vaccines at this time (choice E) is incorrect because these children require vaccines.
The vaccines that should be given to both children are DTaP, IPV, H.flu type b, hepatitis
B vaccine, MMR, and the influenza vaccine. The HIV-positive child should also be given
the pneumococcal vaccine. The HIV-negative child should be given the varicella vaccine
and it should be considered for the HIV-positive child as well.
A 47-year-old man with a long history of alcohol abuse comes to the
emergency department after "vomiting blood." He denies any abdominal pain,
chest pain, shortness of breath, weakness, or numbness but does feel
lightheaded when he stands and noticed some dark stool yesterday. He drank
"some whiskey" last night but denies any smoking or injection drug use. He
has no prior history of gastrointestinal bleeding or known liver disease. He
occasionally takes aspirin for his hangovers. His temperature is 37.0 C (98.6
F), blood pressure is 128/72 mm Hg, and pulse is 98/min lying down.
Standing, his blood pressure is 98/61 mm Hg, pulse is 112/min, and
respirations are 16/min. Physical examination shows 1+ pitting edema in the
lower extremities and black, guaiac-positive stool. A chest and abdominal xray are normal. An electrocardiogram shows sinus tachycardia at 98/min.
Laboratory studies show:
The most appropriate next step in the management of this patient is to
A. administer intravenous cimetidine
B. begin intravenous fluid resuscitation
C. order a CT scan of the abdomen
D. perform nasogastric lavage
E. perform an upper endoscopy
Explanation:
The correct answer is B. In evaluating and managing a patient with a GI bleed
the most important first step is to stabilize their vital signs. By history and
exam this patient has evidence for a significant upper GI bleed and he is
orthostatic. It is therefore imperative to obtain intravenous access and to start
fluid resuscitation preferably with normal saline immediately.
In the management of patients with acute upper GI bleeding intravenous H2
blockers (choice A) are usually prescribed even though the evidence for this
common practice is weak. Nevertheless, in our patient the most pressing
issue is to correct his intravascular volume depletion as soon as possible.
In evaluating patients with upper GI bleeds it is usually not necessary to obtain
an abdominal CT scan (choice C) . In this patient with possible cirrhosis
secondary to his alcohol abuse it might be useful at some point to image his
liver with a CT scan or ultrasound when he stabilizes, but it is certainly not the
first step.
Likewise, nasogastric lavage (choice D) is commonly performed in the
emergency department on patients presenting with upper GI bleeding, but as
mentioned previously the first priority should be to hydrate the patient.
Upper endoscopy (choice E) should be performed on all patients who present
with a significant upper GI bleed. Endoscopy is the gold standard for the
diagnosis of the particular lesion that caused the bleeding and can also
provide important prognostic information on the chance for rebleeding.
Therapeutic endoscopy can also help stop the bleeding with various
techniques. The timing of the endoscopy is also somewhat controversial but
all authorities agree that the first step should be to fluid resuscitate the patient.
A 64-year-old woman with a diagnosis of chronic paranoid schizophrenia has
moved recently to town to live with her sister. She comes to the clinic for the
first time, since she needs to have a new doctor and needs her medication.
She has been healthy most of her life, and except for left hip replacement
surgery, has had no other interventions or treatments. She has been
maintained well for years on thioridazine. She has not been in the hospital for
the past 15 years. She hears voices occasionally, but has developed
strategies to distract herself. She shares that she has not really had any other
problems. She goes to church, walks her sister's dog and watches TV. She
denies any current complaints. Her physical examination is unremarkable.
Given this patient's history of treatment and current age, the effect of
thioridazine that she should be evaluated for is
A. cataracts
B. diarrhea
C. hypersalivation
D. hypertension
E. prolongation of the Q-T interval
Explanation:
The correct answer is E. Thioridazine belongs to the phenothiazine group of
antipsychotics that can cause prolongation of Q-T interval, inversion of the T
wave, and sometimes a bifid T or U wave. Changes are reversible. It has been
noted that several sudden deaths occurred secondary to cardiac arrests in
patients treated with thioridazine. The use of EKG periodically is thus advised.
Cataracts (choice A) have been described in beagle dogs treated with
quetiapine. Thioridazine can cause pigmentary retinopathy that is dose
dependent. Because of its anticholinergic effects, it may precipitate a
glaucoma attack.
Diarrhea (choice B) is not a usual side effect of thioridazine. On the contrary, it
has anticholinergic properties, thus causing dry mouth, constipation, and
paralytic ileus.
Hypersalivation (choice C) is a side effect of clozapine. Thioridazine, because
of its anticholinergic effects, causes dryness of the mouth and other mucous
membranes.
Hypertension (choice D) is not a side effect of thioridazine. All phenothiazines
tend to cause hypotension, secondary to the action on alpha-adrenergic
receptors.
An 18-year-old man with type I diabetes mellitus is brought to the emergency
department by a friend after being found comatose. There is a known history
of noncompliance with medications, however, there is no known history of
drug use. Vital signs are: temperature 37 C (98.6 F), blood pressure 80/65
mm Hg, pulse 110/min, and respirations 17/min. Oxygen saturation obtained
while the patient is receiving supplemental oxygen of 2 L/min via nasal
cannula is 98%. The patient is comatose and is taking rapid, shallow breaths.
Deep tendon reflexes are hypoactive. An intravenous line has been placed in
the field. A fingerstick glucose is 430 mg/dL. An arterial blood gas, basic
chemistry panel, and toxicology screen has been sent to the laboratory. The
next step in the management of this patient is
A. a chest x-ray
B. an endotracheal intubation
C. an intravenous fluid replacement with insulin
D. methadone
E. a pulmonary artery catheter insertion
Explanation:
The correct answer is C. This patient is suffering from diabetic ketoacidosis
(DKA) caused by a severe deficiency of insulin. Clinical symptoms include
coma, rapid and shallow breathing, high serum glucose levels, and metabolic
acidosis. The immediate management of this patient includes intravenous fluid
replacement and insulin infusion. When laboratory results return, electrolyte
imbalances must also be corrected.
A chest x-ray (choice A) would be complementary to a complete the
evaluation of any comatose patient. In this patient with a picture of diabetic
ketoacidosis, a chest x-ray would be a secondary concern. The first priority is
intravenous fluid replacement and insulin therapy.
Endotracheal intubation (choice B) is not necessary at this point as the patient
has a normal oxygen saturation. Adequacy of respiration will need to be
reassessed when the arterial blood gas results are available. The first priority
is intravenous fluid replacement and insulin therapy.
Methadone (choice D) is used to treat heroin dependency.
A pulmonary artery catheter (choice E) is not yet necessary as the patient is at
the present time hemodynamically stable. The first priority is intravenous fluid
replacement and insulin therapy.
A 54-year-old man presents to his primary care physician's office over a
concern regarding prostate cancer. The patient has no history of the disease,
but his father died of prostate cancer at the age of 61 and the patient was told
that he has an increased risk for developing the cancer. The patient reports
that he has had digital rectal examinations each year, but that he would like to
be "screened" for prostate cancer. He has no other medical history and takes
only a low-dose aspirin daily. He denies smoking and illicit substance abuse
and admits to drinking alcohol socially.The most appropriate response to this
patient is:
A. "Normal digital rectal examinations exclude any likelihood of prostate
cancer."
B. "PSA blood tests are available."
C. "PSA blood tests are available but only for patients with known cancer."
D. "PSA urine tests are available for all men over the age of 50."
E. "There is no effective screening test for prostate cancer."
Explanation:
The correct answer is B. Men have between a 15-20% lifetime risk of
developing prostate cancer. The PSA test was introduced in the late 1980s
and quantifies a glycoprotein produced by the prostate that spills over into the
blood. Although current guidelines differ by society, clinical practice and
standard of care is to test high-risk men over the age of 40 for PSA levels. Any
level greater than 4.0 ng/mL requires a prostate biopsy. The positive and
negative predictive values of the test vary tremendously with patient
population, but roughly 30% of patients with elevated PSA levels will have
prostate cancer. These tests are also used to follow therapy in patients with
known prostate cancer (choice C).
Although digital rectal examinations (choice A) are capable of detecting
enlarged prostates and nodules, they do not effectively exclude small tumors,
which fail to distort the gland or that present in areas of the gland which are
not peri-urethral.
PSA tests are for detection of PSA in blood, not urine (choice D). There is no
PSA present in ejaculate or in urine although ejaculation does transiently
increase serum PSA levels for up to 48 hours.
Although the PSA test has variable sensitivity and specificity, it is a fairly
robust test when used to screen patients at high risk or with enlarged prostate
glands on physical examination. Therefore, it is an effective screening (choice
E) test, but the efficacy depends very much on the population being tested
and specific characteristics of the patient such as the presence or absence of
benign hypertrophy.
A 4-year-old boy is admitted to the hospital with right eyelid swelling and
redness. Approximately 3 days prior to admission he developed nasal
discharge, fever, and then 24 hours prior, developed the right eyelid swelling
and erythema. He has a history of mild asthma and his vaccinations are up to
date. His temperature is 38.9 C (102 F), blood pressure 114/68 mm Hg, and
pulse is 78/min. There is mild proptosis on the right. Extraocular motor
examination is remarkable for difficulty moving the right medially, pupils are
equally round and reactive to light. No lymphadenopathy is present.
Laboratory studies show: leukocyte count 18,000mm3, hematocrit 35%, blood
urea nitrogen 10 mg/dL, creatinine 0.6 mg/dL, sodium 135 mg/dL, and
potassium 4.1 mg/dL. Intravenous cefuroxime therapy is initiated. The most
appropriate next step in management is to
A. administer amphotericin B, intravenously
B. discharge him after 24 hours of antibiotics
C. order a CT scan of the orbits with intravenous contrast
D. prepare him for surgical exploration of the right orbit
E. prepare him for surgical exploration of the sinuses
Explanation:
The correct answer is C. This patient has a cellulitis of the right eyelid. One of
the most important things to find out in order to guide therapy is whether the
infection is preseptal or postseptal. The orbital septum can be thought of as
running along the anterior aspect of the bones of the orbit separating the
superficial tissues from the orbital contents themselves. Some signs of
postseptal infection that this patient has are ophthalmoplegia and proptosis.
The risk of visual loss and spread to deeper structures such as the cavernous
sinus are why this diagnosis is important to make, as opposed to a superficial
orbital cellulitis, which is fairly easily treated. It is also important to look for any
abscesses that might need to be drained. CT scanning can help to elucidate
many of these.
Amphotericin B (choice A) is an important tool in the treatment of fungal
infections such as mucormycosis. Infection with this fungus is extremely
deadly and should be considered in the differential diagnosis of any patient
with a severe sinus infection. There is often evidence of devitalized tissue and
patients often have some sort of relative immune suppressive disease such as
diabetes or chronic renal insufficiency. There is nothing in this case to suggest
that amphotericin should be added at this point.
Discharge of the patient should be only after adequate treatment, which may
take more than 24 hours (choice B). Intravenous antibiotics can be
discontinued and switched to oral after the patient has been stable and
afebrile for 24 hours.
Surgical exploration of the sinuses or orbit (choice D and E) should not be
performed prior to a CT scan. If an abscess or other signs of more severe
infection are seen on the scan, then surgical debridement may be warranted.
A 43-year-old female former nurse comes to the clinic complaining of a rash
around a healing laceration on her left dorsal forearm. She reports that she cut
her forearm with a knife while removing it from her dishwasher 5 days ago.
The cut was not deep and she did not seek medical attention. She has been
cleaning it with hydrogen peroxide and applying neomycin ointment, followed
by wrapping it in a bandage twice a day. Yesterday evening during the
dressing change she noted some pruritus and erythema in the area of the
wound; this morning she was alarmed to find the area extremely itchy and with
a "horrible rash." She denies fevers or chills and states that besides the
situation with her wrist, she feels well. Looking at her chart and speaking with
her you find her only medication is lisinopril for hypertension. She has no other
medical problems of which she is aware. Physical examination of the left
forearm reveals a shallow, healing, 2.5 cm long laceration. Extending
approximately 2 cm from the wound in each direction is erythema and minute
vesicles filled with clear fluid. There is no lymphadenopathy and her
temperature is normal. The most appropriate management is to
A. prescribe acyclovir ointment
B. prescribe acyclovir tablets
C. prescribe cephalexin tablets
D. prescribe hydrocortisone ointment
E. recommend bacitracin ointment
Explanation:
The correct answer is D. The appearance of the skin, the history of applying
neomycin ointment, and the time course all point to an allergic contact
dermatitis. Neomycin is a very common sensitizer, with as many as 5% of the
population being allergic to the compound in some studies. Treatment for mild
cases of allergic contact dermatitis is stopping the offending agent and
applying topical steroids. This case also highlights the fact that many people
wrongly assume that topical products, especially over-the-counter products,
are not medicines.
Acyclovir ointment (choice A) has been shown to be marginally effective for
speeding healing of herpes labialis, but has not been studied for extralabial
sites. For a large area of herpes vesicles, oral acyclovir would be the preferred
treatment.
Acyclovir tablets (choice B) would be reasonable if this was a herpetic
outbreak. The classic appearance of herpes is grouped vesicles on an
erythematous patch or edematous plaque. The localization of herpetic lesions
to the forearm would be unusual, as would the subjective symptom of pruritus
since herpetic lesions are usually painful. Lack of lymphadenopathy would be
another factor that would militate against a diagnosis of herpes. Definitive
diagnosis of herpes would be either positive culture or identifying
multinucleated cells on a Tzanck prep.
Cephalexin (choice C) is a treatment for impetigo, particularly if it is
widespread. Also, if you suspected the patient had cellulitis, systemic
antibiotics would be indicated. Clinically cellulitis would have erythema around
the wound, but fever and/or lymphadenopathy would be expected. In addition,
cellulitis does not present with vesicles nor with pruritus.
Bacitracin ointment (choice E) may be a reasonable choice if you suspected
impetigo, but that entity consists of erythematous plaques with a prominent
scale-crust. This crust is classically described as honey-colored. Impetigo
generally does not exhibit vesicles. It would also be uncommon to develop in
an area being cleaned and having antibacterial ointment applied twice a day.
Note, the lack of fever and the lack of lymphadenopathy are entirely consistent
with impetigo. Another reason bacitracin would be a poor choice in this
situation is a high percentage of patients with contact sensitivity to neomycin
will cross-react to bacitracin, thereby aggravating the contact dermatitis.
A 51-year-old woman who your partner has been treating for low back pain calls the
office complaining of a change in symptoms. She reports that although she has had
intermittent low back pain for a number of years, in the past few weeks there has been a
profound increase in her pain. She also reports that her left leg often tingles and is numb.
In reviewing her records, it is noted that the patient has been seen in the office over the
years for back pain and has recently been informed that she can no longer obtain
narcotics. The patient works as a daycare manager and often lifts small children. She is
married with a 4-year-old daughter. The most appropriate next step is to
A. ask the patient to come to the office for a neurological evaluation
B. order an MRI of her lumbar spine
C. prescribe additional narcotic analgesia
D. refuse to see the patient as she is malingering
E. schedule the patient for epidural steroid injections
Explanation:
The correct answer is A. This patient suffers from low back pain. This entity is one the
most challenging disorders in all of medicine. It has been estimated that 70% of the
population suffers from some sort of low back pain at an annual cost of between $20 and
$50 billion. The etiology is often unknown in 85-90% of cases but most cases resolve with
support and mild pain relief. The goals of evaluation are primarily to exclude serious
underlying visceral or systemic disorders. This patient, despite being seen for back pain
in the past, has new neurological symptoms which suggest an underlying etiology that is
not simply musculoskeletal. She requires a full physical examination.
In the absence of a full exam, which would point to a specific diagnosis, it is not
appropriate to order an MRI of her lumbar spine (choice B) without seeing the patient.
In the presence of new signs or symptoms such as the ones this patient is describing,
masking the pain with narcotics (choice C) is incorrect. In addition, narcotics have been
shown to be only minimally effective in managing low back pain and are not considered
standard management for this condition.
Even though this patient has had a complex past history with the clinic, her new signs
and symptoms point towards a specific diagnosis such as spinal stenosis or disk
herniation and she requires evaluation. To consider her a malingerer (choice D) without
evaluation is inappropriate.
Epidural steroid injections (choice E) is a specific therapy aimed at reducing inflammation
in certain conditions such as epidural adhesions, facet degeneration, and refractory low
back pain. Their efficacy is uncertain and the treatment is usually attempted only after
more serous disease is excluded and the patient has failed physical therapy and oral
antiinflammatory agents.
A 58-year-old alcoholic with hepatitis C cirrhosis is admitted to the hospital for
management of his ascites. He has been managed as an outpatient with
diuretics and oral lactulose, but over the past few weeks, he reports increasing
abdominal girth, weight gain and lower extremity edema. He has been
noncompliant with his low-sodium diet. His medications include furosemide,
spironolactone, lactulose, ciprofloxacin, and thiamine. On physical
examination, he appears grossly edematous and appropriately responsive.
His lungs are clear and his heart is without extra sounds or murmurs. His
abdomen is tense with a fluid wave and shifting dullness on percussion. He
has numerous non-blanching telangiectasias on his torso and abdomen. His
testes are small for his age and there is no asterixis. Admission laboratory
studies show:
Sodium
121 mEq/L
Potassium
4.3 mEq/L
Bicarbonate
29 mEq/L
BUN
38 mg/dL
Creatinine
1.5 mg/dL
Urinalysis shows some granular casts and a urinary sodium concentration of
<10 mmol/L. The most appropriate therapy is to
A. administer hypertonic saline
B. administer sodium chloride tablets
C. increase the dose of furosemide
D. increase the dose of spironolactone
E. salt restrict
Explanation:
The correct answer is E. True hyponatremia is always hypotonic. There are
then three types of hyponatremia: hypovolemic, euvolemic, or hypervolemic.
The therapy for the hyponatremia depends on both the urinary concentration
of sodium as well as the volume status for each patient. Once the patient's
total body volume status has been estimated, urinary sodium usually allows
focusing of the differential diagnosis. For this patient, he clearly has total body
volume overload. He has cirrhosis and portal hypertension. The standard
therapy for these patients is salt restriction and management of volume status
with diuretics. This patient, at the core of his therapy requires salt restriction.
Although some forms of hyponatremia respond to administering hypertonic
saline (choice A), knowing the pathophysiology of this patient's hyponatremia,
it can be clearly seen that this intervention will only aggravate this patient's
condition.
Giving the patient sodium chloride tablets (choice B) will clearly not be
beneficial to this patient given the above explanations.
Increasing the dose of furosemide (choice C) or the dose of spironolactone
(choice D) will certainly serve to diurese the patient, but in the presence of a
salt load, the kidneys will respond by avid retention of salt and water and thus
aggravate the patient's condition.
A 24-year-old third-year medical student is brought to the emergency
department from the operating room after she began to have trouble breathing
after she put on her gown, latex gloves, and mask. This was going to be her
first time "scrubbing in" to a case on the first day of her surgery rotation.
Another medical student who was "scrubbed in" to the case, tells you that the
she was grasping at her throat and attempting to rip off her gown. She is now
on a stretcher, gasping for air. She is brought into a room and all of her
clothes are taken off, revealing red hands and an eruption of wellcircumscribed, erythematous, raised lesions covering her entire body. Her
blood pressure is 70/50 mm Hg, pulse is 110/min, and respirations are 45/min.
She has marked laryngeal edema and audible wheezes. The other student
says that this is the second day of their third year, and that the only other
patient contact that they have had was during the first year when they went as
a group to evaluate a patient in respiratory isolation. At this time the most
correct statement about her condition is:
A. Her rash is unrelated to her respiratory symptoms
B. Her symptoms are caused by the anxiety and stress from the first
surgical experience
C. Her symptoms are due to an infectious pathogen
D. Her symptoms are the result of bioactive mediators released when
exposed to an antigen
E. She should be given indomethacin immediately to reduce the respiratory
symptoms
Explanation:
The correct answer is D. This patient is experiencing an anaphylactic reaction,
which is a life-threatening response in a sensitized individual to a specific
antigen, most likely the latex in the gloves, and is associated with the release
of lipid mediators, secretory granule preformed mediators, and cytokines. The
manifestations typically occur within seconds to minutes of exposure and
include respiratory distress, vascular collapse, urticaria and angioedema,
gastrointestinal symptoms, and shock. The reaction involves IgE-dependent
activation of mast cells, basophils and the release of mediators such as
histamine, cytokines, and lipid mediators. The immediate treatment involves
the ABCs (intubation, oxygen, and intravenous saline), epinephrine, a
vasopressor agent such as dopamine, an antihistamine such as
diphenhydramine, and glucocorticoids to alleviate later recurrence of
symptoms.
It is incorrect to say that the rash is unrelated to her respiratory symptoms
(choice A) because they are both associated anaphylactic reactions. Anxiety
and stress from the first surgical experience (choice B) may be associated
with vasovagal (vasodepressor) syncope, which is the cause of the common
"faint". It frequently occurs during periods of stress or fear and often occurs in
medical students during new experiences. The symptoms include
hypotension, bradycardia, nausea, pallor, diaphoresis, and a sudden, transient
loss of consciousness. Lying down and elevating the legs reverses the
symptoms. This is very different from a life-threatening anaphylactic reaction
where hypotension, tachycardia, respiratory failure, shock, and skin
manifestations occur. Assuming a supine position during an anaphylactic
reaction does not reverse the symptoms.
This patient is most likely experiencing an anaphylactic reaction from the latex
gloves, not an infectious pathogen (choice C). Pathogens may cause septic
shock by releasing toxins that then activate cytokines, platelet activating
factors, arachidonic acid metabolites, and humoral defense systems. This
manifests as hypotension, tachycardia, changes in mental status, and cold
extremities. The laryngeal edema and specific rash that this patient has are
more consistent with an anaphylactic reaction than septic shock.
She should definitely not be given indomethacin immediately to reduce the
respiratory symptoms (choice E) because it is a nonsteroidal anti-inflammatory
drug that may cause similar life-threatening respiratory reactions in patients
with asthma. You do not know her medical history. Also, this is not part of the
management of anaphylaxis, which includes epinephrine, saline,
vasopressors, and intubation.
A 45-year-old homeless man is brought to an emergency department by
ambulance after being found in the street obtunded, combative, and agitated.
The patient is unable to provide any history in the emergency room due to his
altered mental status. His temperature is 38.1 C (100.6 F), blood pressure is
200/110 mm Hg, and pulse is 110/min. While in the emergency department,
the patient has a generalized tonic-clonic seizure lasting approximately 30
seconds. The patient's presentation is most consistent with withdrawal from
A. alprazolam
B. caffeine
C. cocaine
D. heroin
E. tricyclic antidepressants
Explanation:
The correct answer is A. Withdrawal from a benzodiazepine, such as
alprazolam, is a potentially fatal condition that can involve psychomotor
agitation, frank delirium, autonomic instability, and seizures. In this patient,
another consideration with a similar clinical presentation would be delirium
tremens from alcohol withdrawal. Both conditions would be treated with
supportive measures and acute parenteral benzodiazepine treatment.
Caffeine (choice B) has a potential withdrawal syndrome associated with
irritability, possible concentration difficulties, and headache. However, even
abrupt discontinuation of caffeine use is not considered to be at all dangerous.
Cocaine withdrawal (choice C) is characterized by intense feelings of
dysphoria, loss of energy, increase in appetite, and intense cravings for the
drug. It is not usually associated with delirium, autonomic instability, or
seizures and does not carry an associated risk of fatality.
Heroin withdrawal (choice D) is characterized by piloerection, nausea, and
diarrhea and can include modest increases in blood pressure and heart rate.
However, it is not characterized by delirium, withdrawal seizures, or autonomic
instability and is not considered to carry a high risk of fatality. Clonidine or a
long-acting opiate agonists such as methadone are the treatments of choice
for withdrawal from heroin.
Tricyclic antidepressants (choice E) do not have an associated withdrawal
syndrome and may be discontinued abruptly, especially if treatment-emergent
complications arise.
A 36-year-old woman comes to the emergency department with a 36-hour history
of fever, chills, and right flank pain. She has had similar, but less severe episodes
in the past which all improved with antibiotics and hydration. Her temperature is
39.1 C (102.4 F), blood pressure is 110/68 mm Hg, pulse is 115/min, and
respirations are 24/min. Physical examination shows severe right sided
costovertebral tenderness and mild right lower quadrant tenderness without
rebound. She vomits twice during your exam. Urinalysis shows 18 red blood cells
per high power field. A CT scan without contrast shows moderate right-sided
hydronephrosis with perinephric stranding and a 7 mm stone in the mid right
ureter. The appendix is not visualized. Laboratory studies show:
The most appropriate intervention at this time is
A. extra-corporeal shock wave lithotripsy of ureteral stone
B. intravenous hydration
C. intravenous hydration and antibiotics
D. intravenous hydration, antibiotics, and placement of right percutaneous
nephrostomy tube
E. observation alone
F. repeat CT scan with oral contrast to evaluate for appendicitis
Explanation:
The correct answer is D. This patient is suffering from obstructive uropathy leading
to pyonephrosis (distention of the renal pelvis with pus) and sepsis. Treatment of
the infected, obstructed collecting system is priority to preventing further renal
failure and sepsis. Relief of obstruction (decompression) can be performed with
either a percutaneous nephrostomy tube or insertion of ureteral stent (the
preferred modality is controversial with available data supporting either modality).
Manipulation of an infected renal collecting system at the time of infection should
be kept to a minimum, and treatment should be limited to procedures that allow for
adequate drainage of the obstructed kidney.
ESWL (choice A) is contraindicated during infection. This therapy, which provides
shock waves to renal/ureteral calcifications, would not provide relief of urinary
obstruction.
Antibiotics and hydration (choices B and C) are important therapeutic modalities
for this patient. However, they are not the definitive therapy and must be
accompanied by decompression of the dilated kidney. Broad spectrum antibiotics
should be implemented immediately and tailored according to cultures from the
pus that is drained at time of nephrostomy tube placement. Without relief of
obstruction, any antibiotic treatment will be incomplete.
Because this patient has had similar less severe episodes in the past, one might
be tempted to observe her (choice E). However, while working in the emergency
department it is important to realize that this patient exhibits none of the
prerequisites that would make her eligible for observation (afebrile, no white count,
no azotemia, able to tolerate oral intake and oral analgesics).
This patient is clearly very sick. While acute appendicitis (choice F) must always
be included in the differential of a patient with right lower quadrant tenderness,
fever, and elevated white blood cell count, this patient's complete clinical picture is
not consistent with appendicitis. Delaying definitive treatment for further radiologic
studies would only prolong clinical improvement and would further endanger this
patient's health.
A 32-year-old HIV-positive intravenous drug abuser is admitted to the hospital
after being found on the city sidewalk having a generalized seizure. A CT scan
of the head is performed and shows several rim-enhancing lesions with
minimal mass effect. You notice in his chart that he takes multiple "HIV
medications" but he is unsure of the exact names and his grandmother died of
a ruptured cerebral aneurysm. The most appropriate next step in management
is to
A. get a cerebral angiogram
B. order a ventricular cerebrospinal fluid (CSF) aspiration
C. perform a lumbar puncture and include cerebrospinal fluid for Epstein
Barr virus (EBV) PCR in tests ordered
D. stop all antiretroviral therapy
E. treat with intravenous acyclovir
Explanation:
The correct answer is C. The most common etiologies of rim-enhancing brain
lesions in AIDS patients are primary CNS lymphoma (PCNSL) and
Toxoplasma gondii infection. Other etiologies such as bacterial or fungal
abscess are also possible. CSF EBV PCR test is highly sensitive and specific
for PCNSL.
A cerebral angiogram (choice A) should be done if you suspect an aneurysm
or vascular malformation. These are unlikely in this case.
Since there is no mass effect it is safe to do a lumbar puncture. Therefore, a
ventricular cerebrospinal fluid (CSF) aspiration (choice B) is not necessary.
There is no reason to stop all antiretroviral therapy (choice D).
Intravenous acyclovir (choice E) is used to treat herpes encephalitis, which is
unlikely in this case.
A 22-year-old medical student has a PPD placed at his yearly physical exam. He has no
medical problems and is currently taking no medications. He returns to your office three
days after his initial appointment with 8 mm of induration around his PPD. He is
concerned that he is infected with tuberculosis. At this time you should
A. begin sputum collection for AFB staining and culture
B. explain that he needs an extended course of single drug therapy
C. explain that he needs an extended course of four drug therapy
D. explain that no intervention is indicated at this time
E. recommend a chest x-ray to determine appropriate treatment
Explanation:
The correct answer is D. When evaluating a patient for a positive tuberculin skin you must
first classify which risk category the patient belongs. Patients are divided into three
groups: High risk patients are considered to have a positive skin test if the area of
induration is greater than or equal to 5 mm. These are people with HIV or those at high
risk for HIV (i.e. IV drug users), people who have close contact with patients with active
TB, or those who have had TB or evidence of prior TB on chest x-ray. Patients with
induration greater than or equal to 10 mm are considered to have a positive test if they
immigrated from countries with a high prevalence of TB, are known HIV negative IV drugs
users, are medically underserved populations, prisoners, mental institution residents,
nursing home residents, those with some chronic medical problems, or hospital workers.
All other people are considered to have a positive test if the area of induration is greater
than 15 mm. Our patient has an area of induration of less than 10 mm. No further work up
is needed in this patient.
Collecting sputum for acid fast bacilli (AFB) (choice A) is helpful in confirming the
diagnosis of suspected tuberculosis. If this patient reported typical symptoms of
tuberculosis such as fevers, night sweats, weight loss, and cough a positive AFB smear
or culture would help us to make the diagnosis of TB.
Extended course of single drug therapy (choice B) would be necessary if this patient had
a positive skin test with no evidence of active infection. Preventative therapy has been
shown to reduce the incidence of reactivated tuberculosis.
An extended course of four-drug therapy (choice C) is the treatment of choice for active
tuberculosis. A typical four-drug regimen includes isoniazid, rifampin, pyrazinamide, and
ethambutol or streptomycin.
A recent converter of their skin test should get a chest x-ray (choice E) to evaluate for any
evidence of active disease and to have a baseline chest x-ray that could be followed in
the future. Once a patient has a positive skin test, it is no longer valuable to perform
further skin tests. Rather, yearly chest x-rays for evidence of reactivated TB should be
done.
A 59-year-old man is admitted to the hospital for shortness of breath. The
patient has a long-standing cardiac history and has suffered two non-Q wave
infarctions in the past 20 months. The patient reports bright red blood in the
toilet bowl during his last bowel movement. Laboratory data are remarkable
for a hematocrit of 22%. Given the patient's known coronary disease, his
attending cardiologist recommends a blood transfusion. As appropriate, you
order 2 units of appropriately matched red bloods cells in order to transfuse
the patient to a target hematocrit above 30%. While the first unit is being
administered, the patient becomes febrile and develops chest and flank pain.
You are immediately summoned to his side and on arrival you note erythema
around the intravenous access site and a small volume of dark colored urine
in his Foley catheter bag. The remainder of the physical examination is
unremarkable. The most likely diagnosis is
A. acute febrile hemolytic reaction
B. anaphylaxis
C. delayed hemolytic transfusion reaction
D. pulmonary embolus
E. transfusion associated lung injury
Explanation:
The correct answer is A. This patient is most likely having an acute febrile
hemolytic reaction. This occurs when there is a mismatch between the
patient's and donor's ABO types and classically presents with fever,
chest/back pain, nausea, pain, and erythema around the infusion site and
hemoglobinuria.
Anaphylaxis (choice B) is unlikely given that the patient has no known drug
allergies. Also, there is no mention of the bronchospasm or urticaria that
would be expected with this process.
A delayed hemolytic transfusion reaction (choice C) is also unlikely given the
acuity of presentation. This reaction typically presents about 3-10 days
following a transfusion reaction and has a mild course with fever and malaise.
A pulmonary embolus (choice D) becomes less likely given that there is no
mention of dyspnea or impaired oxygenation. There also is no mention of the
classic physical stigmata of a pulmonary embolus such as a loud pulmonic
component of the second heart sound or pulmonic tap.
Transfusion associated lung injury (choice E) is unlikely given the acuity of
symptoms. This process, which typically manifests as an acute lung injury
type pathology, typically presents with a more delayed course.
A 28-year-old woman comes to the office because of a 4-day history of
"itching, burning, and an awful-smelling vaginal discharge". She says that she
and her boyfriend had similar symptoms a few months ago, which resolved
after treatment by his physician. Now, she believes that he is "fooling around,"
because this "disease" has returned. On physical examination her vulva is
erythematous and there are patches of petechiae in the upper vagina and on
the cervix. There is a copious amount of yellowish-green, "frothy", malodorous
vaginal discharge. Examination of the discharge on a saline wet mount will
most likely reveal
A. branching hyphae and spores
B. epithelial cells with clumps of bacteria and "ground-glass" cytoplasm
C. giant multinucleated cells with intranuclear inclusions
D. koilocytes
E. motile, flagellated organisms
Explanation:
The correct answer is E. This patient has the signs and symptoms most
consistent with a Trichomonas vaginalis infection, which is diagnosed by
finding motile, flagellated organisms on a saline wet mount smear preparation.
Patients with T. vaginalis typically experience vulvar itching and burning, a
"frothy" malodorous discharge, dysuria, dyspareunia, and frequency and
urgency of urination. Vaginal and cervical petechiae ("strawberry cervix") may
be present. The treatment is metronidazole and it is important to treat the
partner because T. vaginalis can be transmitted by sex.
Branching hyphae and spores (choice A) are associated with an infection with
Candida albicans, which is characterized by intense pruritus and a thick, white
("cottage-cheese") discharge. This patient's discharge is not consistent with a
Candida infection.
Epithelial cells with clumps of bacteria and "ground-glass" cytoplasm (choice
B) are associated with an infection with Gardnerella vaginalis, which is
characterized by gray-white, "fishy" discharge. Irritation of the vaginal
epithelium is not usually seen. This patient's discharge is not consistent with a
Gardnerella infection.
Giant multinucleated cells with intranuclear inclusions (choice C) are
associated with an infection with herpes simplex virus, which is characterized
by vesicular lesions, ulcers, paraesthesia, and dysuria. The diagnosis is
confirmed with viral cultures and scrapings. Giant multinucleated cells with
eosinophilic intranuclear inclusions are seen when stained with Wright's stain.
A saline wet mount smear preparation is not used to diagnose herpes
infections. Also, this patient's discharge is not consistent with a herpes
infection.
Koilocytes (choice D) are associated with an infection with the human
papilloma virus (HPV) which is characterized by soft, fleshy lesions on the
genital region (condyloma acuminata). The diagnosis is established with a
biopsy of the lesions. A Pap smear may show "koilocytes", which are cytologic
changes associated with HPV. A saline wet mount smear preparation is not
used to diagnose HPV. This patient's signs and symptoms are inconsistent
with HPV.
A 60-year-old man with diabetes and hypertension comes to the clinic
because his wife is worried that his skin is turning yellow. The patient's wife
reports that she first noticed the skin changes about 1 month ago and now she
says "even his eyes look bright yellow!" He drinks a case of beer a week and
smokes 2-3 packs of cigarettes a week. He says he has been feeling well and
denies abdominal pain, nausea, or vomiting. Vital signs are normal. He is a
thin male and the abdominal examination is normal. Laboratory studies show:
The most appropriate test at this time is
A. CT of abdomen and pelvis
B. endoscopic retrograde cholangiopancreatography (ERCP)
C. mesenteric angiography
D. serum CA19-9
E. upper gastrointestinal barium study
Explanation:
The correct answer is A. The presentation of painless jaundice is highly
suspicious for a pancreatic head mass and in particular adenocarcinoma of
the pancreas. Adenocarcinoma of the pancreas accounts for more than 90%
of pancreatic malignancies and jaundice is present in about 65% of patients.
Risk factors for pancreatic adenocarcinoma include smoking and diabetes.
The best initial evaluation for pancreatic masses is by CT of the abdomen and
pelvis.
Endoscopic retrograde cholangiopancreatography (ERCP) (choice B) is not
the best initial test to evaluate for pancreatic masses because it primarily
evaluates the biliary duct system in the liver and pancreas. It will not define a
mass that does not involve the biliary ducts. An ERCP may show a discrete
stricture in the main pancreatic duct with proximal dilatation.
A mesenteric angiography (choice C) is not the best initial test to evaluate for
pancreatic masses because it primarily evaluates the vascular structures that
supply and drain the pancreas and abdominal organs. Angiography can be
useful preoperatively because it may show displacement or encasement of the
pancreatic or duodenal arteries by a mass. The venous phase is also useful if
the superior mesenteric vein or splenic vein is occluded due to tumor
extension.
Serum CA19-9 (choice D) is not the best initial test to evaluate for pancreatic
adenocarcinoma. It is a tumor marker that has been associated with
adenocarcinoma of the pancreas. It is not useful as a screening test, but it has
a high sensitivity and specificity as a marker for recurrent disease or
metastases after the primary pancreatic tumor is resected.
An upper gastrointestinal barium study (choice E) is not the best initial test to
evaluate for pancreatic masses. It is a useful study to evaluate for mucosal
lesions within the esophagus, stomach, and small bowel. In the setting of
pancreatic malignancy or other masses, the upper gastrointestinal barium
study may show a widened loop or an "inverted 3 sign" caused by abnormal
indentation of the pancreas along the medial aspect of the duodenum.
A 31-year-old man undergoes an uneventful standard "open" repair of the hernia using
"mesh plug" technique. As planned, he is discharged home on the same day of surgery.
At his first postoperative visit, he complains of numbness over the upper aspect of his
right thigh and the right side of the scrotum. Physical examination reveals an incision
that is clean and dry without any evidence of infection. The right hemi-scrotum is
moderately edematous and minimally tender, both testicles are normal to palpation.
Motor sensation of his right lower extremity is completely intact. Sensory examination
reveals decreased sensation to touch over the medial aspect of the thigh just below the
inguinal ligament as well as over the right side of the scrotum. He is given instructions to
observe this complaint and follow up in 2 weeks. The injured nerve is most likely the
A. iliohypogastric
B. ilioinguinal
C. lateral femoral cutaneous
D. obturator
E. pudendal
Explanation:
The correct answer is B. Although it is now possible to perform inguinal hernia repairs
laparoscopically, the traditional approach is through an incision over the inguinal canal. At
the time of surgery, the inguinal canal is opened via sharp dissection through the external
oblique aponeurosis, (the anterior wall of the inguinal canal). The ilioinguinal nerve is then
encountered as it runs on the anterior aspect of the spermatic cord. If not properly
identified at time of surgery, it is possible to transect the ilioinguinal nerve or to "trap" it
during closure. If the ilioinguinal nerve is transected on "entrapped" in closure, the patient
will complain of numbness over the nerve's distribution, (i.e. the upper medial aspect of
the thigh and the anterior portion of the scrotum on the affected side).
The iliohypogastric nerve (choice A) is derived from L1 (as is the ilioinguinal nerve), and
runs with the ilioinguinal nerve as they both pierce the transversus abdominis muscle
near the anterior superior iliac spine, and then pass through the internal and external
oblique muscles to supply the skin of the suprapubic and inguinal regions and the
abdominal musculature. The iliohypogastric sends a lateral branch to the skin of the
gluteal region and then continues on, to pass through the superficial inguinal ring.
The lateral femoral cutaneous nerve (choice C) originates from L2 and L3 and is a direct
branch of the lumbar plexus. It enters the thigh deep to the lateral end of the inguinal
ligament, near the anterior superior iliac spine and supplies the skin on the anterior and
lateral aspects of the thigh.
The obturator nerve (choice D) is the nerve of the adductor muscles of the thigh. It arises
from the lumbar plexus (L2, L3, L4), enters the pelvis minor, and then leaves the pelvis
via the obturator foramen. The obturator nerve also sends a small cutaneous branch to
the medial aspect of the mid thigh.
The pudendal nerve (choice E) arises from the sacral plexus (S2, S3, S4), and
accompanies the internal pudendal artery and leaves the pelvis between the piriformis
and coccygeus muscles. The nerve hooks around the sacrospinous ligament to enter the
perineum through the lesser sciatic foramen to supply the muscles of the perineum,
including the external anal sphincter and then ends as the dorsal nerve of the penis or
clitoris. It also supplies some sensation to the external genitalia.
The iliohypogastric, lateral femoral cutaneous, obturator, and pudendal nerves are not
encountered during traditional inguinal hernia repair.
A 33-year-old woman comes to the office because of generalized weakness
and a "pins and needles" feeling in her lower extremities for the past 3 weeks.
She states that she feels "unsteady" on her feet. She exercises daily, rarely
drinks alcohol, and is a strict vegetarian. Since this is the first time you have
met this woman, she tells you that she has not had any major illnesses, but
has been hospitalized multiple times over the past few years for anorexia
nervosa. Her temperature is 37 C (98.6 F), blood pressure is 110/70 mm Hg,
pulse is 60/min, and respirations are 18/min. Examination shows weakness of
the proximal and distal muscles of the lower extremities. There is impaired
proprioception and vibratory sensation. Deep tendon reflexes are increased.
The gait is ataxic. The most likely diagnosis is
A. Guillain-Barre syndrome
B. Lambert-Eaton syndrome
C. myasthenia gravis
D. polymyositis
E. subacute combined degeneration of the spinal cord
Explanation:
The correct answer is E. This patient most likely has subacute combined
degeneration of the spinal cord, which is a condition caused by vitamin B12
deficiency. It is most often caused by pernicious anemia, but it may be
acquired in patients with strict vegetarian diets or small bowel disease. The
clinical manifestations include weakness, paraesthesias, loss of vibratory
sensation, increased deep tendon reflexes, and extensor plantar responses.
The gait is ataxic. Mental changes may also occur. The diagnosis is usually
established by measuring serum vitamin B12 levels. The treatment is vitamin
B12 replacement.
Guillain-Barre syndrome (choice A) is an acquired demyelinating neuropathy
that usually follows a viral respiratory infection or immunizations. It is
characterized by ascending weakness. Sensation is intact. A main diagnostic
clue is absent deep tendon reflexes.
Lambert-Eaton syndrome (choice B) is a neuromuscular disorder that causes
proximal muscle weakness, ptosis, and diplopia. Deep tendon reflexes are
depressed or absent. Repetitive nerve stimulation shows increased
responses. Small-cell carcinoma of the lung has been associated with this
disorder.
Myasthenia gravis (choice C) is a neuromuscular disorder due to
autoantibodies to the acetylcholine receptors. It is characterized by weakness
and easy muscle fatigability. The extraocular and eyelid muscles are affected,
leading to diplopia and ptosis. Deep tendon reflexes are preserved.
Polymyositis (choice D) is a skeletal muscle disorder that is characterized by
progressive proximal muscle weakness. Patients often complain of difficulty
climbing stairs and brushing hair. Ocular muscles are generally not affected.
Creatine kinase is elevated. Muscle biopsy and electromyography confirm the
diagnosis.
A 54-year-old African American man comes to the office complaining of
swelling in his left scrotum. He states that the swelling has slowly gotten
worse over the past 6 months and he can no longer feel his left testicle. As per
the patient, the swelling itself does not cause pain. However, the swollen
scrotal skin is rubbing against his thigh causing an irritation. The patient's
urologic history is significant for 2 episodes of epididymitis in the past 5 years.
He denies any trauma to the scrotum, dysuria, hematuria, infertility, or prior
similar episodes. There are no constitutional symptoms elicited with further
questioning. The patient is afebrile and on examination the left hemi-scrotum
is obviously enlarged and the scrotal skin is tense. There is no erythema of
the scrotum. The left testicle is non-palpable. You are able to transilluminate
light through the left scrotal mass. The mass is not reducible through the
inguinal ring and it does not change in size or consistency with Valsalva or
when the patient lies down. The right testicle is descended and normal to
palpation. There is no urethral discharge, inguinal adenopathy, or
abnormalities on rectal examination. Urinalysis and laboratory values are
normal. The most likely underlying cause of this patient's scrotal swelling is
A. dilatation of the pampiniform plexus
B. fluid collection within the tunica vaginalis
C. renal-cell carcinoma with invasion into the left renal vein
D. testicular neoplasm
E. torsion of the spermatic cord
Explanation:
The correct answer is B. This patient has a hydrocele. A hydrocele is a
collection of fluid within the tunica (or processus) vaginalis. The diagnosis is
made by finding a rounded cystic intrascrotal mass that is not tender unless
underlying inflammatory disease is present. The mass transilluminates,
helping to differentiate it from a testicular neoplasm. If the fluid is allowed to
continue to build up, then it may eventually become large enough to prevent
appropriate evaluation of the testicle. If a hydrocele develops in a young man,
without apparent cause, then careful evaluation of the testicle and epididymis
should be done in order to rule out cancer or infection. If the exact diagnosis is
in question, then a scrotal ultrasound should be performed.
Varicocele is caused by dilatation of the pampiniform plexus (choice A). It is
more common on the left side. On exam these patients have a mass of
dilated, tortuous veins lying posterior to and above the testis. It may extend up
to the external inguinal ring and is often tender. The degree of dilatation can
be increased by the Valsalva maneuver. In the recumbent position, venous
distention disappears. The sudden development of a varicocele in an older
man is sometimes a late sign of a renal tumor that has invaded the left renal
vein, thereby obstructing left spermatic vein drainage (choice C).
Testicular neoplasm (choice D) as previously stated does not transilluminate.
Also, the history associated with a testicular tumor usually involves a dull,
heavy feeling within the testicle. These masses have the potential to grow
rapidly. Some testicular tumors are associated with a small hydrocele. With a
testicular tumor, the testicle is hard, enlarged, and irregular in shape.
Testicular torsion (choice E) occurs when the spermatic cord twists, causing
strangulation of the blood supply to the testis. Examination reveals a swollen,
tender testicle that is retracted upward. It is more common in young boys who
develop acute testicular pain.
A 41-year-old-man comes to the clinic because of right facial weakness for the
past 6 days. He denies any pain or changes in hearing. The patient visited the
office last week because of an upper respiratory infection for which you
advised symptomatic therapy. He has no significant past medical history and
takes no medications. Vital signs are: temperature 37 C (98.6 F), blood
pressure 90/70 mm Hg, pulse 90/min, and respirations 15/min. Oxygen
saturation is 99% on room air. Physical examination reveals weakness of the
right side of the face with a droop. The left side of the face is normal. The
remainder of a complete neurologic examination and a mini-mental status
examination are normal. The next step in the management of this patient is to
A. administer prednisolone intravenous pulse therapy
B. advise him to take aspirin
C. order a CT scan of the head
D. order an MRI of the brain
E. order a nerve conduction study
F. prescribe ampicillin
G. provide reassurance and close clinical follow up
Explanation:
The correct answer is G. This patient has Bell's palsy, an idiopathic
inflammatory neuropathy of the facial nerve usually following a viral upper
respiratory infection. Over 90% of patients recover without residual symptoms
in 1 month. Oral prednisone with or without acyclovir is sometimes used to
shorten the recovery period and possibly improve the outcome. However, this
is based on a limited amount of studies.
Prednisolone therapy (choice A) is not indicated. High-potency intravenous
steroids are not used in Bell's palsy as they have no proven benefit and many
side effects. Low-potency oral steroids are used anecdotally, although they
have no proven benefit.
Aspirin (choice B) has no direct role in the treatment of Bell's palsy.
A CT of the head (choice C) is not necessary in the evaluation of Bell's palsy.
Focal facial nerve neuropathy following an upper respiratory infection is the
classic presentation of Bell's palsy. The presence of other neurologic
symptoms or signs would point to a primary neurologic process and would
then warrant some type of neuroimaging.
An MRI of the brain (choice D) is not necessary in the evaluation of Bell's
palsy. Focal facial nerve neuropathy following an upper respiratory infection is
the classic presentation of Bell's palsy. The presence of other neurologic
symptoms or signs would point to a primary neurologic process and would
then warrant some type of neuroimaging.
Nerve conduction studies (choice E) are not necessary in the evaluation of
Bell's palsy.
Ampicillin (choice F) is not indicated because a facial nerve neuropathy is not
due to a bacterial infection. The patient should be managed conservatively for
Bell's palsy.
A 17-year-old girl is brought to the office by her mother with a chief complaint of "severe
acne that never responds to oral antibiotics." The patient's mother refuses to allow a trial
of oral contraceptives for the fear that it will encourage her daughter toward sexual
behavior with her boyfriend. The patient has heard that "the pill" makes you gain weight,
"which is not acceptable." They both have heard about isotretinoin and would like to be
started on this medication as soon as possible. She has multiple inflammatory, deepseated, cystic lesions on her face with evident pitted scarring. She has similar lesions on
her chest and back. On further questioning, she denies being sexually active with her
boyfriend and does not want to be on any form of birth control. Laboratory studies,
including a liver function test and complete blood count, are unremarkable. Your nurse
tells you the urinary HCG is negative from today's visit. You should:
A. Explain to her and her mother that the Federal Drug Administration (FDA) requires
effective contraception used one month before, during, and one month after isotretinoin
therapy
B. Give her more oral antibiotics
C. Inject some of her cysts and reassure her of spontaneous improvement within 1-2
years
D. Prescribe isotretinoin and have her follow up in 1 month with urinary HCG and
blood work
E. Tell her that isotretinoin is only for patients who have failed courses of oral
contraceptives with oral antibiotics
Explanation:
The correct answer is A. Isotretinoin is a vitamin A derivative that has been associated
with a high risk of birth defects. Since a number of pregnancies were reported in females
on isotretinoin, the Food & Drug Administration now emphasizes the importance of two
forms of effective contraception in females of reproductive age unless absolute
abstinence is chosen (choice A).
Oral antibiotics (choice B) are not a good choice because this patient has already been
on oral antibiotics with resultant nodulocystic acne and evidence of scarring. In this case,
isotretinoin is the most effective agent, which will reduce follicular keratinization and
sebum production.
Intralesional injection with steroid solution (choice C) is not correct because while this
modality will offer her temporary improvement in the areas injected, the number of acne
scars she will develop if not treated properly will have a deep impact on her psychosocial
development.
Prescribing isotretinoin (choice D) without discussing contraceptive issues further at this
point is incorrect, as stated above.
Prior to starting patients on isotretinoin, it is useful to try less teratogenic medications
such as oral antibiotics, but there are no specific rules requiring patients to fail both oral
contraceptives and antibiotics prior to initiation (choice E).
A 57-year-old woman with diabetes and nephrolithiasis is admitted to the medical
services for evaluation of her chest pain that began when playing with her grandchild.
She had a previous myocardial infarction and is status-post a three-vessel bypass two
years prior. Her medications include atenolol, lisinopril, allopurinol, and atorvastatin daily.
You are called to the patient's room because the patient is currently complaining of chest
pain. She reports that while talking on the telephone, she became very angry with her
daughter and developed chest pain. On arrival she is lying in bed and appears
uncomfortable. She is diaphoretic and appropriately anxious. Her blood pressure is
190/110 mm Hg and pulse is 110/min. She has an S4 gallop and scant bi-basilar rales.
An electrocardiogram shows sinus tachycardia with a left axis deviation. Voltage criteria
are met for LVH and there are ST segment depressions of 2.5mm in leads V1-V5. The
most appropriate next step in management is to
A. administer verapamil, intravenously
B. give furosemide, intravenously
C. give morphine, intravenously
D. give nitroglycerin, intravenously
E. obtain a chest radiograph
Explanation:
The correct answer is D. This patient is having cardiac ischemia in the setting of
increased myocardial oxygen demand. She has known coronary disease and her vital
signs at the time of her angina episode show hypertension and tachycardia. The patient
must have her blood pressure lowered acutely to the goal of terminating the angina with
nitroglycerin.
Verapamil (choice A) is a calcium channel blocker that is used most often to slow the
ventricular rate in patients with atrial fibrillation. These drugs are generally contraindicated
in an acute myocardial ischemia as they tend to be associated with worse outcome.
Furosemide (choice B) may work to relieve some pulmonary congestion that this patient
has as a result of her ischemia and increased left-sided filling pressures. It however fails
to address the primary problem of her increased demand.
Morphine (choice C) is usually given to relieve pain, but nitroglycerin is more important at
this time because it can relieve pain and also reduce the underlying ischemia.
A chest radiograph (choice E) would not be without benefit in this emergent situation, it
has no role in altering any decision making or therapeutic intervention. This patient must
have her blood pressure and heart rate lowered.
You are seeing a 34-year-old woman in the emergency department who is
now 10 days post emergent C-section for preeclampsia at 29 weeks, and
presents with a 3-day history of a headache. She describes it as "pain starting
at the right temple and shooting through the head to the left temple". There is
some associated photophobia and phonophobia. At its worst, the headache is
a 10 of 10 on the pain scale, with 10 being the worst pain that she ever felt in
her life. This is similar to a headache she was having a month ago during her
pregnancy. There is no relief with acetaminophen. Prior to delivery she had
numbness of her right cheek that was transient. Her blood pressure is 160/90
mm Hg. Examination shows bilateral papilledema. The most appropriate next
step in management is to
A. begin intravenous heparin
B. call a neurosurgical consult
C. discharge the patient with a prescription for naratriptan
D. discharge the patient with a soft cervical collar and prescription for
metaxalone
E. order a brain MRI and MRV
Explanation:
The correct answer is E. This presentation is concerning for venous sinus
thrombosis. MRV is the best noninvasive test to confirm this diagnosis. If the
MRI/MRV is negative, then she should have a lumbar puncture to rule out
idiopathic intracranial hypertension (pseudotumor cerebri).
Intravenous heparin (choice A) is the usual treatment for a venous sinus
thrombosis, however, the diagnosis should be confirmed first if possible.
Therefore, a brain MRI and MRV is the most appropriate next step in
management.
A neurosurgical consult (choice B) is not indicated in this case.
Naratriptan (choice C) is used for the acute treatment of migraine headaches.
This is atypical for a migraine, but she should be further evaluated before she
is discharged and treated.
Soft cervical collar and metaxalone (choice D) are used to treat headaches
from cervical muscle spasms. There was no neck muscle spasms noted on
exam, and she should be further evaluated before she is discharged.
A 35-year-old woman comes to the office asking you to drug test her 12-yearold son. She states that her son is normally a very kind and interactive child,
however, for the last 3 months, he has become increasingly withdrawn. He is
in his room most of the time except for when he is at school. His grades have
dropped from an A- average to C- this past semester. He refuses to see any
of his friends and does not even eat much during dinner anymore. She has
confronted her son multiple times about his situation, but he continually denies
everything, including drug or alcohol use. The mother is visibly upset at this
situation and is tearful about it. She tells you that you are her son's "only help."
The most appropriate response to the mother's request is:
A. "Just bring your son in and we can then get a urine test for alcohol and
substance use."
B. "I am sorry but I can't legally test your son for any drugs without him
consenting to it first."
C. "It is possible that your son may be suffering from depression and I think
that you should bring him in for me to talk to him."
D. "These are classic symptoms for schizophrenia. The best thing to do
would be to start him on some anti-psychotic medications."
E. "Your son is most probably abusing drugs and alcohol, so you should
send him to a rehabilitation facility immediately."
Explanation:
The correct answer is C. The mother is right to be concerned over her son's
change in behavior. Certainly drugs and alcohol abuse can cause such
dramatic behavioral changes. However, more likely would be a mental illness
such as depression. The DSM–IV criteria for major depressive episode
includes that at least one of the symptoms is either (1) depressed mood or (2)
loss of interest in pleasure. The patient would also need to have 5 (or more) of
the symptoms such as depressed mood, weight loss, insomnia, fatigue,
suicidal tendencies, etc. Additionally, the symptoms should cause clinically
significant distress or social impairment. It would also be important to screen
the son for suicidality when the mother brings him in.
"Just bring your son in and we can then get a urine test for alcohol and
substance use." (choice A) would be appropriate if the son consented to the
drug test. However, covert use of the son's urine would only cause him to
distrust not only his mother but also the physician, making any future
interventions more difficult.
"I'm sorry but I can't legally test your son for any drugs without him consenting
to it first." (choice B) is not true because the child is a minor and the parent is
consenting to a drug test. However, it wouldn't be going to the root of this
child's problem since he most likely suffers from a depressive disorder.
"These are classic symptoms for schizophrenia. The best thing to do would be
to start him on some anti-psychotic medications." (choice D) would be a
premature statement since the diagnosis of schizophrenia requires both
positive and negative symptoms occurring for 6 months. However, further
questioning of the child may reveal that he is having a beginning prodrome for
schizophrenia and starting him on anti-psychotics may be appropriate in the
future.
"Your son is most probably abusing drugs and alcohol, so you should send
him to a rehabilitation facility immediately." (choice E) is inappropriate given
the high likelihood that the son is suffering from depression. It is important to
make sure that his depression is not due to the direct effects of a substance
(e.g., a drug of abuse, a medication). However, evidence of substance abuse
should first be obtained through discussion with the son or consented
toxicology screens.
A 21-year-old woman comes to the clinic because of painful urination. She also reports
that her urine has progressively turned from pink to red over the past 2 days. She has
never had symptoms like this before. She is sexually active with 1 partner and they use
condoms for birth control. She has a past medical history of depression treated with
psychotherapy. Vital signs are: temperature 38.9 C (101.9 F), blood pressure 100/70 mm
Hg, pulse 102/min, and respirations 12/min. Physical examination reveals left-sided
costovertebral angle tenderness. The pelvic exam is unremarkable. Laboratory studies
show:
The next step in the management of this patient should be
A. a blood culture and spinal fluid analysis
B. a blood culture and urinalysis
C. an MRI of the lumbar spine
D. an ultrasonography of the kidneys
E. an x-ray of the lumbar spine
Explanation:
The correct answer is B. Blood cultures and urinalysis are necessary because this patient
has symptoms of pyelonephritis. The presentation of urinary symptoms, fever,
leukocytosis, and bandemia is clinically diagnostic of pyelonephritis. A urine culture
should also be obtained prior to starting antibiotics. Empiric broad spectrum antibiotics
should be started and coverage can be narrowed when culture and sensitivity data are
available. A renal ultrasound can also be performed at a later time to evaluate the
kidneys if the patient does not respond to antibiotic therapy. The renal ultrasound is
useful to evaluate for the complications of pyelonephritis such as abscess or obstruction.
This patient has no mental status changes or headaches that would necessitate a lumbar
puncture (choice A)
This patient has no radiculopathy or back pain that would necessitate an MRI of the
lumbar spine (choice C). Costovertebral or flank tenderness is often a sign of kidney
inflammation or infection.
An ultrasonography of the kidneys (choice D) is not necessary at this time. A renal
ultrasound can be performed if the patient does not respond to antibiotic therapy. The
ultrasound is useful to evaluate for complications of pyelonephritis such as abscess or
obstruction.
The physical examination of the back reveals flank, not lumbar spine pain. Hence, an xray of the lumbar spine (choice E) is not necessary.
A 63-year-old woman comes to the office because of a 3-week history of a "dull, achy"
headache. She says that it started out as an intermittent headache that was exacerbated
by bending down, lifting heavy objects, sneezing, defecating, and coughing, but lately it
has become constant. She cannot associate the headaches with food or hunger,
alcohol, weather or barometric pressure changes, sounds, or irregular sleep patterns.
She is generally very healthy, but recalls having some nausea and vomiting a few weeks
before the headaches started. She has never had headaches before. She does not take
any medications, rarely drinks alcohol, and exercises regularly. Her temperature is 37.0
C (98.6 F), 130/80 mm Hg, pulse is 70/min, and respirations are 15/min. Physical
examination is unremarkable. An erythrocyte sedimentation rate, complete blood count,
and electrolytes are normal. A trial of oral prednisone, sublingual ergotamine, and oral
sumatriptan is ineffective. The most appropriate next step is to
A. administer glucocorticoids, intravenously
B. administer sumatriptan, intramuscularly
C. give her oxygen inhalation therapy
D. obtain a temporal artery biopsy
E. order an MRI of the head
F. perform a lumbar puncture
Explanation:
The correct answer is E. In any patient with a new headache that is associated with
nausea and vomiting and exacerbated by exertion or positional changes, a brain tumor
should be suspected. This is the typical presentation of a posterior fossa brain tumor.
Weight loss, systemic symptoms, and an elevated erythrocyte sedimentation rate are not
typically associated with a primary brain tumor. Papilledema, caused by increased
intracranial pressure, is rarely found in patients over 55 years old with an intracranial
mass. An MRI or CT scan of the head will establish the diagnosis.
Administration of glucocorticoids, intravenously (choice A) is the appropriate treatment to
prevent blindness in a suspected case of temporal arteritis. Temporal arteritis typically
presents with a unilateral headache, myalgias, jaw pain, fever, and weight loss. A tender,
reddened temporal artery is often found. Visual changes may occur and blindness is a
feared complication. This patient does not have any of the usual systemic symptoms
associated with temporal arteritis. The diagnosis of temporal arteritis is established with a
temporal artery biopsy (choice D). The erythrocyte sedimentation rate is typically elevated
in temporal arteritis
Administration of sumatriptan, intramuscularly (choice B) is the treatment for a migraine
headache which typically presents with a throbbing headache, nausea, vomiting,
photophobia, and functional impairment. An aura consisting of hallucinations and
scotomas sometimes occurs before the headache. An intermittent headache that
becomes constant is not the typical pattern of a migraine. The sublingual ergotamine and
oral sumatriptan that this patient was taking are typically effective in treating a migraine.
Oxygen inhalation therapy (choice C) is the treatment for a cluster headache. A cluster
headache is an episodic headache that typically presents with a few short headaches a
day for a few weeks that is associated with periorbital pain, reddening of the eye, and
lacrimation. This patient's history is inconsistent with a cluster headache.
A lumbar puncture (choice F) should be performed in a suspected case of meningitis and
possibly in a suspected intracranial hemorrhage, if the CT scan fails to show the bleed.
Meningitis often presents with a headache, nuchal rigidity, and photophobia. Fever and a
rash may be present. An intracranial hemorrhage may present with a headache and
nuchal rigidity, seizures, and confusion. A CT scan of the head usually establishes the
diagnosis. The patient's history is inconsistent with meningitis and is more consistent with
a brain tumor than an intracranial bleed. A lumbar puncture should not be performed in
this patient or any patient with a suspected brain tumor because it may cause brain
herniation.
A 31-year-old woman comes to the office for an effective, reversible form of
contraception. She has recently gotten married and she and her husband are
tired of using condoms. She took oral contraceptive pills (OCPs) while she
was in college and she wants you to prescribe them for her again. She is
generally very healthy and has no personal or family history of cancer or
thromboembolic disease. She exercises 5 days a week, eats a low-fat diet,
drinks a couple of glasses of wine each night with dinner, does not take any
medications, and smokes a pack of cigarettes a day. You flip through her
chart and note that she had an episode of pelvic inflammatory disease 4 years
ago. Her blood pressure is 110/80 mm Hg. Physical examination is
unremarkable. Pap smear and cervical cultures are normal. A urinary
pregnancy test is negative. At this time you should
A. encourage her to stop smoking, explain risks, benefits, and synergistic
effects, and prescribe OCPs
B. have her schedule an appointment on the first day of her next menstrual
period
C. inform her that an intrauterine device is a better choice than OCPs
because she smokes
D. recommend condoms, as they have the same estimated failure rates as
OCPs and are safer for smokers
E. tell her that you will prescribe oral contraceptive pills when she quits
smoking
Explanation:
The correct answer is A. Patients who take OCPs and smoke are at an
increased risk for having a myocardial infarction or stroke. These risks are
greatest at 35 years of age and older. As with all smokers, she should be
encouraged to stop smoking. You should explain risks, benefits, and
synergistic effects, and then you can prescribe OCPs.
There is no reason for you to have her schedule an appointment on the first
day of her next menstrual period (choice B). All of her laboratory studies were
normal, including a pregnancy test, so after you counsel her, you can give her
a prescription. Even if the pill should be started on the first day of the next
menstrual period, there is no reason not to give her the prescription now.
It is incorrect to inform her that an intrauterine device (IUD) is a better choice
than OCPs because she smokes (choice C) because a prior history of pelvic
inflammatory disease is a contraindication for an IUD.
The estimated failure rates (choice D) are not the same for OCPs and
condoms. The estimated failure rate is 1-3% for OCPs and 5-15% for
condoms.
While she is at an increased risk for a myocardial infarction and stroke, it is
inappropriate to tell her that you will prescribe oral contraceptive pills when
she quits smoking (choice E). Smoking in a 31-year-old woman is not a
contraindication for OCPs. You should encourage her to quit, explain the risk
and benefits, and prescribe OCPs.
You are seeing a 79-year-old woman with hypertension, diabetes, coronary artery
disease, and rheumatoid arthritis in your office for a routine follow-up visit. Her
medications include spironolactone, amiloride, NPH insulin, aspirin, prednisone, and
ketorolac. Her temperature is 37 C (98.6 F), blood pressure is 99/56 mm Hg, pulse is
58/min, and respirations are 19/min. Physical examination is unremarkable. An
electrocardiogram shows a sinus rhythm and non-specific ST and T wave abnormalities.
Her serum sodium is 136 mEq/L and serum potassium is 5.8 mEq/L. A review of her
laboratory values over the past 6 months indicates that her serum potassium has been
gradually increasing. The most appropriate management at this time would be to
A. add clonazepam to her current therapy
B. add metoprolol to her current therapy
C. discontinue the amiloride
D. discontinue the ketorolac
E. discontinue the prednisone
Explanation:
The correct answer is C. Spironolactone, an aldosterone antagonist, causes an increase
in serum potassium. Amiloride, a potassium sparing diuretic, has a similar effect. The use
of both these agents can lead to potentially dangerous hyperkalemia. If hyperkalemia
does manifest, one of them should be discontinued.
Clonazepam (choice A), a benzodiazepine anxiolytic, is not warranted since there is no
evidence that the patient is anxious.
Metoprolol (choice B) will not decrease serum potassium. Infact, beta-blockers can
increase serum potassium. The patient's heart rate and blood pressure also do not
mandate a beta-blocker at this juncture.
Discontinuation of ketorolac (choice D), a nonsteroidal antiinflammatory agent, is
unnecessary since it is not associated with hyperkalemia.
Discontinuation of prednisone (choice E) is not needed, since prednisone does not cause
or worsen hyperkalemia.
A 25-year-old woman comes to the office because of a 3-day history of
extreme vulvar itching and a thick, white vaginal discharge. She says that she
is so uncomfortable that she cannot sit still at work because she "constantly
feels the need to itch." You notice in her chart that she has had 2 similar
episodes over the past 6 months, and her boyfriend was simultaneously
treated both times due to her intense fear of reinfection. She plays on the
company softball team, goes hiking with her boyfriend and dog, and is
currently training for a marathon. She says that she showers and changes her
clothing immediately after working out, does not wear tight pants, and only
uses panty liners during her menstrual period. She has been taking
doxycycline daily for acne for the past 2 months, and was taking erythromycin
for acne during the 6 months prior to switching to doxycycline. Physical
examination shows erythematous vulvar and vaginal walls, and a thick, white
vaginal discharge that is adherent to the vaginal walls. The pH of the
discharge is 4.8. Laboratory studies show:
She is concerned that she has had recurrent infections. The most important
information to share with this patient is that
A. her boyfriend may be having sexual relations with other people and
transmitting the organism back and forth
B. she probably has early diabetes and she needs to return for serial
urinalyses during the next 6 months
C. she should have an HIV test to rule out an underlying immunodeficiency
D. the condition is most likely due to the alteration in vaginal flora and pH
caused by the antibiotics
E. this chronic condition is most likely due to the warm, moist environment
caused by the panty liners
Explanation:
The correct answer is D. This patient has Candida vaginitis that is most likely
due to the long-term use of broad-spectrum antibiotics. The chronic use of
antibiotics alters the finely balanced vaginal environment that is normally very
acidic from the vaginal flora. Lactobacillus is one of the main components of
normal vaginal flora that helps to maintain this acidic environment. Any factors
that affect the normal pH and decrease the lactobacilli, increase the risk that a
vaginal infection with a pathologic organism may occur. Antibiotic use is a
known risk factor for altering the normal microbiology of the vagina. Some
physicians prescribe prophylactic therapy with an antifungal agent when they
prescribe long-term systemic antibiotics.
It is inappropriate to tell her that her boyfriend may be having sexual relations
with other people and transmitting the organism back and forth (choice A)
because her recurrent infections are most likely due to the long-term use of
broad-spectrum antibiotics for acne. These infections coincide with the use of
the antibiotics. Also, since her boyfriend was treated during her previous
infections, it is unlikely that he is the source of reinfection. If she had no other
risk factors for Candidal infections, this may be a possibility, but this is not the
case.
She should not be told that she probably has early diabetes and that she
needs to return for serial urinalyses during the next 6 months (choice B)
because her glucose levels are normal and she has been taking broadspectrum antibiotics for months, which is a known risk factor for Candidal
infections. Serial urinalyses for the next 6 months is not indicated.
If she had chronic Candidal infections and no known risk factors that alter the
vaginal environment, it may be appropriate that she have an HIV test to rule
out an underlying immunodeficiency (choice C). However, her recurrent
infections are most likely due to the long-term use of broad-spectrum
antibiotics for acne.
Since she only uses panty liners during her menstrual period, it is unlikely that
this chronic condition is most likely due to the warm, moist environment
caused by the panty liners (choice E). Panty liners create a warm, moist
vaginal environment that may increase the risk of infections with Candida.
However, this usually occurs in individuals who wear panty liners every day.
This answer is also unlikely because she has been taking broad-spectrum
antibiotics for months, which is a known risk factor for Candidal infections.
A 37-year-old woman comes to the office because of a "burning sensation" in
the chest for the past 3 months. The "burning" typically begins in the "upper
stomach and travels up to the neck." The symptoms worsen when she lies
down to go to sleep. She is a chef at a local American restaurant, has 3
children, and has been married for 12 years. She "tries" to eat a healthy diet,
but it is difficult because she is around food all day and night. She has no
chronic medical conditions, takes no medications, and does not drink alcohol
or caffeine-containing beverages. She recently quit smoking. Her temperature
is 37.0 C (98.6 F), blood pressure is 120/80 mm Hg, pulse is 65/min, and
respirations are 14/min. Physical examination is unremarkable. An
electrocardiogram is unremarkable. A complete blood count and metabolic
profile are normal. Serologic testing for H. pylori is negative. The most
appropriate next step is to
A. order ambulatory esophageal pH testing
B. order an upper gastrointestinal barium radiograph
C. recommend elevation of the head of bed and avoidance of food before
bedtime
D. schedule an upper endoscopy
E. schedule esophageal manometry
Explanation:
The correct answer is C. This patient complains of the classic symptoms of
gastroesophageal reflux disease (GERD). Reflux disease is usually worse at
night because the recumbent position allows gastric acid contents to go up
into the esophagus. Since all of the tests ordered in the case were normal,
you should first recommend non-pharmacologic therapy before continuing with
further diagnostic studies. Elevation of the head of bed, avoiding eating before
bed, and avoiding alcohol, tobacco, chocolate, and caffeine should all be
recommended. Alcohol, tobacco, chocolate, and caffeine all lower the lower
esophageal sphincter pressure leading to gastric reflux. If these measures are
ineffective, pharmacologic therapy with a H2 blocker such as cimetidine,
famotidine, or ranitidine is indicated. For more severe symptoms, a proton
pump inhibitor, such as omeprazole or lansoprazole, is indicated.
Ambulatory esophageal pH testing (choice A) is usually reserved for patients
who fail nonpharmacologic and pharmacologic management.
An upper gastrointestinal barium radiograph (choice B) is useful in detecting
esophageal rings or strictures, which typically present with dysphagia. This
patient complains of heartburn, not dysphagia.
An upper endoscopy (choice D) is usually indicated only after the failure of
nonpharmacologic and pharmacologic management for GERD and when a
patient has GERD for >5 years, and upper endocsopy is recommended to
screen for Barrett's metaplasia. However, it is not indicated at this time.
Esophageal manometry (choice E) is typically reserved for cases of GERD
when surgical therapy is being considered.
A 71-year-old man is postoperative day number 12 from a heart transplant.
The patient has a long-standing history of ischemic cardiomyopathy and
successfully underwent a 5-hour transplant from a 22-year-old donor. During
the procedure he was started on his immunosuppressive therapy that has
continued. In the immediate postoperative period he did well. He was
extubated on day number 1, had his pulmonary artery and radial arterial
catheters removed on day number 3, and was transferred from the coronary
care unit on day number 4 with excellent pain control. Over the past 3 days
however, he has had increasing fever, lethargy, and mediastinal tenderness.
The sternal wound appears mildly erythematous but nonsuppurative. On
palpation, the incision site is tender. The most appropriate therapy is at this
time is
A. broad-spectrum antibiotics
B. decreasing dose of immunosuppressive drugs
C. initiate antifungal therapy
D. surgical debridement
E. there is no therapy indicated
Explanation:
The correct answer is A. This patient likely has mediastinitis, a common
postoperative complication after heart transplant. The combination of the
nature of the wound (median sternotomy) with the profound
immunosuppression creates a furtive environment for mediastinal infection.
Such patients require broad-spectrum antibiotic therapy.
Because of the transplant and the possibility of acute-rejection, decreasing the
dose of immunosuppressive drugs (choice B) is not tenable.
Bacteria cause most mediastinal infections in the immediate and intermediate
postoperative periods. Therefore, fungal infections in transplant recipients and
the need for antifungal therapy (choice C) are not usually an issue until 6-9
months of immunosuppressive therapy.
Surgical debridement (choice D) is usually required when the sternum
undergoes dehiscence or there is necrotic tissue present that can become a
culture medium for additional infections. There is no indication that this
patient's infection is that severe.
Because mediastinitis is a serious infection and can cause sternal dehiscence
and chronic infection, antibiotic therapy is indicated despite concerns for
selecting for resistant organisms (choice E).
A 44-year-old woman comes to see you for routine check up. While in your
office, she starts crying hysterically, stating she has not been able to sleep for
the last few months. Also, her mind has been "racing," her palms and soles
are sweaty at all times, and she has thinning of her hair. She denies any
alcohol or tobacco use, but admits to 2 cups of coffee a day. Her mother has
bipolar disorder and her aunt has obsessive-compulsive personality disorder.
Her blood pressure is 130/80 mm Hg and pulse is 100/min. There is notable
exophthalmos bilaterally. He skin appears moist and warm. On the pretibial
regions, there is a woody induration with pitting edema. The most appropriate
next step in evaluation is to
A. determine testosterone level
B. give her aluminum chloride for her sweaty palms and reassure her this
is all psychological
C. order thyroid function tests
D. refer her to an ophthalmologist for evaluation of the exophthalmos
E. refer her to a psychiatrist for evaluation of a psychiatric disorder
Explanation:
The correct answer is C. Ordering a thyroid function test is correct, because
this patient demonstrates not only physiological hyperthyroid changes (i.e.
increased heart rate), but also cutaneous findings classic for Graves disease.
Skin changes are distinctive in hyperthyroidism. The cutaneous surface is
warm, moist, and smooth textured. Palmar erythema or facial flushing may be
seen. The hair is thin and has a downy texture and nonscarring alopecia may
be observed. Graves disease has a female to male ratio of 7:1. Thyroid
acropachy is characterized by digital clubbing and diaphyseal proliferation of
the periosteum in acral and distal long bones (tibia, fibula, ulna, and radius).
Pretibial myxedema consists of bilateral localized, cutaneous accumulations of
glycosaminoglycans and occurs in 4% of patients who have or have had
Graves disease. Improvement in plaques of pretibial myxedema have resulted
from intralesional injections of triamcinolone acetonide and with clobetasol
solution under Duoderm occlusion, applied once weekly for 4-6 weeks.
Systemic steroids are of no benefit.
Checking testosterone level (choice A) is incorrect, because an elevated
testosterone level would only account for hair thinning, but not her other
physiologic and cutaneous changes.
Giving her aluminum chloride (choice B) is incorrect, because this solution will
only alleviate sweaty palms and soles for this patient, but does not help to
diagnose and treat the underlying issue.
Referring her to ophthalmology (choice D) is incorrect, because eventually this
patient may need to see an ophthalmologist for severe exophthalmos, but
diagnosing Graves disease is the more appropriate first step for this patient.
Despite her family history of psychiatric illnesses, referral to psychiatry (choice
E) is incorrect, because this patient's manic appearance is most likely
secondary to thyroid hormone imbalance.
A 34-year-old man comes to the office because of "erectile problems." He
says that he and his wife have not had sexual intercourse in months because
he has not been able to have an erection. He says that it is so upsetting that
they have basically stopped trying because it just makes both of them
"depressed". He has seen so many television commercials lately that he
expects a cure in the form of a "little blue pill." He is married, has 3 kids (age
1,3, and 6), works as a narcotics police officer, and competes in triathalons on
the weekends. He takes no medications, rarely drinks alcohol, and has had no
serious medical conditions. Physical examination is normal. The most
appropriate next step in evaluating this patient's erectile disorder is to ask him
A. "Are you feeling unusually anxious lately?"
B. "Are you sexually attracted to your wife?"
C. "Do you have nocturnal or early morning erections?"
D. "Do you love your wife?"
E. "How often do you ride your bicycle?"
Explanation:
The correct answer is C. The main issue in evaluating impotence is
distinguishing between psychological causes and organic causes. The
presence of nocturnal or early morning erections basically eliminates the
organic causes and leads to a diagnosis of psychological impotence. Up to
90% of erectile disorders are due to psychogenic factors. The most common
psychological causes include anxiety and depression. Nocturnal penile
tumescence, which occurs during REM sleep, can be assessed in a sleep lab
or by a stamp test (wrapping stamps around the penis before bedtime and
checking in the morning if the "ring" is broken at any of the perforated areas).
If nocturnal erections are not present, the impotence is most likely due to an
organic cause. The most likely causes are testicular failure,
hyperprolactinemia, medications, alcohol, opioids, nicotine, trauma, priapism,
diabetes, vascular disease, and neurologic diseases, such as diseases of the
spinal cord, and loss of sensory input. Physical examination should include a
detailed genital exam, evaluation for signs of feminization, neurologic, and
vascular exams. "The little blue pill" (Viagra) is not indicated in a patient with
psychogenic erectile disorders.
"Are you feeling unusually anxious lately?" (choice A) is a good question to
follow the question about nocturnal erections (if he is having them).
"Are you sexually attracted to your wife?" (choice B) is a relevant question if
the patient is having nocturnal erections and is not depressed or anxious. "Do
you love your wife?" (choice D) is a question that may be asked if he is having
nocturnal erections and is anxious and depressed. It may be a little blunt and
can probably be asked in a more subtle manner.
"How often do you ride your bicycle?" (choice E) may be relevant because of
neurologic and vascular compromise caused by the seat. However, it is not
the most helpful question to distinguish between organic and psychologic
causes.
A 42-year-old woman who you have been treating for anxiety calls your office
demanding to talk to you . The nurse gets you immediately because she
sounds "out of control." When you finally get to the phone, she is screaming
that, "a plumber is working in her house and he hurt his eye." You ask for the
details of the injury and she says that all she knows is that he "got something
in his eye and he is freaking out." You instruct her to look at the bottle and
read the label to you, but all you can understand from her hysterical speech is,
" pH of 12.2." You hear a man in the background screaming in pain. You try to
calmly explain to her that the most appropriate, immediate management is to
A. call an ambulance to bring him to the hospital, and place a patch over
the affected eye
B. call the ophthalmologist and schedule an appointment for the next
available time
C. flush his eye with a substance that she can find in her cabinet with a pH
of 6.9 or lower
D. pry the eye open and flush it continuously with cold running water before
going to the hospital
E. pry his eye open and vigorously rub the entire eye, especially under the
upper and lower lids
Explanation:
The correct answer is D. This patient most likely has an alkaline eye burn,
which is extremely destructive, and the process of destruction continues as
long as the substance is in contact with the tissues. Immediate removal is
essential, and the best method is massive irrigation. In the emergency room,
sterile saline would be used, but at home, tap water will do. This irrigation is
important even before going to the hospital (because the alkaline fluid will
remain in the eye if you leave the irrigation to the emergency room
physicians).
It is extremely dangerous to call an ambulance to bring him to the hospital,
and place a patch over the affected eye (choice A). The alkaline chemical will
begin to destroy the eye if you do not begin immediate irrigation while waiting
for help.
Alkaline eye burns are emergencies that require immediate irrigation and
medical attention. Calling the ophthalmologist and scheduling an appointment
for the next available time (choice B) will leave too much time for the chemical
to destroy the eye.
It would be inappropriate to tell her to flush his eye with a substance that she
can find in her cabinet with a pH of 6.9 or lower (choice C). Irrigation with cold
tap water is the treatment while waiting for additional medical help (in the
ambulance or at the hospital).
While it is necessary to try to remove as much of the chemical as possible,
telling her to pry his eye open and vigorously rub the entire eye, especially
under the upper and lower lids (choice E) is incorrect. Massive irrigation is
used to dilute the chemical and wash away any particles remaining in the eye.
Gentle swiping may be appropriate later on in the treatment to wipe away any
little remaining particulate matter, however, this may be done after irrigation.
A 12-year-old boy with asthma is brought to the emergency department by his
mother because of intermittent right hip pain for the last 2 weeks. The pain is
non-radiating and worse with activity. It has now become more constant,
worse with weight bearing, and over-the-counter analgesics only give minimal
relief. There is no history of night pain and he denies any recent trauma,
weight change, or any constitutional symptoms such as fever, chills, or night
sweats. He tells you that he went to his pediatrician's office 10 days ago for
the same hip pain and he was told that his physical examination and
laboratory studies, including a complete blood count and erythrocyte
sedimentation rate, were unremarkable. The pediatrician's diagnosis was a
"pulled muscle or tendon" in the right hip region and he was advised to rest.
Now in the hospital, his physical examination shows an obese patient, a
limping gait, a leg length discrepancy of 0.5 cm, and intact motor-sensory
examination of lower extremities bilateral. The right hip region has intact skin
and no focal tenderness to palpation. Passive range of motion of the right hip
is decreased on internal rotation. When the hip is flexed, the thigh externally
rotates. The most appropriate next step in management is to
A. admit him to the hospital for intravenous antibiotic therapy, non-weight
bearing right lower extremity, and obtain an orthopaedic consultation
B. recommend immediate crutch walking and non-weight bearing of right
lower extremity and obtain pelvic x-rays
C. repeat complete blood count and erythrocyte sedimentation rate; if the
results are normal then send him home with 1 week follow-up in the office
D. send him home with a prescription for a 0.5 cm shoe insert and physical
therapy
E. send him home with a 10-day course of oral antibiotics with a 2-week
follow-up in the office
Explanation:
The correct answer is B. A skeletally immature and obese 12-year-old patient
with this history of hip pain is most likely to have a slipped capital femoral
epiphysis (SCFE). This growth plate condition does not occur after skeletal
maturity, and for males, maturity is on average at 16 years of age. The
obligatory external hip rotation with hip flexion is a very common finding in a
slipped epiphysis. Immediate non-weight bearing protects the femoral
epiphysis from further injury and potential osteonecrosis. Common x-ray
findings are consistent with a slippage of the femoral epiphysis. The slippage
has been compared to a scoop of ice cream that has partially slipped from the
cone. A double density sign and a break in Klein's line are two x-ray findings.
Treatment requires internal fixation of the epiphysis with long screws. The
risks of delayed or non-treatment are further slippage and osteonecrosis.
An orthopaedic consult (choice A) will result in discontinuance of the
antibiotics and ordering pelvic x-rays. Non-weight bearing is correct and
protective, but antibiotics are not indicated because this is not an infectious
process.
This patient does not have transient synovitis or any other infectious or
inflammatory processes.Therefore, repeating a complete blood count and
erythrocyte sedimentation rate, and sending him home if the results are
normal with a one week follow up is not necessary(choice C) is incorrect.
Again, a delay in diagnosis will result in significant morbidity.
Leg length discrepancy is very common, and less than 2 cm does not typically
require treatment. Patients are very often misdiagnosed as having muscle
strains or tendon injuries. This delay in diagnosis results in significant
morbidity. Sending him home with a prescription for a 0.5 cm shoe insert and
physical therapy (choice D) does not address the problem.
This patient does not have an infection. Therefore, antibiotics and discharge to
home (choice E) will only delay the correct diagnosis.
A 67-year-old man presents to your office complaining of severe shortness of
breath and a cough. The man is a long-time patient who you have been
treating for chronic obstructive pulmonary disease (COPD). He has a 120
pack-year smoking history. He was recently hospitalized for a flare of his
COPD and he has been intubated in the past for respiratory distress. His other
medical history is notable for diabetes mellitus and hypercholesterolemia. He
reports to you that for the past 3 days he has increasing dyspnea and a fever.
He has gradually developed a cough. On examination, he is moderately
dyspneic at rest and has marked dyspnea on exertion. His blood pressure is
130/80 mm Hg, heart rate is 78 /min, and respirations are 20 /min. The patient
does not appear cyanotic. His lung exam demonstrates a markedly prolonged
expiratory time and diffusely diminished breath sounds. The most appropriate
intervention at this time is to
A. admit the patient to the hospital
B. obtain a chest radiograph and prescribe antibiotics if an infiltrate is
present
C. prescribe oral cefuroxime and oral steroids
D. prescribe oral cefuroxime and see the patient in seven days
E. refer the patient to the local emergency department
Explanation:
The correct answer is A. This patient has long-standing pulmonary disease
and is suffering from some acute respiratory event, either infection or a COPD
flare. His past medical history is enlightening because it can be appreciated
that this patient has required ventilatory support for his pulmonary disease in
the past. In the office, he is clearly in some distress. With his history and
known lung disease, this patient should be admitted to the hospital for further
care and observation.
Similar reasoning holds for obtaining a chest radiograph and prescribing
antibiotics if an infiltrate is present (choice B). Knowing that his distress is due
to pneumonia does not alter the fact that he needs to be treated in the
hospital.
Prescribing oral cefuroxime and oral steroids (choice C), although a
reasonable regimen for a COPD flare, fails to address how this patient's acute
respiratory compromise will be addressed. He has been intubated in the past
for flares such as this and discharging the patient out of your care could result
in serious harm or even death to this patient.
Prescribing oral cefuroxime and see the patient in seven days (choice D) is
clearly inappropriate since the patient is presently in mild to moderate distress
and the cause for this is unknown. Even if he does have an infection, the
antibiotics will take days to be of any assistance.
It would be inappropriate to refer the patient to a local emergency department
(choice E) where he may sit unobserved in triage or may be discharged home
if another physician had a different impression of the patient.
A 57-year-old woman with coronary artery disease associated with
hyperlipidemia comes to the clinic for a scheduled follow-up appointment. She
saw you for the first time approximately 1 month ago to establish care. In the
interim, she was started on hydrochlorothiazide for elevated blood pressure
(confirmed on a repeat nurses visit) and on simvastatin for a fasting LDL of
190. She has a remote history of alcoholism, but denies any alcohol intake
over the past 10 years. Today, she complains of mild, generalized weakness
and states that her shoulders and thighs are "achy". She denies rhinorrhea,
fevers, chills, nausea, vomiting, or diarrhea. While she does not complain of
any dysuria, she states that her urine has been very dark for the past few
days. She denies abdominal or flank pain. Laboratory studies show a mildly
elevated white blood cell count, a normal hematocrit, and normal electrolytes.
Her AST (or SGOT) is 415 and her ALT is 25. Bilirubin and alkaline
phosphatase are within normal limits. The most appropriate next step in
evaluation is to
A. determine creatinine kinase level
B. obtain an erythrocyte sedimentation rate
C. order a GGT level and a serum alcohol level
D. send Hepatitis A, B, and C serologies
E. send her for a right upper quadrant ultrasound
Explanation:
The correct answer is A. AST is less specific for liver than ALT. AST is found
in multiple organs and will be elevated with any muscle injury. In fact, before
the advent of assays for the MB fraction or troponin, AST was used to assess
for myocardial infarction. The patient was recently placed on simvastatin (an
HMG Co-A reductase inhibitor). While these cholesterol-lowering drugs are
generally benign, myositis is a complication that the prescribing physician
must be aware of. The patient's presentation of fatigue and muscle aches fits
the diagnosis of myositis. In addition, an elevated AST with an otherwise
completely normal liver panel should heighten suspicion that the AST is not
coming from the liver. The patient's dark urine is classic for myoglobinuria.
The patient should be treated with intravenous fluids to maintain renal
perfusion, therefore avoiding renal tubular injury from the myoglobin.
Depending on the level of the creatinine kinase, alkalinizing the urine may also
help protect the kidney from injury in this setting. There is no definitive
treatment for the myositis itself. Typically, the myositis resolves after the
offending agent (simvastatin in this case) is discontinued.
An erythrocyte sedimentation rate (choice B) is incorrect. While an erythrocyte
sedimentation rate may be useful as a sensitive marker of inflammation, it is
not specific for any disease process. As this case illustrates, recently
prescribed medicines should always be considered at the top of your
differential diagnosis as the etiology of a new disease process. The
erythrocyte sedimentation rate would not help make the diagnosis, nor would
the result change management.
An elevated AST to ALT ratio may be suggestive of alcoholic liver injury. This
is thought to be due to the fact that ethanol decreases ALT synthesis. In
addition, ethanol is thought to cause mitochondrial damage in the liver, where
AST lives. However, the ratio of AST to ALT in alcoholic hepatitis is more
frequently closer to 2:1. The extremely high ratio in this case (again, with a
normal ALT) points to an extrahepatic process. An elevated GGT can reflect
alcoholic liver damage, but again, this is unlikely in the face of completely
normal bilirubin and alkaline phosphatase. Therefore, ordering a GGT level
and a serum alcohol level (choice C) is not correct.
Hepatitis A, B, and C serologies (choice D) are incorrect because the viral
hepatitides should not cause an isolated level in AST.
A right upper quadrant ultrasound (choice E) is incorrect mainly for the same
reasons as elucidated above. A right upper quadrant ultrasound is useful for
evaluating suspected structural disease. Typically, structural disease of the
liver is suspected when there are clues of hepatic obstruction. Elevated
bilirubin and alkaline phosphatase are typically elevated in hepatic obstruction,
and both are normal in this case.
A recently published study examined the efficacy of a new drug, Nomomigrane, for
treating migraine headaches. For this randomized, double-blind study, 200
migraine sufferers were give Nomomigrane for 6 months, while another 200
migraine sufferers were given a placebo. At the end of the 6 months, all subjects
reported whether or not they had been migraine free for the last 3 months. A key
table from this study is presented below.
The report included the statement that Nomomigrane worked better than the
placebo and assigned a significance level of p < .001. Based on this and other
studies, Nomomigrane was recently approved by the FDA for the treatment of
resistant migraine headaches. You are considering prescribing Nomomigrane for a
patient suffering from severe migraines. As the treatment option is presented the
patient asks, " I know this worked in the research, but what is the chance that the
drug will really work for me?" Based on the presented research the most
appropriate response is:
A. "I'm very excited about the potential for this new drug and think you will be
very pleased with the results."
B. "One can never tell about these things before hand. We'll just have to try it
and see."
C. "The drug was reported effective in 6 out of 10 patients."
D. "The results are strong and convincing. You can have every confidence that
this drug will work for you."
E. "The statistical significance was high in the study. The chance of the drug not
working for you is less than 1 in a 1,000."
F. "This treatment will cut the number of migraines you have been experiencing
in half."
G. "Why do you ask? Do you have concerns about my treatment
recommendation?"
Explanation:
The correct answer is C. This question asks about clinical efficacy. In the
presented study, 120 of the 200 patients given the drug did not report migraines in
the measurement period (120/200 = 60%).
Expressing enthusiasm (choice A and D) about a prescribed treatment does help
to engender confidence in the treatment on the part of the patient. However, this
response does not answer the patient's question.
Of course, individual patients may be the exception to the rule (choice B), but this
research tells us that the drug is likely to be effective. The probability is that the
drug will work for this patient.
Statistical significance is high, less than one per 1,000 chance of Type I error.
However, statistical significance tells us little about clinical significance. Knowing
the "p-value" tells us nothing about the number of patients who get better or the
chance that an actual patient will get better (choice E).
The treatment group did report half the migraines compared with the placebo
group, but this is a misuse of this ratio. The study did not measure the reduction in
migraines within each person. Rather, it examined the presence/absence of
migraines between the treatment and control groups. In short, this study does not
tell us what sort of reduction in migraines an individual patient might expect,
therefore (choice F) is incorrect.
Inquiring after a patient's concerns over treatment (choice G) is a reasonable tack,
but does not answer the question posed by the patient. If the patient asks a
question, you should try to answer it.
A 35-year-old homeless man is admitted to the hospital because of psychosis and
agitation. He received intramuscular haloperidol while in the emergency department 16
hours earlier. He is now obtunded, with a temperature of 39.1 C (102.4 F) and has
muscular rigidity. He has had wide fluctuations in blood pressure over the past 12 hours.
Laboratory studies show a creatine phosphokinase in excess of 20,000 U/L and a
leukocyte count of 12,800/mm3. Supportive measures such as intravenous fluids and
airway support are given and the patient is noted to be hemodynamically stable. The
pharmacologic agent that would be contraindicated at this time is
A. acetaminophen
B. benztropine
C. bromocriptine
D. dantrolene
E. lorazepam
Explanation:
The correct answer is B. The patient has a diagnosis of neuroleptic malignant syndrome
due to rapid administration of high doses of intramuscular haloperidol. The patient's
relatively young age and male sex are predispositions for the development of neuroleptic
malignant syndrome, though the biggest risk for any patient is the rapid administration of
high potency neuroleptics. Because of the patient's fever and autonomic instability,
benztropine, an anticholinergic medication with a tendency to promote heat retention,
would be contraindicated in a patient with neuroleptic malignant syndrome.
Because of the high fevers associated with neuroleptic malignant syndrome, an
antipyretic such as acetaminophen (choice A) that is usually given parenterally may be
necessary.
Bromocriptine (choice C) is a dopamine agonist that is frequently used in the treatment of
neuroleptic malignant syndrome. One of the etiologic theories of neuroleptic malignant
syndrome is a dysregulation of dopamine receptors in the hypothalamus resulting in
impairment in body temperature control from overblockade of hypothalamic dopamine
receptors.
Dantrolene (choice D) is a muscle relaxant frequently used in the treatment of neuroleptic
malignant syndrome and may have efficacy in preventing renal failure secondary to
myoglobinuria.
Lorazepam (choice E) can be used in the treatment of neuroleptic malignant syndrome
both for its muscle relaxant and sedative qualities.
A 75-year-old man who lives in an elder care facility is brought to the
emergency department because of a sudden onset of severe, colicky
abdominal pain. He is generally in good health and does not take any regular
medications. Alcohol and cigarettes are not permitted in his living facility. His
temperature is 37 C (98.6 F), blood pressure is 110/70 mm Hg, pulse is
65/min, and respirations are 16/min. Physical examination shows abdominal
distention and high-pitched tinkling sounds. An abdominal x-ray shows a
dilated colon, forming an "omega loop," with the narrowed colonic segment
pointing to the right lower quadrant. Intravenous fluids are started and a
nasogastric tube is placed. The most appropriate management at this time is
to
A. admit him to the hospital and begin sulfasalazine and corticosteroids,
intravenously
B. admit him to the hospital and begin vancomycin and metronidazole,
intravenously
C. admit him to the hospital and observe
D. arrange for immediate laparotomy
E. perform a sigmoidoscopy with rectal tube placement
Explanation:
The correct answer is E. This patient most likely has a sigmoid volvulus, which
is caused by the bowel twisting on its mesentery, and leads to obstruction and
possibly vascular compromise. It typically presents with the acute onset of
abdominal pain and obstipation. Patients will have abdominal distention and
bowel sounds that are characterized by high-pitched "rushes and gurgles."
The abdominal x-ray shows a dilated colon forming a loop, with the narrowed
segment pointing to the obstruction site. Nonoperative reduction with a
sigmoidoscopy with rectal tube placement is generally effective in treating the
condition. Individuals in nursing homes and mental institutions have an
increased risk for developing a sigmoid obstruction, however, the reason is
not yet established.
Sulfasalazine and corticosteroids, intravenously (choice A) are used in acute
management of ulcerative colitis (UC), which is a disease that typically affects
younger individuals. It is characterized by fever, bloody diarrhea, and
abdominal pain. A barium enema typically shows a loss of haustral markings,
which is generally referred to as a "lead pipe" pattern. Complications include
toxic megacolon, perforation, colon cancer, and hemorrhage. This patient is
not the typical UC patient, and the clinical findings are more consistent with a
sigmoid volvulus than UC.
Vancomycin and metronidazole (choice B) are used to treat
pseudomembranous colitis, which is due to Clostridium difficile. It typically
occurs during or after the use of antibiotics and presents with severe watery
diarrhea. Treatment of pseudomembranous colitis is vancomycin or
metronidazole, given orally, to reach high levels in the stool. Treatment is
usually with one or the other medications, not both at the same time. This
patient is not taking any medications and the clinical findings are more
consistent with a sigmoid volvulus than pseudomembranous colitis.
It is inappropriate to admit him and observe (choice C), because he most likely
has a sigmoid volvulus, which is a colonic obstruction that requires immediate
intervention to prevent strangulation, vascular compromise, and infarction.
If a patient with a sigmoid volvulus presents with generalized abdominal pain,
fever, and hypovolemia, immediate laparotomy (choice D), may be necessary
because strangulation has probably already occurred. This patient has stable
vital signs, which makes this unlikely.
A 20-year-old comes to the clinic because of problem that has bothered her for "a while".
She has had difficulty throughout her academic career in spite of studying an average of
5 hours per night and taking many extra sessions of tutoring in her classes. She is not
able to concentrate due to having to continually check that the door in her apartment or
study area of the library is locked. She checks to see that it is locked an average of 15
times an hour while trying to study. She is fully aware each time she checks that the
door is locked behind her, but cannot resist the temptation to check to make sure. She
sometimes counts to 100 backward and forward after checking that her door is locked in
order to distract herself, however, this only provides temporary relief. There is no history
of abuse, no history of hallucinations, and the patient states overtly that she does not
fear for her safety while checking her door. An appropriate medication to treat this
patient's condition is
A. haloperidol
B. lithium
C. lorazepam
D. paroxetine
E. valproic acid
Explanation:
The correct answer is D. This patient suffers from obsessive-compulsive disorder and
demonstrates compulsive checking rituals that she acknowledges are not grounded in
reality-based concerns. The treatment of choice for obsessive-compulsive disorder is
higher dose selective serotonin reuptake inhibitors such as paroxetine, fluvoxamine,
sertraline, citalopram, and fluoxetine.
Haloperidol (choice A) is an antipsychotic medication that has no indication in the
treatment of obsessive-compulsive disorder.
Lithium (choice B) is used in the treatment of bipolar disorder and may be used as an
augmentation strategy for the treatment of unipolar depression. It has no indication,
however, for the treatment of obsessive-compulsive disorder.
Lorazepam (choice C) is an anxiolytic and antiepileptic medication that is effective for
treatment of some anxiety disorders such as panic disorder and generalized anxiety
disorder. However, it is not indicated in the treatment of obsessive-compulsive disorder.
Valproic acid (choice E) is an anticonvulsant used in the treatment of bipolar disorder. It
has no indication in the treatment of obsessive-compulsive disorder.
A 62-year-old man with a long history of cigarette smoking comes to the office with a 3month history of painless gross hematuria. Physical examination is unremarkable.
Urologic evaluation, including cystoscopy, reveals a medium-sized bladder tumor. You
recommend a surgeon to the patient and a transurethral resection is performed. The
pathology shows high-grade transitional cell carcinoma invading the muscularis propria.
A metastatic workup is negative and the patient is counseled regarding radical
cystectomy and urinary diversion. A radical cystectomy, pelvic lymph node dissection,
and ileal conduit are performed successfully. The surgical margins and lymph nodes are
all negative. An 18-month follow-up CT scan of the pelvis reveals a 4-cm
heterogeneous, contrast enhancing mass. A biopsy shows a high-grade transitional cell
carcinoma. The patient is referred to an oncologist who suggests chemotherapy using a
platinum-based regimen. He comes back to your office and tells you that he has heard
so many "horror stories" about chemotherapy and that he is concerned about the toxic
side effects of the recommended platinum-based regimen. He should be told that this
regimen significantly increases his risk of developing
A. cardiac toxicity
B. myelosuppression
C. nephrotoxicity
D. neurotoxicity
E. pulmonary toxicity
Explanation:
The correct answer is C. The mainstay of treatment for advanced, metastatic, or recurrent
urothelial carcinoma involves platinum based chemotherapy regimens, usually in
combination regimens. The most commonly used regimen is MVAC using methotrexate,
vinblastine, adriamycin, and cisplatinum. Unfortunately, toxicity often limits the usefulness
of these regimens. The toxicity most often associated with platinum is nephrotoxicity.
Acute tubular necrosis develops in approximately 25% of patients, and is often the doselimiting factor. This toxicity can be prevented by keeping patients well hydrated and using
diuretics during therapy.
Cardiac toxicity (choice A) is associated with doxorubicin (adriamycin). A cumulative
dose-related cardiomyopathy results from doxorubicin treatment and can be fatal.
Myelosuppression (choice B) is an adverse affect of many chemotherapeutic agents.
Most noteworthy are methotrexate, vinblastine, and doxorubicin.
Neurotoxicity (choice D) is associated with the vinca alkaloids, especially vincristine.
Vincristine can produce a dose-related mixed motor-sensory and autonomic neuropathy.
Pulmonary toxicity (choice E) is associated with bleomycin. Pulmonary fibrosis can result
from bleomycin and pulmonary function tests may be necessary in these patients.
A 37-year-old man is hospitalized in an intensive care unit after suffering
major head trauma in a motor vehicle accident. The patient was struck by a
car two days ago and was found on the scene to be non-responsive with a
Glasgow Coma Score of 4. He was intubated at the scene and transferred to
the hospital. In the prior two days, his mental status is unchanged and he
remains intubated and ventilated. You have documented a discussion you had
with the patient three months ago during an admission for pneumonia. During
that discussion the patient clearly stated that he would want to be maintained
on life support only if he were likely to regain a meaningful quality of life. You
and the medical team believe that he does not have a significant chance of
regaining an acceptable level of function. The brother and sister maintain that
the situation is reversible while the wife continues to desire the withdrawal of
care in fulfillment of her husbands' wishes. The most appropriate next step is
to
A. arrange a family meeting hoping to resolve the patient's previously
expressed wishes with those of the children and husband
B. begin the withdrawal of care despite the reservation of the siblings after
discussion with the hospital lawyer
C. consult a psychiatrist to speak to the siblings
D. notify the department of social services for the question of spousal
abuse
E. refer the case to the ethics committee for review
Explanation:
The correct answer is A. Although it is clear that you are ethically bound to
follow the patient's wishes, which in this care appears to be the withdrawal of
care, it is always better to do so with the resolve of the family. Often times,
feelings such as guilt will drive family members to insist on seemingly
unreasonable or inappropriate action. A thoughtful discussion, whereby the
family members are allowed to express their reservations in a supportive
setting, often will produce a resolution among previous disparate views.
While withdrawing care (choice B) appears to be what ultimately is the most
appropriate action, it is worth trying first to have all of the interested parties “on
the same page” before preceding if this resolution can be achieved in a timely
manner without causing suffering on the patient's part.
Whereas consult services such as psychiatry (choice C) may be helpful on
selected occasions, the first attempts to resolve the conflict should fall on the
primary medical team and primary care physicians.
While physicians have the positive duty to report suspected abuse, there is no
indication from the information present that the wife has abused her husband
(choice D).
Ethics committees (choice E) may be helpful in resolving conflict, but, again,
the primary medical providers should first attempt to resolve conflict in order to
respect the patient's wishes in a thoughtful and timely manner.
A young mother brings her 18-month-old son into your office for a well-child
examination. She is concerned that he "seems to be slow for his age." His
birth was without incident and his history is remarkable only for an episode of
otitis media at 7 months that responded well to antibiotics. He has a 5-year-old
sister at home and lives with both parents and a pet dog. When asked to
describe her concern more fully, the mother simply states, "He just isn't doing
the same things that his sister did when she was this age." She reports that he
is walking without help, gives hugs and kisses to family members, is able to
feed himself with a spoon, and has a vocabulary of about 15 words. However,
he is unable to climb stairs by himself, cannot turn a doorknob to open a door,
and shows no interest in potty training. His height is 65th percentile and
weight is 75th percentile. He does not speak to you, but answers simple
questions by nodding or shaking his head. You elicit no specific findings on
physical exam. The most appropriate response to the mother's concerns is:
A. "His lack of social involvement may indicate autism. I'd like to send you
to a pediatric psychiatrist for further evaluation."
B. "I think that your son should be referred to a developmental specialist
since I'm concerned that he shows some delay in motor skills."
C. "I'd like to perform some audiology tests since your son should be
speaking in short phrases by now."
D. "There is no need to be concerned; your son is right where he should be
as far as his development."
E. "You should begin toilet training immediately, he'll find it extremely
difficult if you don't start before age 2."
Explanation:
The correct answer is D. By the age of 18 months, most children will have
developed a specific set of skills which can be divided into: social, self-help,
gross motor, fine motor, and language. Social skills at this age include
greeting people by saying "hi" or something similar, giving hugs and kisses,
and playing patty-cake. Self-help involves drinking from a cup and feeding self
with a spoon. Gross motor skills include walking without help and beginning to
run. Fine motor skills would include scribbling with a crayon and stacking 2
blocks. Language skills should encompass talking in single words, asking for
food or drink with words, and following simple instructions. This patient is
doing well regarding each of these milestones.
"His lack of social involvement may indicate autism. I'd like to send you to a
pediatric psychiatrist for further evaluation" (choice A) is incorrect since the
patient does not exhibit any signs of autism.
Referral to a developmental specialist (choice B) is inappropriate since the
patient's motor skills are at a level with his age. Most children cannot walk up
and down stairs alone or open a door using a doorknob until 2 years of age.
An audiology referral (choice C) is incorrect. By 20 months of age a child
should know at least 10 words. This patient is able to use 15. Most children do
not speak in phrases until after age 2.
"You should begin toilet training immediately, he'll find it extremely difficult if
you don't start before age 2," (choice E) is incorrect since the majority of
children are not toilet trained until 3 years.
You are seeing a 23-year-old woman with diabetes for a routine office visit. Her regular
medications are glyburide and an oral contraceptive pill (OCP). She is an active smoker
and drinks about 4 shots of vodka each weekend. She eats "lots of meat and potatoes"
and exercises 2 times a week. During your conversation, she mentions that her sister
was recently admitted to the hospital with a "blood clot in her lung." On closer
questioning, you also find that her mother and maternal aunt have been previously
admitted for thromboembolic phenomena. She also mentions that they have been
diagnosed with a problem with their "factor V something." Based on this information, the
most important behavioral modification that you can recommend to reduce this patient's
thromboembolic risk is to
A. follow a low-fat diet
B. increase aerobic exercise
C. maintain tight blood sugar control
D. quit smoking
E. reduce alcohol intake
Explanation:
The correct answer is D. The risk of thromboembolic complications in women taking oral
contraceptive agents who also smoke, especially in the setting of a possible factor V
leiden mutation (to which the patient is hinting), has been estimated to be as high as 30
times the baseline risk. Therefore, given the available history, the patient should be
strongly counseled to give up smoking.
Following a low-fat diet (choice A) will help decrease her risk for coronary and vascular
disease in the setting of diabetes, but will do little to decrease the risk for thromboses.
Increasing aerobic exercise (choice B) will help the patient with her glucose control and
decrease her risk of coronary and vascular disease in the long run, but do little to
decrease her long-term risk for thromboses.
Tighter blood sugar control (choice C) will reduce the patients risk for complications from
diabetes such as large/small vessel disease and renal disease, but not affect the patient's
risk for developing thromboses.
Reducing alcohol use (choice E), while providing a diminished risk for alcohol related
complications, will not affect the patient's risk for developing thromboses.
A 29-year-old man comes to the office because one of his 3 sexual partners
recently had a Pap smear that showed dysplasia and koilocytic changes. Her
physician recommended that all of her sexual partners be evaluated. He has
always been healthy and has never had any sexually transmitted diseases. All
of his partners are "on the pill" so they do not use condoms. Physical
examination is completely unremarkable. There are no visible lesions on his
anogenital region. He is still very concerned that he has an infection that you
cannot see. The most appropriate next step is to
A. advise him to return if he develops any lesions
B. apply vinegar to his penis and scrotum
C. recommend that he use condoms during all sexual activity
D. send for a fluorescent treponemal antibody absorption (FTA-ABS)
serology
E. take random biopsies of the penis
F. tell him that he is healthy
Explanation:
The correct answer is B. This patient's girlfriend most likely has human
papillomavirus (HPV) infection, which is associated with dysplastic changes
and cervical cancer. This patient should be evaluated for an HPV infection,
and if there are no visible lesions, acetic acid (vinegar) should be applied to
the anogenital region to detect the presence of the virus. Invisible lesions
typically turn white when acetic acid is applied. This is thought to occur
because the acetic acid causes maceration and swelling of virally induced
epithelial hyperplasia, which usually has an increased glycogen content and
enhanced permeability. Even though this is not specific for HPV and falsepositives can occur, it may enhance the detection of an HPV infection.
If no lesions are found when acetic acid is applied, you should advise him to
return if he develops any lesions (choice A) and recommend that he use
condoms during all sexual activity (choice C). Condoms will probably not
completely prevent the spread of infection, but they should theoretically
reduce transmission.
Since his sexual partner most likely has an HPV infection and he is sexually
active with many partners, syphilis screening may be appropriate, but the
VDRL (Venereal Disease Research Laboratory) and RPR (rapid plasma
reagin) tests are used for screening, not the FTA-ABS (choice D). The FTAABS is more specific, but it is usually not considered a screening test because
it is more expensive and remains reactive in patients with a prior, treated
syphilis infection.
Taking random biopsies of the penis (choice E) is completely inappropriate,
and it will make a patient very unhappy. Acetic acid should be applied to help
see invisible lesions and biopsies can be taken from suspicious areas.
Since many patients infected with HPV have no visible signs and symptoms, it
is inappropriate to tell him that he is healthy (choice F) before further
evaluation (application of acetic acid).
A 16-year-old high school student is brought to the emergency department
because of severe right lower quadrant abdominal pain. She was at volleyball
practice when she suddenly doubled over in pain. She has no significant past
medical history and had a few episodes of right-sided abdominal pain in the
last week. She says that the earlier episodes of pain were much less severe.
She is sexually active with one partner, her boyfriend, and they use condoms
for birth control. Her temperature 38.8 C (101.8 F), pulse is 90/min, blood
pressure is 120/85 mm Hg, and respirations are 14/min. On physical
examination she appears very uncomfortable and is in obvious pain.
Abdominal examination is significant for focal exquisite tenderness and
guarding in the right lower quadrant. Leukocyte count is 11,200/mm3 with 69%
segmented neutrophils and 2% band forms. The most appropriate test or
study at this time is
A. lower gastrointestinal barium enema
B. ultrasonography of the appendix
C. ultrasonography of the pelvis
D. upper gastrointestinal barium study with small bowel follow through
E. urinalysis
Explanation:
The correct answer is B. Severe pain localized to the right lower quadrant,
fever, and leukocytosis with bandemia are classic signs and symptoms of
acute appendicitis. Acute appendicitis is the most common cause of acute
abdomen. The initial study of choice to make this diagnosis is ultrasonography
of the appendix. Ultrasound signs of acute appendicitis are a dilated (greater
than 6 mm), non-compressible appendix, and visualization of an
appendicolith. An appendicolith is calcified fecal material that gets trapped
within the appendix and is usually the cause of appendicitis. The sonographic
finding of an appendicolith is very specific for appendicitis. If ultrasonography
does not reveal such findings, the next appropriate step is a CT of the
abdomen and pelvis. A surgery consult should also be called.
Lower gastrointestinal barium enema (choice A) is not the best study in the
work up of acute appendicitis. Complete filling of the appendix to the bulbous
tip with barium is strong evidence against appendicitis. Nonfilling of the
appendix due to an obstructing appendicolith is what you would expect to see
in acute appendicitis. However, even in normal barium enema studies the
appendix does not always fill. Furthermore, this is a fairly invasive and time
consuming diagnostic study which is not appropriate in this case of an acute
abdomen.
Ultrasonography of the pelvis (choice C) is not indicated in this patient at this
time. Ultrasonography of the pelvis is useful for evaluating the uterus and
ovaries. Although she is sexually active, her symptoms do not suggest a
disease process involving these pelvic organs. The differential diagnosis for
women of child bearing age who present with right lower quadrant pain
includes acute appendicitis, ruptured ovarian cyst, and pelvic inflammatory
disease. The latter two disease processes, however, are not associated with
fever and leukocytosis.
Upper gastrointestinal barium study with small bowel follow through (choice D)
is not indicated in this patient. This examination is used to evaluate the
esophagus, stomach, and small bowel. Common indications for this diagnostic
study include dysphagia, symptoms of peptic ulcer disease, and
gastroesophageal reflux.
Urinalysis (choice E) will not provide useful information in the evaluation of this
patient. She has classic signs and symptoms of acute appendicitis and a
urinalysis will not be significant. Urinalysis is indicated when the diagnosis of a
urinary tract infection is suspected. Symptoms of cystitis include urinary
frequency and urgency. These symptoms associated with flank pain and fever
are suspicious for pyelonephritis.
A 71-year-old man undergoes transurethral resection of the prostate (TURP)
for benign prostatic hyperplasia (BPH) under spinal anesthesia. Towards the
end of the resection the patient begins to complain of visual color changes. He
is alert and oriented and his blood pressure and heart rate have been stable
throughout the procedure. The procedure is aborted and the patient is taken to
the recovery room. In the recovery room the patient's blood pressure is
160/110 mm Hg and pulse is 52/min. He is lethargic and confused. The most
likely explanation for his symptoms is
A. anemia
B. hypercalcemia
C. hyperkalemia
D. hypermagnesemia
E. hypernatremia
F. hypocalcemia
G. hypokalemia
H. hypomagnesemia
I. hyponatremia
Explanation:
The correct answer is I. This patient is suffering from TUR syndrome. This
syndrome is caused by the systemic absorption of irrigating fluids through the
venous sinuses that are opened during TURP. This leads to hypervolemia and
hyponatremia. TUR syndrome occurs in approximately 2-10% of all cases
performed and its mortality rate is 0.2-0.8%. Changes may be seen in the
cardiopulmonary, hematologic, renal, and central nervous systems in varying
degrees. The most common signs and symptoms include hypertension,
bradycardia, nausea and vomiting, mental confusion, blurry vision, lethargy,
and respiratory distress. The most severe cases can lead to shock, acute
renal failure, seizure, coma, and death. The severity of symptoms is usually
low until approximately 2 liters of fluid are absorbed, and then can become
severe if more than 3 liters are absorbed. Treatment of TUR syndrome
requires prompt recognition of symptoms and expeditious termination of the
procedure. Delayed absorption may prevent symptoms from manifesting for
up to 24 hours. Mental status changes do not occur until the serum sodium is
less than 125 mEq/L. Most cases can be treated with the administration of
intravenous normal saline and loop diuretics. Sodium correction should be
done no faster than 1.5 mmol/L/hr to prevent the risk of central pontine
myelinolysis. In the most severe cases intravenous 3% NS may be
administered along with loop diuretics. Because of the risk for TUR syndrome,
it is important to monitor serum electrolytes at all times. If suspicion for the
syndrome is high then sodium levels may be checked during resection. If
clinical concern is minimal then electrolyte evaluation may be deferred until
the patient reaches the recovery room.
While bleeding and anemia (choice A) is an obvious risk associated with
TURP, this would present with hypotension and tachycardia. Currently, the
transfusion rate with TURP is approximately 3.5%.
Hypercalcemia (choice B), hyperkalemia (choice C), hypermagnesemia
(choice D), hypernatremia (choice E), hypocalcemia (choice F), hypokalemia
(choice G), and hypomagnesemia (choice H) are not commonly seen in
patients undergoing TURP.
A 70-year-old man with hypertension, hyperlipidemia, and chronic atrial
fibrillation is brought to the emergency department for confusion. He was
recently diagnosed with multiple myeloma. His medications include
furosemide, captopril, atorvostatin, digoxin, and warfarin. He is allergic to
penicillin to which he gets a rash. His temperature is 37.0 C (98.6 F), blood
pressure is 100/60 mmHg, pulse is 98/min, and respirations are 23/min.
Physical examination shows an irregular cardiac rhythm and a soft systolic
murmur at his cardiac base. An electrocardiogram shows atrial fibrillation.
Laboratory studies show:
Sodium
143 mEq/L
Potassium
4.5 mEq/L
Chloride
104 mEq/L
Bicarbonate
26 mEq/L
Calcium
13 mg/dL
Glucose
109 mg/dL
The most appropriate next step is management is to
A. administer albumin, intravenously
B. administer a dextrose bolus followed by insulin, intravenously
C. administer magnesium sulfate, intravenously
D. administer pamidronate, intravenously
E. hydrate him with normal saline and then administer furosemide,
intravenously
Explanation:
The correct answer is E. The most appropriate acute treatment of
hypercalcemia is hydration followed by a forced diuresis. It is important to
hydrate patients prior to administering the diuretic since most patients with this
condition are hypovolemic from hypercalcemia induced nausea/vomiting and
diabetes insipitus.
Administration of albumin (choice A), albeit a binder of serum calcium, has no
role in the management of hypercalcemia.
Administration of dextrose followed by insulin (choice B) is one of the
treatments of choice for acute hyperkalemia. It has no role in the management
of hypercalcemia.
Administration of magnesium sulfate (choice C) has no role in the
management of hypercalcemia.
Administration of pamidronate (choice D), a bisphosphonate which can
decrease bone resorption, can be used in the chronic management of
hypercalcemia, but has no role in its acute management.
You are seeing a 63–year-old man on rounds in the medical intensive care
unit who was admitted with sepsis related to an infected diabetic foot ulcer.
During his admission, he has had multiple complications including respiratory
failure, a large perioperative myocardial infarction during a left below the knee,
amputation, and atrial fibrillation, which resulted in an embolic stroke. He has
been intubated and ventilator dependent since admission. Over the past 3
days his condition has been slowly improving and he is starting to regain
consciousness. He now indicates that he is having pain in his scrotum. His
temperature is 37.0 C (98.6 F), blood pressure is 112/76 mm Hg, pulse is
92/min, respirations are 22/min (on ventilator). His jugular veins are distended,
and his heart is irregularly irregular with an S3 gallop. His lungs have course
breath sounds bilaterally, abdomen is mildly distended, and his scrotum is
markedly and symmetrically enlarged to approximately four times normal size.
There is 4+ pitting edema in the lower extremities bilaterally. An ultrasound of
the scrotum is performed which shows normal testes and diffuse thickening of
the scrotal skin and a small to moderate sized hydrocele on the left and a
small hydrocele on the right. The most appropriate course of treatment for his
scrotal pain is
A. ciprofloxacin 500 mg via nasogastric tube twice daily for 14 days
B. diuresis as tolerated by his volume status
C. no specific treatment would help
D. percutaneous aspiration of the hydroceles
E. percutaneous aspiration of the hydroceles followed by placement of
drainage tubes bilaterally to prevent reaccumualtion
Explanation:
The correct answer is B. Many patients who are volume overloaded for
various reasons, whether it is due to massive volume resuscitation or
congestive heart failure, will develop some degree of scrotal edema. Often
times, it can be very impressive and can also cause the patient pain. This
patient definitely has signs of heart failure and has likely been heavily volume
loaded, because of his sepsis. The only real treatment is to optimize the
patient's volume status and let the body reabsorb the fluid with diuresis as
tolerated.
There are no signs of infection mentioned in the clinical scenario, thus
treatment with an antibiotic is not necessary (choice A). Occasionally these
patients will get some cellulitic type symptoms in the scrotum, but the
treatment of choice for that would be something other than ciprofloxacin.
No specific treatment would help (choice C) is incorrect as he should be
treated with diureses to improve his volume status.
The hydroceles seen in this patient are not likely to be contributing to the
markedly increased size of the testicles, and percutaneous aspiration (choice
D) is not necessary. Placing a drainage tube in the scrotum (choice E) is
completely unnecessary and not done.
A 42-year-old woman comes to the office because of the sudden onset of
"blurry vision" in her left eye. She says that she wears glasses regularly to see
distant objects, but now she is seeing "blurry" in both near and distant objects.
She has moderate pain, but denies any nausea or vomiting. She does not
have any significant past medical history and does not take any medications.
The most appropriate method for performing the ophthalmologic examination
is to
A. ask the patient to look directly at your nose as you approach with the
ophthalmoscope
B. hold the ophthalmoscope in your left hand when examining the patient's
left eye
C. hold the ophthalmoscope 6 inches in front of your face as you approach
the patient's eye
D. hold the ophthalmoscope up to your right eye as you examine the
patient's left eye
E. press the ophthalmoscope against the patient's cheek and keep the
hand that is holding the instrument in the air
Explanation:
The correct answer is B. When performing an ophthalmologic examination you
should hold the ophthalmoscope in your left hand, up to your left eye when
examining the patient's left eye, and in the right hand when examining the
patient's right eye. If you picture this, you will realize that this allows you to
stand on the patient's side and invade their personal space as little as
possible. If you hold the instrument in your right hand, up to your right eye and
examine the patient's left eye, you need to stand directly in front of them, and
basically on top of them. This is totally inappropriate. It will make you and the
patient very uncomfortable.
You should tell the patient to look straight ahead and stare at a distant target,
not to look directly at your nose as you approach with the ophthalmoscope
(choice A).
You should not hold the ophthalmoscope 6 inches in front of your face as you
approach the patient's eye (choice C). You should function as a single unit
with the instrument by holding it against your own forehead as you approach
her. This allows you to have the most coordinated actions, and allows you to
visualize the red reflex as you approach.
You invade the patient's personal space by holding the ophthalmoscope up to
your right eye as you examine the patient's left eye (choice D). You should
hold the instrument in your left hand to your left eye when examining the
patient's left eye, and then switch to your right hand and eye to examine the
patient's right eye. Also, you cannot really be as stable and visualize the eye
as well if you hold the instrument in your left hand, up to your right eye as you
examine the patient's left eye. And even then you invade their personal space
more than if you held it in left hand to your left eye to examine the patient's left
eye.
You should not press the ophthalmoscope against the patient's cheek and
keep the hand that is holding the instrument in the air (choice E). The
instrument should be held against your forehead and you can gently hold the
hand with the scope against the patient's cheek, but not the instrument, for
stabilization. You are not very stable if you keep the hand with the instrument
in the air.
You are called to see a 45-year-old nursing home resident, who has been there since a
motor vehicle accident that left him paralyzed from the neck down 2 months ago. He
denies any active medical problems prior to his car accident 2 months ago. In the past
month, he has noticed a rash on his back that is occasionally pruritic. He denies any
systemic manifestations associated with the rash. He is confined to his bed and the
nursing staff turns him to his side once per day by propping him back with multiple
pillows. He has notable atrophy of all the extremities. Cutaneous examination reveals
numerous non-folliculocentric inflammatory papules distributed over his posterior trunk.
There is no involvement of the anterior trunk, extremities, face, or oral mucosa. The most
appropriate management of this patient's condition is to
A. prescribe oral minocycline
B. prescribe a topical corticosteroid
C. prescribe a topical tretinoin
D. recommend a topical benzoyl peroxide
E. tell the nurses to rotate the patient on his sides more frequently and keep his room
at a cooler temperature
Explanation:
The correct answer is E. Rotating the patient and keeping the room at a cooler
temperature is correct because miliaria, also known as heat rash, is a common
phenomenon in individuals during prolonged bedrest. It usually involves the posterior
trunk of a bedridden patient. The eccrine sweat duct occlusion is the initial event. The
duct ruptures, leaks sweat into the surrounding tissues, and induces an inflammatory
response. The papular and vesicular lesions resemble folliculitis with one major
distinguishing feature. They are not follicular and therefore, do not have a penetrating hair
shaft. The best treatment for miliaria is to keep the areas involved cool and avoid
occlusive clothing.
Oral minocycline (choice A), topical tretinoin (choice C), and topical benzoyl peroxide
(choice D), are incorrect because these are treatments for folliculitis or acneiform
eruptions. Since the inflammatory papules on this patient are described as nonfolliculocentric, folliculitis is not likely the diagnosis.
A topical corticosteroid (choice B) is incorrect because use of corticosteroids topically
may result in miliaria as an adverse reaction.
You are called to the well-baby nursery after a 27-year-old patient of yours,
delivers a healthy- appearing term baby boy by normal spontaneous vaginal
delivery. You recall that this patient had an uncomplicated prenatal course,
however, initial laboratory studies during her first antenatal visit were positive
for hepatitis B surface antigen. A complete physical examination of the infant
is unremarkable. You should
A. advise the mother not to breast-feed her newborn
B. give routine care to the newborn
C. give the hepatitis B vaccine and hepatitis B immune globulin to the
newborn
D. reassure the mother that no hepatitis B vaccine or treatment is needed
for her newborn
E. send hepatitis serology from the newborn and await results before
providing care
Explanation:
The correct answer is C. Infants born to mothers who are HBsAg positive
should be given the hepatitis B vaccine and hepatitis B immune globulin at
different sites within 12 hours of birth. The second and third doses of the
vaccine should be given at 1 month and 6 months. They should be tested for
anti-HBs and HBsAg 1-3 months after completing the immunization series.
Advising the mother not to breast-feed her child (choice A) is incorrect.
HBsAg-positive mothers can breast-feed because according to the American
Academy of Pediatrics and the Report of the Committee on Infectious
Diseases, the infant is at no additional risk of acquiring the virus.
Routine care (choice B), which is given to infants born to HBsAg-negative
mothers, would be to give a series of 3 hepatitis B vaccines. The first dose at
birth to 2 months, the second dose 1 month later, and the third dose at least 4
months after the first dose and 2 months after the second dose, but not before
6 months of age. This infant is born to a HBsAg-positive mother and so he
also requires the hepatitis B immune globulin to prevent infection.
Reassuring the mother that no hepatitis B vaccine or treatment is needed for
her newborn (choice D) is inappropriate because as stated above, this infant
should be given the hepatitis B vaccine and hepatitis B immune globulin.
It is incorrect to send hepatitis serology from the newborn and await results
before providing care (choice E). He should be given the hepatitis B vaccine
and hepatitis B immune globulin as soon after birth as possible. You should
not wait until any laboratory studies return.
A 53-year-old widowed woman comes to the office for a health maintenance
examination. She is a new patient who recently moved to your city after her
husband died in an office fire 6 months ago. She says that she has no
complaints, except for a cough that she began to notice 4 months ago. She
denies nasal discharge, "a tickle in the throat," frequent throat clearing,
heartburn and the sensation of regurgitation, fever, sputum production,
cigarette smoking, illegal drug use, sexual activity, occupational exposures,
and any other symptoms associated with a respiratory infection. She says that
the cough is not seasonal or associated with wheezing. Her temperature is
37.0 C (98.6 F), blood pressure is 135/90 mm Hg, pulse is 70/min, and
respirations are 14/min. Physical examination is unremarkable. The most
appropriate next step is to
A. order an electrocardiogram
B. order an x-ray of the chest
C. question her about medications
D. refer her for fiberoptic bronchoscopy
E. schedule her for pulmonary function tests
Explanation:
The correct answer is C. This patient has a chronic cough, which is usually
considered chronic because it is lasting more than 3 weeks. It may be due to a
variety of things. However, the important lesson in this question is that before
you turn to diagnostic studies you need, to make sure that you have obtained
a detailed history. The case history will provide the answer to almost every
question that you will need to ask her, except what medications she takes.
Since she is a new patient, you will need to find out if she is taking an ACE
inhibitor, such as captopril or enalapril, which is a frequent cause of a chronic
cough in hypertensive patients. They cause a cough in up to 20% of people
taking them. The exact mechanism is unknown, but it is thought to somehow
be related to bradykinin and substance P. The treatment for the cough is the
discontinuation of the ACE inhibitor.
An electrocardiogram (choice A) is unnecessary at this time in this patient,
complaining of a chronic cough. She is not complaining of chest pain and
there is nothing in her history that suggests an arrhythmia. The most important
next step, is to take a detailed history before you order diagnostic tests.
An x-ray of the chest (choice B) may be appropriate in the near future, but it is
not the next step at this time. Before you order diagnostic studies, you need to
make sure that you ask her any questions that might help you figure out the
etiology of her cough. Asking her about medications is very important because
ACE inhibitors cause a chronic cough in up to 20% of patients taking this
medication.
A fiberoptic bronchoscopy (choice D) is used to obtain histologic and cytologic
specimens and to visualize an endobronchial tumor. Before you turn to such a
specialized study, you need to first obtain a detailed history. If the patient is
not taking an ACE inhibitor, a chest x-ray should usually be performed, and if
this is abnormal, sputum cytology, a high resolution CT scan, and fiberoptic
bronchoscopy should be considered.
Pulmonary function tests (choice E) are used to assess airway
hyperresponsiveness for patients in which you suspect asthma, and lung
volumes and diffusion capacity in patients in which you suspect a diffuse
interstitial lung disease. A detailed history is necessary before using any of
these studies.
A 73-year-old man with emphysema comes to the clinic with complaints of
food getting stuck when he swallows, which has been getting progressively
worse over the last 8 months. He denies problems swallowing liquids and
thinks he has lost about 5 pounds. He used alcohol heavily for many years but
quit drinking 10 years ago. He still smokes 1 pack of cigarettes per day and
has done so since age 20. He uses albuterol, steroid inhalers and
theophylline. His blood pressure is 123/73 mm Hg, pulse is 87/min, and
respirations are 20/min. Physical examination reveals bilateral scattered
wheezes in the lungs. A chest x-ray shows hyperexpansion and no nodules.
The most appropriate next step in management is to
A. order a barium esophagram
B. order an esophageal manometry
C. order an esophageal pH probe
D. treat with omeprazole and follow up in 3 months
E. treat with ranitidine and follow up in 3 months
Explanation:
The correct answer is A. This patient most likely has an esophageal
squamous cell carcinoma (the most common type of esophageal malignancy).
In any patient with dysphagia that is progressive for only solids, it suggests a
growing and obstructive lesion. The history of tobacco and alcohol use, puts
this person at a much higher risk of carcinoma. The two ways to diagnose this
are a barium swallow study, which will show the mucosal mass, or an upper
endoscopy study to directly visualize and biopsy the lesion.
Esophageal manometry (choice B) is used to evaluate dysphagia caused by
motility disorders. These typically present with dysphagia for solids and liquids
and may or may not be progressive.
A pH probe (choice C) is used to evaluate esophageal reflux disease, which
does not in itself typically cause dysphagia, but over long periods of time will
increase the risk of esophageal adenocarcinoma.
Both omeprazole (choice D) and ranitidine (choice E) are used to treat
symptoms of gastroesophageal reflux disease and would not address his
dysphagia. Furthermore, waiting 3 months to see the patient again would be
inappropriate.
A 26-year-old woman comes to the office for routine postpartum care after
delivering a healthy baby boy 6 days earlier. This is her first child and she is
concerned about her breastfeeding, diaper changing, and bathing techniques.
You ask her to explain how she performs these activities, and it seems as if
she is doing everything correctly. She then tells you that she is very upset
about her weight, that she gained 45 pounds during the pregnancy, and now
is still 37 pounds "overweight". Her husband is a good friend of yours from
medical school. Her blood pressure is 140/90 mm Hg and her pulse is 95/min.
Physical examination shows dilated pupils, but is otherwise unremarkable.
You suspect that she is using drugs and so you ask her in a direct,
nonjudgmental way. She admits to using cocaine twice since the delivery of
her son. The most appropriate next step is to
A. advise her to stay away from her son when she is using cocaine
B. call her husband and ask if he knows that she is using drugs
C. contact the child welfare association and inform them of your findings
D. obtain a urine sample to test for cocaine and other illegal drugs
E. tell her that she should not breast feed her son if she is using cocaine
Explanation:
The correct answer is E. Cocaine is contraindicated during breast feeding and
so you need to advise the mother to stop breast feeding if she is using
cocaine. Possible effects of cocaine in the breast milk include growth
retardation and neurologic damage. Cocaine use during pregnancy is
associated with fetal demise, spontaneous abortions, premature rupture of
membranes, preterm labor and delivery, placental abruption, intrauterine
growth retardation, and congenital anomalies. Other medications and drugs
that are contraindicated during breastfeeding are bromocriptine,
cyclophosphamide, cyclosporine, doxorubicin, lithium, ergotamine,
methotrexate, amphetamines, heroin, marijuana, nicotine, and phencyclidine.
The most appropriate next step is to tell her is to discontinue breastfeeding if
she is using cocaine, not to advise her to stay away from her son when she is
using cocaine (choice A). This answer makes it seem like it is okay that she is
using drugs, but she should just not be near her baby. This is actually not true.
If she is using the cocaine to lose weight, you should recommend a healthy
diet and exercise regimen.
It is incorrect to call her husband and ask if he knows that she is using drugs
(choice B) or to contact the child welfare association and inform them of your
findings (choice C). You should first try to reason with the patient and
encourage her to discontinue using cocaine and tell her that she is not only
risking her own life now, but also that of her son's because she is breast
feeding.
She already admitted to you that she is using cocaine and so it is unnecessary
to obtain a urine sample to test for cocaine and other illegal drugs (choice D).
An 18-year-old boy with Hodgkin's lymphoma is admitted to the hospital for his
third course of chemotherapy which includes prednisone. On his 6th hospital
day, you are called by the nurse to assess a new rash which has slowly
progressed to cover the entire anterior and posterior trunk, as well as his arms
and part of the face. The nurse is concerned about a possible allergic
reaction, secondary to the multiple medications this patient is taking. On
examination, you note numerous, small, pinpoint pustules and inflammatory
papules of the same stage scattered mostly over his trunk and proximal arms.
There are some pustules of the same stage over his forehead as well. The
patient denies any significant history of acne prior to the initiation of
chemotherapy. At this time the most correct statement about his condition is:
A. This is acne vulgaris and should be treated with oral minocycline and
topical benzoyl peroxide
B. This is a drug eruption and you should stop all the medications and
reinitiate one every week
C. This is a drug eruption secondary to his chemotherapy and topical
corticosteroid will alleviate the problem
D. This is miliaria secondary to the occlusion effect by his hospital gown
E. This is steroid acne and topical tretinoin cream may help
Explanation:
The correct answer is E. Acneiform eruptions is correct because it is
characterized by papules and pustules resembling acne lesions, not
necessarily confined to the usual sites of acne vulgaris. The eruptions are
distinguished by their sudden onset. Oral medications such as iodides,
bromides, testosterone, cyclosporine, antiepileptic medications, lithium, and
systemic corticosteroids are common agents that can lead to acneiform
eruption. When medium or high doses of corticosteroids are taken for as short
a time as 3-5 days, a distinctive eruption may occur, known as steroid acne. It
is a sudden out-cropping of inflamed papules, most numerous on the upper
trunk and arms, but also seen on the face. The lesions typically present as
papules rather than comedones. Tretinoin cream applied once or twice daily
may clear the lesions within 1-3 months, despite the continuation of high
doses of corticosteroid.
Acne vulgaris (choice A) is incorrect because of the atypical distribution
(proximal arms), and the sudden onset of numerous lesions.
An allergic drug eruption (choice B) is incorrect because typical drug eruptions
do not present with pustules. His medication should not be terminated.
Typical drug eruptions do not present with pustules and topical steroids
(choice C), especially of the fluorinated types, or when applied under
occlusion, may also induce an acneiform eruption and worsen this patient's
skin problem
Miliaria (choice D) is incorrect. This is retention of sweat as a result of the
occlusion of eccrine sweat ducts and pores, producing an eruption that is
common in patients with prolonged bedrest. These patients typically present
with numerous folliculocentric inflammatory papules on the posterior trunk,
resulting from extended periods of being on their back. The face, central
chest, and arms are typically not occluded in bedrest patients.
A 45-year-old disheveled man is brought to the emergency department by
paramedics stating, “I can't see!” The paramedics tell you that he is a
homeless man who was found this morning locked in an automotive garage
when the manager of the garage went to open for business. They also state
that he was found next to an empty bottle of window washing fluid and an
empty 40oz beer can. The patient is alert, but uncooperative during
questioning and keeps repeating “Doctor... I just can't see!” As you are
examining him, he begins to retch, holds his abdomen and vomits twice. His
temperature is 37 C (98.6 F), blood pressure is 150/92 mm Hg, pulse is
100/min, respirations are 24/min, and oxygen saturation is 97% on room air.
His pupils are dilated at 5mm and are only sluggishly reactive. Funduscopic
examination reveals hyperemia of both discs without papilledema; the cranial
nerves are otherwise intact. The remainder of the physical examination is
unremarkable except you notice that he is taking very deep and rapid breaths.
The most appropriate first diagnostic step is to
A. administer intravenous ethanol and monitor for reversal of his visual
deficit
B. observe him in the emergency department as this man is obviously
intoxicated with alcohol and just needs to sleep it off
C. order arterial blood gas and serum electrolytes and osmolality
D. perform a CT scan of his head to rule out intracranial pathology
E. request an immediate ophthalmology consult
Explanation:
The correct answer is C. This patient presents with the classic signs and
symptoms of acute methanol toxicity. Methanol is commonly found in
windshield washing fluid, solvents, and paint thinners. Methanol is converted
by alcohol dehydrogenase into formaldehyde, which is then converted into
formic acid, which produces an anion gap metabolic acidosis. The formic acid
also affects the optic nerve function resulting in optic papillitis and retinal
edema leading to blindness. An arterial blood gas would help you determine
the pH and amount of acidemia, with an associated low bicarbonate level and
likely low pCO2 level from respiratory compensation. The respiratory
compensation for a metabolic acidosis would explain his rapid respiratory rate
at 24/min. The serum electrolyte panel will help determine the anion gap using
the formula (Anion Gap = Na-Cl-HCO3). The anion gap is normally less than
12. Methanol poisoning also produces an osmol gap which is the difference
between the measured osmolality and calculated osmolarity.
The normal measured serum osmolality is 280-295mOsm (milliosmols). The
calculated osmolarity can be determined using the formula [osmolarity =
(2xNa)+(BUN/2.8)+(glucose/18)]. An osmol gap of greater than 10mOsm is
abnormal and indicates that osmotically active substances are present in the
blood (methanol, ethylene glycol, isopropyl alcohol).
Administration of IV Ethanol (choice A) may be necessary as a treatment for
methanol toxicity, as well as possible dialysis. However, diagnostic studies
need to be performed first to confirm the diagnosis.
Observation without intervention (choice B) is improper management as this
patient warrants further investigation.
A CT scan of the head (choice D) is likely to be normal in this patient, and thus
of minimal benefit in ascertaining a diagnosis.
An ophthalmology consultation (choice E) will help rule out other pathological
causes of blindness, but should not be your first step in the management of
this particular patient.